Освіта та самоосвіта

Реферати, дослідження, наукові статті онлайн

Відповіді до екзамену «Порівняльна адвокатура»

  1. Поняття порівняльного правознавства. Основні завдання курсу «Порівняльна адвокатура»

Порівняльне правознавство, або юридична компаративістика (лат. comparativus — порівняльний), — наука, яка займається дослідженням загальних і специфічних закономірностей виникнення, розвитку і функціонування сучасних правових систем світу.

Об’єктивно необхідним є процес пізнання права в загальносвітовому порівняльному аспекті (між різними правовими системами світу), тобто міжнаціональне порівняння, порівняння різних правових систем, існуючих у державах світового співтовариства. В основі такого порівняння лежать процеси, що закономірно відбуваються у світовому співтоваристві — розроблення і формування правових систем молодих держав, розширення і поглиблення зв’язків між державами і групами країн, інтеграція низки країн у єдине ціле. Порівняння уможливлює класифікацію державно-правових явищ, властивих правовим системам різних країн, з’ясування їх історичної послідовності, генетичних зв’язків між ними, ступінь запозичення елементів (норм, принципів, форм права) однієї правової системи в іншої.

Порівняльне правознавство доцільно розмежовувати на загальне і спеціальне (галузеве і внутрішньогалузеве).

Загальне порівняльне правознавство. Предметом порівняння є правові системи в цілому, загальні та специфічні закономірності виникнення, розвитку і функціонування правових явищ (законодавства, системи права, співвідношення права і закону, правовідносин, тлумачення, законодавчої техніки та ін.), тобто все те, що належить до загальнотеоретичного знання правового поля держави

Спеціальне (галузеве і внутрішньогалузеве) порівняльне правознавство. Предметом порівняння є окремі компоненти (елементи) системи права — галузі: цивільне, кримінальне, адміністративне право та ін., інститути права, ї конкретні норми, тобто все те, що належить до спеціального знання галузевої юридичної науки.

Порівняльна адвокатура відноситься до спеціального виду порівняльного правознавства. Цей курс запроваджений з метою та завданням більш глибокого розуміння ролі й функції адвокатур в Україні для чого варто звернутися до великого досвіду устрою адвокатури в інших країнах, а також вивчення систем адвокатури в закордонних країнах допомагає прогнозувати розвиток інституту адвокатури в нашій країні.

  1. Місце порівняльного правознавства у системі юридичних наук.

Науку порівняльного правознавства слід відрізняти від порівняльно-правового методу. Порівняльно-правовий метод — це зіставлення юридичних понять, явищ, процесів одного порядку і з’ясування подібності та відмінності між ними. Залежно від об’єктів порівняльне-правовий метод застосовується вибірково за обов’язкової умови їх порівнянності.

Предмет науки порівняльного правознавства — загальні і специфічні закономірності виникнення, розвитку і функціонування правових систем світу в їх порівняльному пізнанні. Порівняльне пізнання може походити як мінімум із двох правових систем. Важливо підкреслити, що предмет порівняльного правознавства як самостійної науки не тільки містить у собі процес порівняння окремих нормативно-правових актів, галузей права або інститутів, а й охоплює правові системи в цілому. Порівняння припускає діяльність, у ході якої відповідні елементи порівнюваних правових систем зіставляються для того, щоб визначити наявні між ними подібності і відмінності.

Важливою особливістю загального порівняльного правознавства є його орієнтація передусім на дослідження стикових проблем, що лежать у площині порівняльного аналізу різних правових систем світу.

Об’єктами загального порівняльного правознавства можуть бути: 1) сучасні типи правових систем (загальне міжтипове порівняння), 2) правові системи різних типів (окреме міжтипове порівняння); 3) правові системи, що належать до одного типу (внутрішньотипове порівняння).

Загальне міжтипове порівняння припускає з’ясування ідентичності і своєрідності прояву їх загальних ознак. На рівні порівняння окремих правових систем (окреме міжтипове порівняння і внутрішньотипове порівняння) виявляється єдність загальних, особливих і специфічних (таких, що відрізняються) ознак. Все це сприяє їх цілісному сприйняттю.

Наука порівняльного правознавства покликана охопити порівнянням правові системи, виявити сучасну правову карту світу, де, по можливості, право кожної країни віднесено до певного типу (сім’ї) правових систем. Отже, одним з основних завдань науки порівняльного правознавства є наукова класифікація правових систем світу.

  1. Класифікація правових систем світу.

Типологія (або наука класифікації) — вчення про типи правових систем. Типологія і порівняльний метод перебувають у тісному зв’язку. З одного боку, порівняння припускає попереднє встановлення типології, з іншого — встановлення типології без порівняння неможливе. Питаннями типології правових систем, їх порівняльним вивченням займаються фахівці в галузі науки порівняльного правознавства. Існує чимало класифікацій правових систем. Запропоновано декілька варіантів критеріїв, за якими вони об’єднуються у типи (сім’ї). Кожний із них заслуговує на увагу. Однією з найбільш популярних виявилася класифікація правових сімей, наведена Р. Давидом у книзі «Основні правові системи сучасності» (1953). Вона ґрунтувалася на поєднанні двох критеріїв: ідеології, яка включає релігію, філософію, економічні і соціальні структури; юридичної техніки, яка включає джерела права як основний елемент.

Німецькі вчені К. Цвайгерт і Г. Кьотцпоклали в основу класифікації правових систем критерій «правового стилю». «Стиль права» складається, на думку авторів, із п’ятьох чинників: 1) походження і еволюція правової системи; 2) своєрідність юридичного мислення; 3) специфічні правові інститути; 4) природа джерел права і засоби їх тлумачення; 5) ідеологічні чинники.

З урахуванням наявних точок зору на цю проблему можна навести таку сукупність критеріїв, що визначають класифікацію правових систем світу.

  1. Спільність історичних коренів виникнення і подальшого розвитку (включаючи ступінь рецепції римського права). Правові системи об’єднані природою їх розвитку, генетичними коренями, що знаходяться в тій самій стародавній державі. Вони розвивалися в подібних умовах і зберегли однаковість у доправових регуляторах, перших джерелах, характері і ступені сприйняття джерел права інших національних правових систем, їх об’єднують історичні пам’ятники права, тенденції виникнення і розвитку державних і правових структур, а головне — ґрунтування на тих самих правових началах, принципах, нормах.
  2. Спільність основного юридичного джерела права (форми права) — нормативно-правовий акт, нормативно-правовий договір, правовий прецедент, правовий звичай, релігійно-правова норма. Роль, значення і співвідношення цих джерел права неоднакові в різних правових системах та їх типах.
  3. Єдність у структурі системи права і норми права. Правові системи країн, об’єднані в один правовий тип (сім’ю), мають подібність структурної побудови нормативно-правового матеріалу. Це виражається в поділі системи права на підсистеми (публічну і приватну) і підрозділи — об’єктивне і суб’єктивне право, загальносоціальне і спеціально-соціальне право, природне і позитивне право та ін. Загальним є поділ норм права за галузями, підгалузями, інститутами та іншими елементами. Однакову будову має норма права.
  4. Спільність принципів регулювання суспільних відносин. Одні правові системи керуються ідеями свободи суб’єктів, їх формальної рівності, справедливості правосуддя і т.д., другі — віддають перевагу релігійним засадам (мусульманські країни), треті — звичаям і традиціям (країни Африки), четверті — ідеям панування закону, що виражає монополію держави в житті суспільства і особи (соціалістичні країни), тощо.
  5. Єдність юридичної техніки, включаючи термінологію, юридичні категорії, поняття, конструкції. Вживання правовими системами однакових або подібних за своїм значенням термінів пояснюється єдністю їх походження або проведенням уніфікації законодавства, обумовленої цією єдністю. Законодавці країн, що належать до одного правового типу (сім’ї), керуються тими самими принципами побудови нормативно-правового матеріалу, застосовують подібні або ідентичні юридичні конструкції, засоби викладу тощо.

Сама по собі класифікація — лише допоміжний засіб, що застосовується для впорядкування різних правових систем світу.

Пропонуємо класифікацію правових систем за такими типами:

  1. Романо-германський тип правової системи (країни континентальної Європи: Італія, Франція, Іспанія, Португалія, Німеччина, Австрія, Швейцарія та ін.). Усередині романо-германського типу правової системи виділяють два підтипи, або дві правові групи: романську (Франція, Бельгія, Люксембург, Голландія, Італія, Португалія, Іспанія. Провідною у цій групі є національна правова система Франції); германську (Німеччина, Австрія, Швейцарія та ін. Провідною у цій групі є національна правова система Німеччини).
  2. Англо-американський тип правової системи (Англія, Північна Ірландія, США, Канада, Австралія, Нова Зеландія, деякою мірою — колишні колонії Британської імперії (нині 36 країн є членами Співдружності) та ін.). Усередині англо-американського типу правової системи розрізняють два підтипи, або дві групи: англійське загальне право (Англія); американське право (США). Крім того, поділ англо-американського типу правової системи можливий на дві групи: європейське загальне право (Англія, Ірландія); позаєвропейське загальне право (США, Канада). Право кожної країни поза Європи набуло широкої автономії в рамках англо-американського типу правової системи.
  3. Змішаний тип правової системи виник на стику двох класичних типів правової системи: романо-германської та англо-американської і має специфічні риси.

У рамках цього типу можна виділити дві групи: північноєвропейські (скандинавські) правові системи (Данія, Швеція, Норвегія, Ісландія, Фінляндія); латиноамериканські правові системи (Аргентина, Парагвай, Уругвай, Чилі та ін.).

  1. Релігійно-традиційний тип правової системи має декілька підтипів: релігійно-общинний (мусульманська, індуська, іудейська, християнська групи); далекосхідний-традиційний (основні групи — — китайська, японська); звичаєво-общинний (африканська група — країни Африки, Мадагаскар).
  1. Поняття, роль та задачі адвокатури.

Адвокатура як соціальний інститут є найважливішим досягненням сучасної цивілізації. Сьогодні всі держави усвідомили необхідність наявності в себе професії адвоката для нормального функціонування своєї правової системи.

Тривалий час точаться дискусії між юристами щодо ви­значення сутності адвокатури, її основних завдань. І спра­ва тут зовсім не в ідеології, як це може здатися на перший погляд. Так, один з найвідоміших процесуалістів дорево­люційного часу Є.В. Васьковський писав, що адвокат — це уповноважений суспільства, який зобов’язаний пильнува­ти суспільний інтерес. І навпаки, уже в сталінські часи цю позицію піддав критиці професор М.М. Полянський, який про сутність роботи адвоката висловився коротко і чітко: «Метою захисту і є захист». Аналогічні дискусії тривали і на Заході. Переважна більшість фахівців підтримує пози­цію, яку влучно й образно висловив Е. Пікар: «Як магнітна стрілка компаса вказує на північ, так і адвокат повинен перш за все захищати». Тож адвокат повинен керуватися насамперед інтересами клієнта. Однак при цьому є багато обмежень, які зобов’язують адвоката дотримуватися пев­ної поведінки. М.Ю. Барщевський при цьому виокремлює ос­новні її складові: чесність, компетентність і добросовісність.

Розуміння сутності адвокатури в наш час набуло зовсім нового забарвлення. Зокрема, якщо в радянські часи, не без впливу комуністичної тоталітарної державної ідеології, в суспільстві поширеним було ставлення до адвокатів як до людей, які захищають злочинців і зрадників Бать­ківщини, то тепер таке «розуміння» сутності адвокатури зміни­лося на інше: адвокат — це ділок, який за шалені гроші об­слуговує багатіїв, кримінальні структури та захищає службовців-злодіїв від справедливої кари. У такій ситуації по­трібно шукати наукове, юридично обґрунтоване визначен­ня, забезпечуючи практичне застосування його положень.

О.Д. Святоцький пише, що за своєю природою адвокатура є громадською, самостійного виду організацією професійних юристів, яка виконує важливу суспільну функцію — захист прав і законних інтересів громадян та організацій.

Серед юридичних засобів захисту прав людини чільне міс­це належить адвокатурі. Про це свідчить, зокрема, те, що на VIII Конгресі ООН, що відбувся у серпні 1990 р. в Нью-Йор­ку, було прийнято спеціальний документ — «Основні поло­ження про роль адвокатів» (далі — «Основні положення»).

Сприяти забезпеченню належного рівня захисту покли­кані приписи «Основних положень», які стосуються за­гальних обов’язків адвокатів. У них, зокрема, зазначено, що такими обов’язками є: консультування клієнта про йо­го права й обов’язки; надання допомоги клієнту законним способом і здійснення певних дій для захисту його інтере­сів; надання клієнту допомоги в судах, трибуналах, ад­міністративних органах. Важливі приписи вміщено в «Основних положеннях» щодо забезпечення доступності юридичної допомоги, яку покликані надавати адвокати, а саме: обов’язок держав­них органів гарантувати реальний і рівний доступ до адво­катури для всіх осіб, які проживають на території держа­ви, зокрема забезпечити фінансування юридичної допомо­ги незаможним людям. Головна соціальна місія, фундаментальне призначення адвокатури — це захист прав людини. Це положення має бути визначальним у вирішенні проблеми досконалого функціонування цього надзвичайно важливого право забезпечувального інституту.

Отже, поняття і роль інституту адвокатури можна сформулювати так: адвокатура — це незалежна само­стійна громадська організація професійних юристів, яка у встановленому законом порядку виконує важливу суспільну функцію — захист прав і законних інтересів громадян і ор­ганізацій, а також надає їм необхідну правову допомогу.

  1. Предмет та система курсу «Порівняльна адвокатура». Особливості методології курсу

Предмет курсу «Порівняльна адвокатура» визначається ко­лом питань та проблем, які повинні вивчатися в його межах. Предметом вивчення дисципліни в широкому розумінні є процес виникнення та сучасний устрій адвокатури — незалежної самостійної громадської організації професійних юристів, яка у встановленому законом порядку виконує важливу суспільну функцію — захист прав та законних інтересів громадян і організацій, а також надає їм необхідну правову допомогу; напрями та завдання діяльності адвока­тури, її взаємодія з іншими правоохоронними, громадсь­кими, державними утвореннями.

Система курсу охоплює питання, присвячені правовим основам функціонування органів адвокатури у різних країнах, правовому статусу адвоката та його помічника, організаційним формам та видам діяльності адвокатів, формам участі адвокатів у кримінальному та цивільному судочинстві, розгляді госпо­дарських спорів, спорів у Європейському суді з прав людини.

Для більш упорядкованого та послідовного вивчення курсу «Порівняльна адвокатура», з метою отримання глибших знань, цей курс, як і інші навчальні юридичні дисципліни, систематизовано. Згідно з цією систематизацією він скла­дається з двох частин: загальної та особливої.

У загальній частині вивчаються питання та проблеми, характерні для будь-якої діяльності будь-якого адвоката або адвокатського об’єднання у будь-якій державі, а також історія розвитку адвокатури: загальні основи функціонування та напрями діяльності, вимоги, які ставляться до адвокатів, їх правовий статус, статус помічника адвоката, дисциплінарна відповідальність і порядок притягнення до неї.

В особливій частині курсу розглядаються питання здій­снення адвокатом у різних державах окремих видів діяльності: надання консультацій; складання документів правового характеру;  участь адвоката в дізнанні та досудовому слідстві, у розгляді кримінальних справ у судах; участь адвоката в розгляді цивільних справ у судах;  участь адвоката у вирішенні господарських спорів; участь адвоката в Європейському Суді з прав людини.

Особливості методології курсу «Порівняльна адвокатура». Методологія порівняльного правознавства є багаторівневою. Порівняльне правознавство використовує практично весь методологічний арсенал сучасної юридичної науки. Тому методологія компаративістики не може бути обмеженою використанням виключно порівняльно-правового методу. Разом із тим останній є провідним, домінуючим в науці порівняльного правознавства. Отже, буде справедливим говорити про те, що порівняльне правознавство засноване на свідомому, теоретично і методично виваженому застосуванні порівняльного методу як основного наукового методу в дослідженні.

Порівняння — невід’ємна частина людського мислення. Порівняльний метод активно використовується в більшості суспільних і природничих наук. У філософській літературі справедливо зазначається, що порівняння є одним з основних логічних прийомів пізнання зовнішнього світу. Проте воно, безумовно, не може розглядатися ізольовано від інших логічних прийомів пізнання (аналізу, синтезу, індукції, дедукції тощо). Окремо один від одного ці компоненти загальної системи пізнавальних засобів існують лише як уявна абстракція.

Порівняння як метод є способом, за допомогою якого встановлюється схожість або розбіжність об’єктів (явищ, речей, процесів), що досліджуються. Такий результат досягається шляхом зіставлення, розрізнення об’єктів за будь-якою ознакою, властивістю. Тому порівняння має сенс лише у випадках, коли об’єкти, що порівнюються, мають якусь схожість (навпаки, не рекомендується порівнювати пуди з аршинами, а їжака із шевською щіткою).

Порівняння широко застосовується в правознавстві при виявленні загальних і специфічних рис таких явищ, як норми права, джерела права, правопорушення тощо. Ця процедура неминуче передує стадії формування правових понять, категорій, законів. Перш ніж пізнати важливе, суттєве, необхідне, те, що становить зміст поняття, закономірності, треба попередньо виявити, що ж є загальним і сталим в об’єкті дослідження. Одержанню цих відомостей і сприяє порівняльний метод.

Порівняльний метод дозволяє констатувати спільність правових систем, окремих правових галузей та інститутів, норм права і виявити те, чим вони розрізняються. У той же час причини повторюваності, сталості цієї спільності у процесі порівняльного дослідження найчастіше залишаються нерозкритими. Отже, порівняльно-правовий метод головним чином сприяє накопиченню емпіричних знань. Теоретичні ж знання здобуваються завдяки іншим науковим методам. Так, статистичний метод аналізує кількісну сторону порівняння, далі застосовуються методи абстрагування тощо.

Для порівняльного правознавства характерним є активне використання загальнофілософських методів: принципів, законів та категорій діалектики, системного підходу тощо. Крім того, при проведенні порівняльних досліджень використовуються інші методи. Це, зокрема, формально-юридичний, історичний, соціологічний методи, метод правового моделювання, математичні і кібернетичні методи.

  1. Адвокатська діяльність у структурі правових систем сучасності, порівняльно-правовий аспект.

Законодавство різних держав дає своє трактування поняття «адвокатської діяльності». Наприклад, у Вірменії адвокатська діяльність визначена як «вид правозахисної діяльності», спрямованої на здійснення не забороненими законом засобами й способами інтересів, переслідуваних одержувачем юридичної допомоги. Законодавство Таджикистану говорить про адвокатури як незалежному професійному об’єднанні, що забезпечує надання юридичної допомоги фізичним і юридичним особам, і дає її визначення як «виду соціальної допомоги», надаваної в правовій області фізичним і юридичним особам, що полягає у використанні будь-яких законних засобів і способів з метою захисту прав і законних інтересів цих осіб. У Киргизстані адвокатська діяльність визначена як «діяльність адвоката по наданню кваліфікованої юридичної допомоги фізичним і юридичним особам при здійсненні захисту їхніх прав, воль і законних інтересів». Законодавство Німеччини визначає адвокатури як незалежний орган правосуддя.

У правових системах різних держав спостерігається дві тенденції по визначенню адвокатської діяльності як некомерційної або як підприємницької. Більшість учених, а з ними й законодавців дотримуються правової аксіоми про те, що адвокатська діяльність — це вид професійної, але ніяк не підприємницької діяльності.

Друга правова концепція деяких держав визначає адвокатську діяльність як комерційну. Наприклад, Закон Киргизстану, визначаючи форми діяльності адвокатів, дозволяє їм здійснювати адвокатську діяльність або через адвокатські установи, або як індивідуальну підприємницьку діяльність. У Законі Таджикистану адвокат-повірник прямо визначений як «підприємець, що робить юридичну допомогу на підставі ліцензій». 

  1. Зародження та особливості інституту адвокатури у Стародавній Греції.

Держави-міста, розташовані у найдавніші часи на Пелопоннеському півострові, про які збереглися історичні хроніки, являли собою політичні союзи первинної формації з твердо встановленим і визначеним панівним центром. Усе внутрішнє правління перебувало в руках спадкової династії царів. Суд вершили або самі царі, або старі досвідчені мужі знатного походження. Процес будувався на принципах усності, гласно­сті та змагальності. Позивачі приходили на суд і особисто захищали свої права.

З розвитком суспільного життя з’явилася адвокатура. Процес її становлення можна простежити на прикладі Афін, порівняно з якими інші грецькі держави-міста були незрівнян­но нижчими в культурному відношенні і не залишили будь-яких пам’яток щодо адвокатури. В Афінах рано створилися умови для виникнення адво­катури: демократичний устрій, розвиток громадянського сус­пільства, процвітання ораторського мистецтва, усність та пуб­лічність розгляду справ судами, панування змагального принципу як у цивільному, так і в кримінальному процесі. За таких умов виникла потреба в судовому захисті для осіб, які не володіли юридичними знаннями та красномовством. Особ­ливе значення надавалося останньому. Щодо судової сфери, то в ній красномовство відігравало особливо важливу роль. Оскільки суддями були рядові грома­дяни, які недостатньо розумілися в юриспруденції, то й не дивно, що головну увагу було звернено на красномовство позивачів. Той, хто погано говорив, як правило, програвав справу. Особи, які були позбавлені природою дару слова, не звільнялися від обов’язку захищати себе на суді, до чого їх примушував закон, що породжувало прагнення обійти його, а це, в свою чергу, призвело до складання для позивачів промов, які заучувалися і промовлялися в суді. Спочатку такі промови (логографії) складалися для рідних і друзів, але потім складан­ня їх стало зайняттям особливого прошарку осіб, які назива­лися логографами чи диктографами. Поряд з логографіями розвивається усний захист, спочат­ку у вигляді родинної адвокатури, а потім вільнодоговірної. Хоч логографи існували і у пізніші часи.

Характеризуючи адвокатуру цих часів, слід визначити деякі її особливості, знайомство з якими є необхідним для наступного дослідження. Насамперед зауважимо, що адвока­тура міст-полісів була більш пов’язана з ораторським мистец­твом, ніж із правознавством. Пояснюється це різними причи­нами. З одного боку, еліни приділяли мало уваги розвитку Юриспруденції. З іншого, будучи народом, схильним до прек­расного., вони натхненно віддавали перевагу різним мистецт­вам, філософії, спорту, військовим справам. Якщо взяти до уваги простоту і загальнодоступність законодавства, усність і гласність судочинства, відкритий розгляд справ перед обшир­ною народною аудиторією, то стане зрозумілим, що професій­ні синегори були лише ораторами — майстрами політичних дискусій. Юридичних знань від них ніхто не вимагав. Для цього існував особливий прошарок юрисконсультів, законни­ків (прагматиків), які супроводжували ораторів на суд і підка­зували їм в разі необхідності потрібні факти і відомості. Згідно з характером адвокатури, підготовка до неї полягала не у вивченні правових дисциплін, а в заняттях ораторським мистецтвом та ін. Звичайно, деякі з ораторів були добре обізнані у законах, на першому ж плані стояло красномовство.

Другою особливістю адвокатури було те, що позивачі та їх захисники не завжди могли висловлюватися в суді стільки часу, скільки вони вважали за потрібне. Як це не дивно, дебати сторін були обмежені певними часовими рамками. Цікаво, що таке обмеження існувало для важливих справ, в той час дрібні і незначні не піддавалися ніяким обмеженням. Час визначався водяними годинниками, які називалися клепсидрами. Тому всі справи поділялися на «справи з водою» та «справи без води». Для кожної справи потрібним був різний рівень води в сосуді, залежно від важливості процесу. Можна лише припусти­ти, що кожній стороні визначався певний об’єм води у годин­нику, не дивлячись на кількість ораторів, що мали виступити. Течія води зупинялася під час допиту свідків і ознайомлення з документами чи текстами законів. Використання клепсидри було пов’язане з прагненням дещо обмежити багатослівність ораторів, проте є очевидним, що цей захід, безсумнівно, заш­коджував інтересам захисту.

Зрештою, слід звернути увагу ще на одну сторону антич­ної адвокатури: різкість, брутальність і, навіть, просто непри­стойність багатьох промов ораторів. У запалі ораторського захвату адвокат не жалів нічого: ні доброго імені свого супер­ника, ні честі його жінки і матері, ні скромності слухачів.

  1. Виникнення та розвиток адвокатури у Римі (період імперії та республіканський період).

Римська адвокатура заслуговує особливої уваги. З неї фактично почався висхідний розвиток світової адвокатури. В Римі, як і в Греції, первісною формою була родинна адвока­тура. Наступним перехідним етапом у її розвитку був, так званий, інститут патронату. Відносини між патронами та клі­єнтами будувалися за аналогією з родинними відносинами. Клієнт, який обрав собі патрона із кровних римських грома­дян (патриціїв), приписувався до їх роду, до родового культу і отримував право називатися родовим іменем. Його ставлення до патрона було не лише родинним, а й вважалося навіть вищим, священнішим, бо родичі не завжди брали участь в родовому культі. Ні він, ні патрон не могли позиватися один з одним, свідчити один проти одного. Клієнт був зобов’язаний ставитися до патрона з повагою, робити йому послуги, випла­чувати за нього і за його дітей викуп у випадках, коли вони потрапляли у полон до ворогів, брати участь своїм майном у покритті боргів чи витрат при відправленні громадської служ­би. Зв’язок з патроном вважався постійним і навіть спадко­вим, і якщо клієнт вмирав бездітним, то його майно перехо­дило до патрона. У свою чергу патрон мав всіляко протегувати клієнту, захищати його інтереси перед судом та ін.

Патронат, як такий, не вніс принципово нового у розви­ток захисту, оскільки був результатом його поширення на осіб, які перебували у стосунках, подібних до родинних. Таке яви­ще, тільки в іншій формі, спостерігалося в Греції, Разом з тим патронат започаткував розвиток вільної адвокатури. Це відбу­валося так. Як відомо, обізнаність в праві та його застосуванні були в Стародавньому Римі прерогативою патриціїв. Лише вони одні допускалися до виконання громадянських обов’яз­ків, участі в державних справах, відправлення правосуддя і захисту своїх клієнтів у суді. У часи відсутності писаного права вони до видання законів XII таблиць, і навіть пізніше, були єдиними, обізнаними в праві людьми. Поки домінував інсти­тут патронату, потреба в юридичному захисті повністю задо­вольнялася адвокатурою родичів і патронів. Але вже при пер­ших царях внаслідок нових соціальних і політичних умов патронат став розхитуватися і розпадатися. Швидке розширен­ня території Риму і збільшення населення за рахунок підко­рення сусідніх племен зробили його цілком недієздатним.

У республіканський період адвокатура залишалася вільною професією. Проте адвокатура в цей період практично так і не отримала законодавчої регламентації. Адвокатура у своїй діяльності керувалася правилами, виробленими практикою і звичаями. Умови зайняття адвокатською діяльністю, як-от: освітній ценз, фахова підготовка, моральні якості тощо — не були визначені. Молоді люди, що виріши­ли присвятити себе цій діяльності, прослуховували курс риторики у викладачів-ораторів, спостерігали, як надаються консультації відомими правознавцями, відвідували засідання судів. Але ні порядок, ні строк, ні навіть обов’язковість цих занять не були встановлені законом.

Питанням, що отримало законодавчу регламентацію, стало питання оплати праці адвокатів. Законом Цінція 204 р. до н. є. адвокату було заборонено брати або обумовлювати гонорар до початку розгляду справи. Гонорар адвокат міг отри­мати лише після закінчення розгляду справи судом у вигляді подарунка.

Адвокатура вважалася почесною, шляхетною діяльністю. Незважаючи на те, що видання законів XII таблиць зробило її більш доступною, все ж аристократич­на тенденція панувала в адвокатурі весь час і в більшості випадків нею займався забезпечений і освічений клас патриціїв.

За часів імперії римська адвокатура, як і судові установи, зазнала певних змін. Перш за все, це позначилося на значному обмеженні адвокатської професії як вільної. Так, за кодексами Юстиніана адвокатура вводилася у рамки певної сис­теми, в якій адвокатська професія прирівнювалася до державної служби. Допуск до адвокатури залежав від вищого адміністративно-судового чиновника провінції або міста. У ній не могли брати участь неповнолітні, особи з фізичними вадами (глухі, німі), позбавлені громадянської честі, притягнуті до кримінальної відповідальності, жінки та ін. Кандидат мав закінчити спеціальний (п’ятирічний) курс в одній з юридичних шкіл та скласти іспити. За часів Юстиніана юридичні школи існували тільки у Константинополі, Римі й Бериті. Усі інші були заборо­нені. Адвокати заносилися до списку за префектурами у порядку їх допуску до професії. Перший у списку звався старшиною. Усі адвокати поділялися на два розряди: штатних та позаштатних. Різниця між ними полягала в тому, що перші, які складали меншість, мали право виступати в усіх судах, а другі, кількісно не обмежені, практикували у нижчих судах. Штатні адвокати призначалися прави­телем провінції з позаштатних.

Дисциплінарний нагляд за адвокатами здійснював правитель провінції. Серйозними професійними порушеннями вважалися: зрада клієнту, вимагання великих гонорарів, наклепи та ін. За ці та інші порушення професійних обов’язків накладалися дисциплінарні стягнення у вигляді штрафу, заборони займатися адвокатською практикою з виключенням із списку. Для адвокатів була встановлена особлива професійна присяга, яку вони виголошували не при вступі до професії, а на початку розгляду кожної судової справи.

На суді адвокат був зобов’язаний утри­муватися від образливих висловів, крім того йому заборонялося свідомо зволіка­ти процес. Щодо гонорару, то до розгляду справи адвокат не мав права наперед обумовлювати винагороду, але після захисту він вже міг ставити таку умову. За наявності домовленості розмір гонорару визначався адвокатом, а у разі відсут­ності, за його позовом гонорар призначав суд, враховуючи складність справи, талановитість адвоката, традиції адвокатури і ранг судової інстанції, але розмір гонорару не міг перевищувати такси, встановленої законом. Незаможним грома­дянам гарантувався захист за призначенням. Зайняття адвокатською діяльністю було заборонено лише суддям та намісникам провінцій.

  1. Характеристика адвокатури періоду раннього та пізнього Середньовіччя.

У середні віки (V—XV століттях) принципи організації адвокатури зазнають певних змін, зокрема, щодо допуску до адвокатури. У Франції для цього необхідно було мати диплом ліценціата прав (юридичну освіту), виголосити присягу та бути внесеним до списків адвокатів. Практичний досвід не був обов’язковим. У середньовічній Німеччині адвокатура являла собою абсолютно вільну професію. Будь-яка особа могла от­римати право на зайняття адвокатською діяльністю на невизначений строк.

Класичний порядок допуску до адвокатури сформувався у цей період в Англії. Тут особа, що виявила бажання присвя­тити себе адвокатській діяльності, мала пройти восьмирічний курс навчання в судовій колегії, і через три роки отримувала звання «внутрішніх адвокатів» (inner barristers), які не мали права виступати в судах. Ще через п’ять років навчання внутрішні адвокати перетворювалися на «зовнішніх» (outer, utter barristers) і отримували право практикувати.

Для внутрішньої організації адвокатури у середньовіччя характерним було таке. У Німеччині, наприклад, такої організа­ції фактично не було. Адвокати були абсолютно самостійними і незалежними від своїх колег за професією, тобто вони не утво­рювали особливого стану. У Франції вже спостерігається зародок станової організації. Тут у XIV ст. в складі релігійного «братства св. Миколи» (патрон юристів) утворилася община адвокатів і повірених, на чолі якої стояли депутати, що обиралися її члена­ми. Вони розпоряджалися майном общини, були її представни­ками у зносинах з урядовими установами та захищали права й привілеї своїх членів. В Англії в XIII ст. виникають перші чотири «судові колегії», в яких й сьогодні об’єднуються у єдиній корпо­рації практикуючі юристи — судді та адвокати.

Гонорарна практика у цей період стає на шлях, накресле­ний юстиніановим законодавством. Так, у Франції починає застосовуватися такса, яка встановлювала максимум винагороди, яку міг визначити і отримувати адвокат, обумовлювати її розмір до початку процесу. Коли ж такої умови не було, або якщо клієнт вимагав зменшення гонорару, то його розмір визначався парламентом. Адвокат міг також звернутись з по­зовом до суду у випадку ухилення клієнта від сплати винаго­роди. В Німеччині існувала такса юридичних послуг, за пору­шення якої адвокат позбавлявся права практикувати, і разом з клієнтом, який переплатив, піддавався штрафу, а іноді навіть тілесному покаранню.

Професійна діяльність адвокатів, в описувані часи, здій­снювалася у таких формах: надання юридичних порад, захист у суді, складання судових паперів.

Значних змін зазнала у середні віки діяльність адвокатів в кримінальному процесі. У цей період публічний і загальний процеси почали перетворюватися у таємний та інквізиційний,, чим обмежувалася участь адвокатів у кримінальному процесі. Наприклад, у Франції в 1539 p. указом короля Франциска І участь адвоката у процесі дозволялася лише за спеціальним дозволом суду.

Загалом для адвокатури середньовіччя характерним було: відсутність чіткої станової організації; відокремлення правоза­ступництва від судового представництва; відносна свобода професії; тісне спілкування з судом; дисциплінарна залежність від суддів; римська система визначення гонорару.

  1. Проведення порівняльного аналізу становлення інституту адвокатури у сучасних європейських країнах: Великобританія, Франція, Німеччина.

Впродовж XVI—XIX століть адвокатура поступово наби­рає іншого вигляду та набуває сучасних форм. У нові часи вона стає самостійною і виробляє станову організацію, яка існує по сьогоднішній день.

Так, у Франції утвердження самостійного стану сталося внас­лідок розпаду «общини адвокатів і повірених», про яку йшло­ся вище. Органом самоврядування адвокатської общини стали комітет або рада.

В Англії організація судових колегій набрала більш стру­нкого вигляду. Так, вони перетворилися на самоврядні общини з виборними старшинами, членами яких були як адвокати, так і кандидати. Вони присвоювали останнім звання адвокати або позбавляли його, здійснювали нагляд за внутрішньою дисципліною і, загалом, відали усіма справами общини.

У Німеччині за статутом 1876 р. адвокати об’єднувалися у адвокатську камеру (або адвокатську колегію) на чолі з виборною радою.

У цей період твердо утвердилася додаткова умова щодо допуску до адвокатської професії — наявність практичного стажу юридичної роботи або стажування. Наприклад, у Фран­ції згідно з указом 1822 р. вимагалася наявність 3-річного практичного стажу або 5-річне стажування. У Німеччині статут 1878 p. встановив 3-річну практичну підготовку до самостійної адвокатської діяльності.

Зазнала докорінної зміни і гонорарна практика. Тепер винагорода за захист в суді або надання юридичної поради перестала бути платнею за особисту послугу, перетворившись на почесний дарунок, який не можна було ні обумовлювати, ні вимагати по суду (зокрема, у Франції та Англії). Але й клієнт, сплативши гонорар, не міг вимагати його повернення. У Німеччині за законом 1879 p. визначення гонорару чинилося як за домовленістю, так і за таксою.

Отже, минаючи масу проміжних і змішаних форм, що існували в середні століття в Європі, до початку новітнього часу сформувалося два основних типи побудови адвокатури: англо-французький і німецький. Розглянемо коротко основні риси кожної з них.

  1. Англо-французька система організації адвокатури будувалася на наступних принципах: І. Відносна (на відміну від абсолютної) воля професії адвоката. ІІ. Станова організація адвокатури. ІІІ. Відділення адвокатури від судового представництва. ІV. Тісний зв’язок адвокатури з магістратурою. V. Відносна безплатність праці адвоката.

Розкриємо суть кожного принципу. 1-й принцип означає, що, незважаючи на те, що доступ в адвокатуру був відкритий для кожного, проте  існував визначений освітній і моральний ценз для бажаючих стати адвокатом. 2-й принцип виражений у тому, що адвокатура являла собою самокеровану організацію, і тільки член цієї організації (колегії адвокатів) вважався адвокатом. 3-й принцип представлений як досить суворий організаційний і законодавчий поділ осіб, що займаються адвокатською діяльністю і судовим представництвом. 4-й принцип означає, що адвокатура і судова влада знаходяться на одному соціальному і правовому рівні. Адвокати мали переважне право на заняття суддівських посад. 5-й принцип виражений у тому, що оплата роботи адвоката вважалася даром вдячного клієнта за участь у його справі. Заборонені були всі угоди з приводу гонорару. Гонорар виплачувався тільки після закінчення справи, причому особисто клієнтом.

  1. Німецька система. По ній організація адвокатури ґрунтувалася вже на трохи інших принципах, це: І. Відносна воля адвокатської професії. ІІ. Станова організація адвокатури. ІІІ. Сполучення адвокатської діяльності з правовим представництвом. ІV. Відсутність якого-небудь зв’язку адвокатури з магістратурою. V. Еквівалентна оплатність праці адвоката.

1-й і 2-й принципи вже описані в англо-французькій системі. Вони збігаються з німецькою. Але 3-й принцип означає відсутність організаційного розподілу між адвокатами і правовими представниками. Адвокати поєднують у собі сутність і діяльність обох цих суб’єктів. 4-й принцип полягав у тому, що адвокати не мали ніякого відношення до судової влади. Вони вважалися набагато нижче неї, і рідко хто з адвокатів міг стати суддею. 5-й принцип виражався в тому, що гонорар за надання адвокатом послуги був заробленою винагородою, а не даром вдячного клієнта, він визначався заздалегідь за згодою сторін або твердій таксі.

Коротко розглянувши обидва види зародження адвокатських систем Західної Європи, можна назвати основні причини розходження англофранцуської і німецької адвокатур. Очевидно, що ці причини знаходяться тільки усередині організації самої адвокатури. Нагадаємо, що два перших принципи побудови обох систем однакові. Тому причина розходжень систем знаходилася в інших трьох, а саме в наступному: там, де адвокатська діяльність з’єднана з представництвом, а адвокатура не може йти в порівняння із судом, причому адвокати за свою працю одержували винагороду в такому ж порядку, як і сфера обслуговування, там адвокатура швидко занепала, опустилася до рівня звичайного ремесла. Так вийшло з адвокатурою німецького типу. Англо-французька ж система адвокатури, навпаки, розцвіла.

Історія адвокатури кінця XІХ — початку XX століття, на жаль, розвивалася по типі саме німецької системи. Існуючий в багатьох країнах поділ власне адвокатури і правового представництва був згодом  ліквідований, причому остаточно, адміністративними мірами. Наприклад, у Франції це відбулося досить пізно, із прийняттям у 1971 р. закону «Про реформу деяких судових і юридичних професій». Їм уперше була введена «єдина професія адвоката».

Така тенденція двадцятого сторіччя мала свої об’єктивні причини. Одна з них складалася в розширенні функції правового представництва. Судове представництво стало помітним свідченням майже повної регламентації громадського життя правом.

На цій базі сформувався новий напрямок юридичної діяльності — консалтинг, тобто надання юридичних консультацій у сфері бізнесу. Отже, був сформований новий тип юриста, що обслуговує ділових людей.

Інша тенденція сучасної західної адвокатури — це її інтернаціоналізація. Адвокати можуть тепер займатися своєю діяльністю не тільки на «своїй» території, але і практично у всіх країнах. Це насамперед стосується в першу чергу сфери обслуговування бізнесу, оскільки він став у принципі інтернаціональний. Інтернаціоналізація ж судової системи змінилася набагато в меншій мері.

  1. Загальна характеристика етапів розвитку адвокатури у Росії.

Історія розвитку адвокатури Росії налічувала чотири етапи.

  1. У Росії, на відміну від країн Європи, де правозаступництво і судове представництво розвивалися як два самостійних інститути, перше виник не як самостійний інститут, а в зв’язку із судовим представництвом. Аж до XV ст. у Росії існував так називаний «принцип особистої явки». За псковською судною грамотою запрошувати повірників могли тільки жінки, діти, ченці, старезні старі і глухі. Потім у суспільстві як захист з’являється родинне представництво. Безпосередньо за ними зародилися і наймані повірники. Функції їх могли здійснювати всі дієздатні особи. Повірників називали заступниками по справах, стряпчими. Тільки в XІХ ст. зі звичайного правового інституту судове представництво перетворюється в юридичний інститут присяжних стряпчих, що вносилися в особливі списки, що існували при судах.
  2. Російська присяжна і приватна адвокатура народилася в умовах демократичних судових реформ 60-х рр. XІХ століття в царювання імператора Олександра ІІ. У відомій мері це було змушене рішення: уведення суду присяжних і змагального судового процесу не могло відбутися без професійної адвокатури.

До кандидатів у присяжні повірники пред’являлося вимогу наявності закінченої вищої освіти. Склад присяжних повірників контролювався Судовою палатою округу, при якій створювався Рада присяжних повірників, діяльність якого також була піднаглядна Судовій палаті. У присяжні повірники не допускалися іноземці, неспроможні боржники, особи, що знаходяться на урядовій службі, засуджені по вироку суду до позбавлення або обмеження прав існування, що знаходяться під слідством, особи, що виключені з числа присяжних повірників і ін.

Рада присяжних повірників розглядала питання прийому і відрахування, скарг на дії присяжних, контролювала дотримання ними законів і правил професії, включаючи розміри стягнутих винагород, установлювала черговість «ходіння по справах осіб, що користуються на суді правом бідності». Скарги на рішення Ради присяжних подавалися в Судову палату.

Таким чином, незалежність створюваної в результаті реформи адвокатури була досить відносною, а самоврядування — у межах, контрольованих судом.

Формувалися визначені моральні орієнтири професії адвоката — у вимогах до кандидатів у присяжні, а також при визначенні їхніх правових обов’язків і обмежень.

Так, присяжним повірником заборонялося здобувати права своїх довірителів за позовами, вести справи проти своїх родичів (батьків, дружини, дітей, братів і сестер), представляти відразу обидві сторони або переходити з однієї сторони на іншу, оголошувати таємниці свого довірителя.

Існувала і такса оплати праці адвоката, затверджена Міністром юстиції для ведення цивільних справ у 1868 р. (відлік гонорару йшов від ціни позову).

З присяжного повірника довіритель міг стягнути заподіяні недбальством адвоката збитки. Було можливе і кримінальне переслідування присяжного повірника за серйозне зловживання.

  1. Жовтнева революція 1917 р. зруйнувала, і судові установи, і адвокатуру. Через кілька тижнів після жовтневого перевороту, 24 листопаду 1917 р. був прийнятий Декрет про суд № 1. Він був орієнтований на створення «радянських судів». Адвокатура ним створена не була, однак як захисники і обвинувачі були допущені «усі непорочні особи обох статей».

Інструкцією Наркомюста 19.12.1917 р. при трибуналах засновувалися «колегії правозаступників», у які могли вступати будь-як особи, що бажають «допомогти революційному правосуддю» і відповідні рекомендації, що мають, від Рад депутатів.

Декрет про суд № 2 від 7 березня 1918р. передбачив створення колегій правозаступників при Радах депутатів. Пізніше (Декретом «Про народний суд РСФСР» від 30.11.1918 р.) колегії правозаступників стали іменуватися колегіями «захисників, обвинувачів і представників сторін у цивільному процесі». Їх складали посадових особи, що одержували зарплату від держави за кошторисом Наркомюста.

Незабаром ці колегії були скасовані під тим приводом, що в них були «сильні елементи буржуазної адвокатури».

Захист став здійснюватися в порядку трудової повинності особами, включеними в списки місцевими виконкомами.

Ці міри були спрямовані на остаточне знищення дореволюційної адвокатури Росії.

Днем народження радянської професійної адвокатури прийнято вважати 26 травня 1922 р., коли рішенням ІІІ сесії ВЦИК ІX скликання було затверджено перше Положення про адвокатуру. Колегії захисників створювалися при губернських відділах юстиції. Загальними зборами захисників обиралася Президія, що відала прийомом і відрахуванням адвокатів, накладенням дисциплінарних стягнень, рішенням фінансових і адміністративних питань.

З цього часу почався ріст чисельності колегій адвокатів, створення мережі юридичних консультацій. Приватнопрактикуючи адвокати були оголошені поза законом.

Положеннями про адвокатуру від 27 лютого 1932р., 16 серпня 1939 р. розвивалися принципи організації адвокатури, установлені першим положенням, більш чітко визначалися права й обов’язки структурних підрозділів колегії адвокатів і їхніх членів.

Положення від 16 серпня 1939 р. уперше закріпило принцип, відповідно до якого в адвокатуру могли вступати лише особи, що мають юридичну освіту, або не менш трьох років досвіду роботи суддею, прокурором, слідчим або юрисконсультом. Підвищувалася самостійність колегій шляхом розширення повноважень загальних зборів адвокатів і обираємої ним таємним голосуванням Президії. Оплата праці адвокатів регулювалася таксою, затверджуваної Наркомюстом.

Роль адвокатури в правоохоронній системі як органа надання населенню юридичної допомоги і забезпечення захисту по кримінальним справам зростала в міру твердження гасла про зміцнення соціалістичної законності.

Перший Закон СРСР «Про адвокатуру в СРСР» був прийнятий Верховною Радою СРСР 30 листопада 1979 р. Цим законом вносилася однаковість у принципи організації і діяльності адвокатури всіх радянських республік.

Уперше було встановлене правило, що діє і нині, про порядок створення добровільних об’єднань осіб, що займаються адвокатською діяльністю. Остаточно затверджувався принцип обов’язкової вищої юридичної освіти для членів колегій адвокатів, установлювалися терміни стажування для претендентів, що не мають практичного досвіду, розширювалося поле діяльності адвокатів.

На базі союзного закону про адвокатуру було прийняте Положення про адвокатуру РСФСР від 20 листопаду 1980 р. Воно передбачало поряд із традиційними регіональними колегіями (область, край, автономна республіка) створення «міжрегіональних і інших колегій адвокатів», що дозволило при переході до ринкових відносин формувати спочатку юридичні кооперативи, а потім і так звані рівнобіжні колегії. Положення 1980 р. поряд з деяким розвитком демократичних принципів організації і діяльності колегій адвокатів зберіг значні права Мін’юсту і виконкомів обласних (крайових і т.д.) Рад по «керівництву адвокатурою».

Розширення прав колегій адвокатів, розвиток їхнього самоврядування і незалежності — повільний процес, що відбувається нині в руслі судово-правових реформ.

  1. Судово-правова реформа 1990-х рр., що викликає не тільки позитивні оцінки, але й обґрунтовану критику, істотно змінила правовий статус адвоката й адвокатських об’єднань. У 2002 р. набрав сили РФ Закон про адвокатуру й адвокатську діяльність, що у корені змінив правову природу адвокатури — ця організація стала цілком незалежної від держави, розширилися права адвокатів, цілком змінився структурно-організаційний устрій адвокатських утворень. Адвокат одержав дуже широкі процесуальні права, став більш значимою фігурою в кримінальному, цивільному, арбітражному процесах, придбав додаткову вагу в господарському житті юридичних осіб, у захисті прав і законних інтересів російських громадян, а статус самої адвокатури закріплений у новому законі як інститут громадянського суспільства.

Свої задачі перед суспільством російська адвокатура реалізує шляхом виконання принципів своєї діяльності, визначених у ч. 2 ст. 3 Закону про адвокатуру. До цих принципів відносяться: принцип законності; принцип незалежності; принцип самоврядування; принцип корпоративності; принцип рівноправності адвокатів.

  1. Порівняльно-правовий аналіз розвитку інституту адвокатури в Росії та країнах Європи.

У Росії, на відміну від країн Європи, де правозаступництво і судове представництво розвивалися як два самостійних інститути, перше виник не як самостійний інститут, а в зв’язку із судовим представництвом. Аж до XV ст. у Росії існував так називаний «принцип особистої явки». За псковською судною грамотою запрошувати повірників могли тільки жінки, діти, ченці, старезні старі і глухі.  Потім у суспільстві як захист з’являється родинне представництво. Безпосередньо за ними зародилися і наймані повірники. Функції їх могли здійснювати всі дієздатні особи. Повірників називали заступниками по справах, стряпчими. Тільки в XІХ ст. зі звичайного правового інституту судове представництво перетворюється в юридичний інститут присяжних стряпчих, що вносилися в особливі списки, що існували при судах.

Російська присяжна і приватна адвокатура народилася в умовах демократичних судових реформ 60-х рр. XІХ століття в царювання імператора Олександра ІІ . У відомій мері це було змушене рішення: уведення суду присяжних і змагального судового процесу не могло відбутися без професійної адвокатури. Законодавство, що регламентує діяльність присяжних повірників дореволюційної Росії, не приймаючи в повному обсязі, як австро-німецьку, так і англо-французьку школи права, мало свої позитивні еволюційні тенденції. Воно з’явилося прикладом творчої адаптації всіх кращих досягнень юриспруденції й практики західноєвропейських країн.

Жовтнева революція 1917 р. зруйнувала, і судові установи, і адвокатуру.

Днем народження радянської професійної адвокатури прийнято вважати 26 травня 1922 р., коли рішенням ІІІ сесії ВЦИК ІX скликання було затверджено перше Положення про адвокатуру. Роль адвокатури в правоохоронній системі як органа надання населенню юридичної допомоги і забезпечення захисту по кримінальним справам зростала в міру твердження гасла про зміцнення соціалістичної законності. Перший Закон СРСР «Про адвокатуру в СРСР» був прийнятий Верховною Радою СРСР 30 листопада 1979 р. Цим законом вносилася однаковість у принципи організації і діяльності адвокатури всіх радянських республік.

У  результаті  радянських таких  перетворень  адвокати  стали  більше  походити  на виробничі одиниці, що діяли по вказівці завідувача  юридичною консультацією (до того ж що одноосібно управляли нею), ніж на юристів. Вільно організовані  колективи  скасовувалися,  і  адвокати  змушені  були  переходити  в юридичні  консультації,  які  підкорялися  президії  колегії  адвокатів. В даному випадку, адвокатура стала жорстким еталоном німецької системи адвокатури.

Судово-правова реформа 1990-х рр., що викликає не тільки позитивні оцінки, але й обґрунтовану критику, істотно змінила правовий статус адвоката й адвокатських об’єднань. У 2002 р. набрав сили  РФ Закон про адвокатуру й адвокатську діяльність, що у корені змінив правову природу адвокатури — ця організація стала цілком незалежної від держави, розширилися права адвокатів, цілком змінився структурно-організаційний устрій адвокатських утворень. З цього часу сучасна адвокатура Росії наближається до світових стандартів адвокатури, які зводяться до затвердження німецької моделі адвокатури. Така тенденція двадцятого сторіччя мала свої об’єктивні причини. Одна з них складалася в розширенні функції правового представництва. Судове представництво стало помітним свідченням майже повної регламентації громадського життя правом.

  1. Становлення інституту адвокатури у США.

Професія юриста споконвічно зайняла в США унікальне по своїй значимості положення. Це було зв’язано не тільки зі специфікою соціально-економічних умов країни, але й з особливостями англійського прецедентного права, сприйнятого й удосконаленого американськими першопоселеннями.

У колоніальний період на Американському континенті не було можливостей і необхідності для існування складно диференційованої судової системи. На відміну від Англії, де в XVІІ в. сер Э. Кок нараховував близько 100 різновидів судів, в американських колоніях судочинство не було спочатку відділене від адміністративного керування. «Ті самі   люди створювали закони, перетворювали їх у життя, вирішували судові справи й управляли колонією. Спеціальна судова система виростала й підрозділялася на складові частини тільки тоді, коли для цього стало досить населення, проблем і території, що б це мало сенс» .

У міру розвитку колоній зміцнилася судова ланка в графствах. Ці органи з 8 — 30 чоловік, відбиваючи інтереси місцевих «кращих сімейств», крім судової діяльності, займалися торгівлею, надавали допомогу бідним, піклувалися про суспільні вдачі й попечительствовали релігійним службам.

Як відомо, в Англії вплив адвокатів і суддівських чиновників було досить велике. Однак по іншу сторону Атлантичного океану місцева еліта аж ніяк не палко бажала з ким-небудь ділити влада.

Положення міняється в XVІІІ столітті з поліпшенням умов життя колоністів, зі зміною їхньої економіки й поглядів. У колоніях відчувається потреба в більше розвиненому праві. На загальне право починають дивитися по-іншому: по-перше, тому, що воно може бути використане для захисту від королівського абсолютизму, і, по-друге, тому, що в ньому бачать сполучну ланку між усім, що є англійського в Америці, проти погроз, що йдуть від французьких колоній — Луїзіани й Канади. Разом з тим масштаби застосування загального права як і раніше залишалися об’єктом сумнівів і спорів, юристів як і раніше не вистачало, і суддя з юридичною освітою був рідкістю. Виникає плин на користь більше широкого застосування загального права; американські суди виражають свій намір застосовувати різні англійські закони (наприклад, 1677 р. про шахрайство); у Філадельфії в 1771-1772 роках були видані коментарі Блекстона.

Незалежність, проголошена в 1776-м і остаточно встановлена в 1783 році, створила для колишніх англійських колоній, що стали Сполученими Штатами Америки, зовсім нові умови. Французька погроза, що трохи зменшилася в результаті анексії Канади Англією в 1763 році, повністю зникла із придбанням Сполученими Штатами Америки Луізиани в 1803 році. Франція стала для США іншому й союзником, а всі ворожі настрої були звернені проти Англії. Після завоювання політичної незалежності виникла й стала популярної ідея самостійного американського права. Республіканські ідеали й ідеї національного права обумовили сприятливе відношення до кодифікації. Здавалося нормальним, що Декларація прав і Конституція США (проголошена 17 вересня 1787 р.) будуть доповнені кодексами.

До середини XІХ століття можна було сумніватися щодо результату боротьби, що велася в Америці між прихильниками загального права й прихильниками кодифікації. У Массачусетсі в 1836 році законодавча комісія зажадала складання кодексу; Конституція штату Нью-Йорк 1846 року передбачала складання систематизованого кодексу, що включає все право штату. Ще в 1856 році історик англійського права Генрі Мен пророкував успіх романо-германської правової системи в США.

Однак Сполученим Штатам Америки призначено було залишитися в системі загального права, за винятком території Нового Орлеана, що стало в 1812 році штатом Луізиана. Інші території, що приєдналися до Союзу, могли теоретично підкорятися французьким, іспанським або мексиканським законам. Однак фактично ці закони там були невідомі, і тому в Техасі (з 1840 р.) і в Каліфорнії (з 1850 р.), у принципі, було адаптовано загальне право Англії, а колишні традиції були збережені тільки для декількох інститутів (шлюбний режим, земельний режим). Усюди затвердилися концепції, які раніше застосовувалися в колишніх англійських колоніях, а вони були тісно пов’язані із загальним правом.

Загальне право беззастережно перемогло в США. Закони більшості штатів прямо обмовили, що загальне право за станом на певну дату є діючим правом. В інших штатах обійшлися без такого уточнення. З цього періоду чітко виокремилися й спеціалісти, які надавали послуги близькі до адвокатських.

Отже, розвиток адковатської справи у США тісно пов’язаний с формуванням правової системи Америки.

У США за законом 1871 р. адвокати Нью-Йорку об’єдна­лися в самоврядну колегію, очолювану виконавчою комісією і головою, а через сім років адвокати усієї країни у Федерацію адвокатських асоціацій штатів — Американську асоціацію адво­катів (American Bar Association), засновану у 1878 p., мета якої полягає в обміні досвідом, обговоренні проблем юридичної освіти, реформуванні законодавства та ін. Хоча зазначена асоціація не є такою для усіх американських адвокатів, вона об’єднує у тій або іншій формі майже усі великі адвокатські організації та нараховує у своєму складі понад 130 тис. членів.

Отже, адвокатська професія в США формувалася впродовж більш ніж 200-літньої історії країни. Американське правництво перейняло британську традицію провідної ролі суду та ідею судді як уособлення правової мудрості та кар’єрну вершину юриста.

  1. Характеристика законодавства про адвокатуру різних країн сучасних Європейського союзу.

Англія. В Англії відсутній будь-який єдиний  закон або нормативний акт про адвокатуру та принципи її побудови й діяльності. Як й інші установи та інститути англійського права, адвокатура у її нинішньому вигляді формувалась протягом століть і зараз функціонує на досить розпливчатому ґрунті королівських хартій, окремих парламентських актів, урядових постанов, правил, прийнятих керівними органами адвокатських організацій, судових рі­шень, традицій та звичаїв. До них, зокрема, можна віднести королівські хартії (1845, 1872, 1903, 1909 та 1954 pp.), на основі яких сформувалася одна з найбільших та впливових організа­цій адвокатів — «Юридичне товариство», а також серію пар­ламентських актів, спеціально присвячених адвокатам — солісіторам. Одночасно приймалися аналогічні акти по Шотландії та Ірландії, а після 1921 p. — по Ольстеру, акти в галузі кримінального права та правосуддя, в кримінальних справах, акти «Про захист малозабезпечених ув’язнених» 1930 p., «Про юридичну допомогу та консультації» 1949 p., «Закон про правову допомогу» 1974 р. та численні постанови, видані на його основі, акти лорда-канцлера (він керує галуззю правосуд­дя), численні правила адвокатських організацій про умови членства в них, про порядок розрахунків з клієнтами, про дисциплінарне провадження, правила вищих судових інстан­цій про порядок допуску представників сторін до участі в процесі (1964 р.). На звичаях та традиціях майже цілком засновано органі­зацію та діяльність адвокатів-баристерів та їх керівного органу — Генеральної ради адвокатури, ряд важливих професійних прав та обов’язків адвокатів (наприклад, відмовитися від ви­конання доручення клієнта, про надання йому консультації з правових питань). Положення, що стосуються адвокатури, містяться також у багатьох нормативних актах, виданих з інших приводів. Наприклад, за актом про розлучення 1969 р. суд має визначити суму витрат та розміри винагороди для тих адвокатів, які брали участь у справі, якщо самі сторони не домовились про це, та інше.

Німеччина. Основним нормативним актом, який закріплює правовий статус адвокатури в Німеччині, є Федеральне положення про адвокатуру 1959 p., що діє з незначними змінами і сьогодні. За цим актом адвокат є незалежною особою у сфері правосуддя. Будучи людиною вільної професії, адвокат є ком­петентним консультантом і представником інтересів громадян. Його право виступати з юридичних питань в загальних судах, арбітражних або адміністративних органах обмежується лише федеральним законом.

Франція. Сучасна адвокатура Франції пережи­ває період свого реформування при збереженні багатих традицій минулого. Першого січня 1992 p. набрали чинності поправки до Ос­новного закону про статус адвокатів № 71-1130 від 31 грудня 1990 p., мета яких реформувати деякі судові та інші юридичні професії. Головна особливість реформи — створити нову професію, члени якої носитимуть звання адвоката (ст. 1 закону № 71-1130 у новій редакції). Проте це не означає будь-яких радика­льних перетворень. Усі основні традиції французької адвока­тури зберігаються. Справа в тому, що коли раніше адвокати, об’єднані у «ордени адвокатів» (далі ми використовуватимемо звичний, хоч і умовний термін, — «колегії адвокатів»), працю­вали окремо від своїх колег — юрисконсультів, які реєструва­лись окремо, то з початком реформи останні за бажанням могли стати членами адвокатських колегій, одержавши право на здійснення саме адвокатської діяльності, а адвокати, збері­гаючи свій статус, можуть виконувати функції, які раніше були властиві тільки професії юрисконсульта

Чехія. В Чеській Республіці існує розпо­діл юридичних професій залежно від практики на адвокатів, комерційних юристів, нотаріусів, суд­дів та інших (наприклад, сімейних). Реклама юридичних послуг не дозволяється. Чеська Палата адвокатів визначає основні завдання адво­катської професії відповідно до Акта діяльності адвокатів, схваленого 21 квітня 1990 p.

  1. Правове положення адвокатів у правових системах світу.

У визначенні правового положення адвокатів у правових системах світу окремої уваги вимагає питання про коло осіб, допущених різними державами до надання юридичної допомоги на своїй території.

У деяких державах надання будь-якого виду юридичної допомоги є винятковою прерогативою адвокатів. Наприклад, в Ізраїльському законодавстві існує таке поняття як «ексклюзивність або винятковість адвокатської професії». Це означає, що наданням будь-якої юридичної допомоги населенню можуть займатися тільки особи, що мають спеціальну ліцензію — ришайон, тобто  члени Ізраїльської колегії адвокатів. Ця заборона ставиться як до представництва в судах, так і іншої юридичної допомоги, наприклад, юридичне консультування.

В інших державах фактична монополія адвокатів закріплена лише стосовно   судового представництва. Так, Федеральний закон Швейцарії «Про судоустрій» відносно  представництва у Федеральному суді встановлює наступне: «Діяти як  представник у цивільних і кримінальних справах можуть тільки патентовані адвокати й викладачі права швейцарських університетів». Так само, згідно Ордонансу В’єтнаму, приватні консультації не мають права брати участь у судових розглядах.

Необхідно відзначити, що зазначені положення не знайшли свого закріплення в законодавстві більшості країн, зокрема, держав — колишніх республік СРСР. У більшості законодавств закордонних держав виключне право надавати юридичну допомогу належить адвокатам лише в кримінальному судочинстві.

Законодавство ряду держав юридично допускає участь не адвокатів як  захисників. Ними можуть бути, у різних правових системах, викладачі правий, у тому числі технічних навчальних закладів, просто юристи або інші особи в індивідуальному порядку допущені судом до захисту. На практиці ж ці норми або не застосовуються або тлумачаться правоприменителем обмежено. Наприклад, в одному з рішень Вищого земельного суду Карлсруэ (Німеччина) відзначається, що особа, що не є адвокатом (якщо це навіть юрист), не має необхідну особливу здатність здійснювати захист по кримінальних справах.

Із приводу надання юридичної допомоги в господарських судах і судах по цивільних справах, більшість держав допускає юристов-неадвокатов до судового представництва. Однак, у правозастосовній практиці більшості держав існує однозначне розуміння того, що тільки адвокати можуть робити будь-яким особам будь-яку юридичну допомогу, у тому числі в рамках судового засідання, а юрисконсульти роблять її тільки в рамках тієї юридичної особи, де вони здійснюють свої професійні функції як  найманий працівник.

Іноді в повне ведення адвокатів віддаються особливо тонкі й складні механізми правового регулювання. Так, у США коло осіб, уповноважених подавати позови через Інтернет, так називана система електронного правосуддя, обмежений винятково адвокатами. Це допомагає підвищити ефективність роботи цієї складної системи й у той же час виключити несумлінність інших юристів, діяльність яких не контролюється професійною етикою.

Необхідно відзначити, що деякі держави обмежують сферу адвокатських послуг. Так, відповідно до законів Нідерландів, адвокати, що є членами Нідерландської Асоціації Адвокатів, не вправі надавати юридичні послуги в сферах бізнесу й керування.

  1. Особливості допуску до адвокатської діяльності у США та Великобританії.

В США існує загальне поняття юридичної професії. Амери­канець, що отримав дозвіл займатися юридичними професія­ми, іменується юристом в широкому розумінні цього слова, яких сьогодні нараховується понад 350 тис. Юрист, маючи ліцензію, може бути адвокатом, суддею, прокурором, юрискон­сультом і т. п. Для зайняття адвокатською діяльністю необхідно пройти нелегкий тривалий шлях. Особа після здобуття середньої осві­ти відвідує університетський коледж, в якому вона перебуває три або чотири роки. Потім три роки правничої школи, яких в США існує близько 200, де американський студент одержує знання, зорієнтовані виключно на практику. Наступним ета­пом є насичений юридичними дисциплінами курс і екзамен, який проводиться під контролем суду і є серйозним випробу­ванням. Лише незначне число студентів витримує його. Нео­бхідно зауважити, що сьогодні у половині штатів наявність університетського диплома для зайняття адвокатською діяль­ністю не є обов’язковою. Претенденти мають бути американ­ськими громадянами та мати ценз осілості — проживати не менше шести місяців у даному штаті. Необхідні для роботи юристом вимоги у кожному штаті є різними. Той, хто визнаний юристом в даному штаті, може працювати у цій якості лише у цьому штаті. Кожний юрист, що одержав дозвіл практикувати в даному штаті, заплативши певну суму доларів, може бути внесений до списку адвокатів, яким дозволено виступати у Верховному суді США.

Великобританія. В Англії порядок прийняття до адвокатури є різним для соліситорів і баристерів. Так, кандидати у соліситори, якщо вони не закінчили університет або юридичний коледж, прохо­дять в юридичному товаристві (професійне об’єднання соліси­торів) п’ятирічне стажування, по закінченні якого складають Відповідні екзамени. Строк стажування для осіб, що мають вищу юридичну освіту — три роки. Крім того, кандидат у соліситори повинен оплатити свою підготовку. У «судові Інни» (об’єднання баристерів) приймаються, як правило, особи, що закінчили Оксфордський або Кембридж­ський університети. До отримання практики баристер має здобути учнівський стаж строком не менше 12 місяців під керівництвом старшого члена корпорації. Умовами прийняття учня у її члени є: вік не молодше 21 року; сплата вступного внеску; заняття протягом восьми семестрів та складання екза­мену за фахом. Протягом року (у зимову, весняну, літню і осінню сесії) влаштовуються обіди в трапезній судового Інна, де за окремим столом сидять судді без мантій та париків, а за другим — студенти і учні. Ця своєрідна форма спілкування дає можливість майбутнім адвокатам придивитися, увійти в роль обраної ними професії, і взагалі просто «поваритися» у казані» «судового Інна». По закінченню стажування кандидат у бари­стери складає екзамен та обирає судовий Їнн, в якому йому доведеться працювати.

  1. Права та обов’язки адвоката за законодавством Німеччини та Франції.

Франція. Адвокат має право на мантію своєї професії. Він наділений свободою висловлення, імунітетом від безпідс­тавних обвинувачень у вчиненні злочину, дифамації. Адвокат має право на гонорари, розмір яких залежить від взаємної згоди з клієнтом.

Адвокат зобов’язаний додержувати професійної таємни­ці, що є одночасно правом і обов’язком. Він не обмежений у листуванні з клієнтом, навіть, з тим, що перебуває під вартою.

Адвокату забороняється розголошувати загальну профе­сійну таємницю, таємницю слідства, відомості, одержані від клієнта. Він не може бути радником, представником чи захи­сником більше як одного клієнта в одній справі. Адвокат зобов’язаний додержуватися гідного способу життя. При непо­рядному ставленні до своїх клієнтів він підлягає цивільно-пра­вовій відповідальності, а при вчиненні злочину — криміналь­ній відповідальності. Адвокат може бути притягнутий і до дисциплінарної від­повідальності. Це може статися в разі порушення законів та регламентів, професійних норм, нехтування вимогами чесно­сті, гідності і професійної делікатності, навіть за межами професійної діяльності.

Німеччина. Федеральний закон про адвокатуру регулює в окремому розділі права і обов’язки адвоката. Так, згідно з

  • 43 цього закону, «адвокат зобов’язаний сумлінно виконува­ти свої професійні обов’язки, бути гідним поваги і довіри, яких вимагає становище адвоката».

Адвокат, до якого звернулися по юридичну допомогу, а він відмовився її надати, має негайно пояснити причини. Він зобов’язаний відшкодувати шкоду, завдану клієнту немотиво­ваною відмовою. Адвокат відстороняється від виконання професійних обо­в’язків: якщо його діяльність суперечить принципам високої моралі та права; коли він консультував або представляв у конкретній справі іншу сторону; якщо він брав участь у тій же, вже розглянутій справі, як суддя, арбітр, прокурор або предс­тавник офіційної установи; коли йдеться про тлумачення до­кумента, котрий він склав як нотаріус.

Адвокат не може виступати у загальних або третейських судах як адвокат клієнта, якого він обслуговує у зв’язку з постійними службовими та іншими діловими відносинами.

Особи, які працюють суддями або державними службов­цями, не мають права виконувати обов’язки адвоката. Проте управління юстиції землі може зробити виняток для конкрет­ної особи, якщо це не зашкоджує інтересам правосуддя.

Адвокат зобов’язаний взяти на себе представництво сто­рони в цивільному процесі, якщо він призначений на підставі норм цивільно-процесуального кодексу і закону про суди в трудових спорах або на підставі інших законоположень про тимчасово неоплачуваний захист. Такий же обов’язковий ха­рактер має захист в кримінальних справах, якщо адвокат призначений захисником на підставі норм кримінально-про­цесуального кодексу, закону про порушення громадського порядку або закону про надання міжнародної правової допо­моги.

Адвокат може надавати обвинуваченому свої послуги в будь-якій стадії процесу як на попередньому слідстві і дізнан­ні, так і під час судового розгляду справи. До обов’язків адвоката віднесено й зберігання матеріалів справи протягом п’яти років після виконання доручення.

Одночасно закон надає адвокатові право відмовити кліє­нту у видачі копій документів справи до задоволення претензій по оплаті мита і витрат.

Якщо адвокат неспроможний протягом понад тиждень виконувати професійні обов’язки, або має намір цього ж строку бути відсутнім у конторі, федеральне положення про адвокатуру зобов’язує його потурбуватися про заступника. Останній має ті ж повноваження, що й адвокат, якого він заступає.

зобов’язаний негайно повідомити правлінню про те, що всту­пає у сторонні трудові відносини або що в них сталися істотні зміни. Йдеться про те, що він тимчасово або певного строку використовуватиметься як суддя, державний службовець, вій­ськовослужбовець, на посаді в держапараті. За порушення спеціальних обов’язків правління палати може накласти на адвоката штраф.

Окремі обов’язки адвоката пов’язані з його відносинами з практикантом. Так, перший зобов’язаний наставляти другого як виконувати поставлені перед ним завдання, надавати ста­жисту різнобічну допомогу оволодівати професійними навич­ками.

Федеральний закон про адвокатуру надав адвокату право ознайомлюватися зі своєю особовою справою. При цьому він може робити витяги з документів або знімати з них копії.

Адвокат, призначений в суд процесуальним уповноваже­ним, може передати представництво іншому адвокату.

  1. Проведення порівняльного аналізу, виявлення загальних рис та відмінностей правового статусу адвоката.

Згідно німецького положення про адвокатуру адвокат є незалежною особою у сфері правосуддя. Будучи людиною вільної професії, адвокат є ком­петентним консультантом і представником інтересів громадян. Його право виступати з юридичних питань в загальних судах, арбітражних або адміністративних органах обмежується лише федеральним законом.

У Франції за законом, професія адвоката визначається як лібераль­на та незалежна. Це визначення створює право на специфічні організаційні форми здійснення професійної діяльності, охоронюваної законом та відокремленої від держави, що є ефек­тивним засобом забезпечення самоврядування, незалежності та захищеності. Віднині вони доповнили свою традиційну боротьбу за права та свободи людини діяльністю по захисту економічних прав та свобод.

Адвокати Бельгії є незалежними, то­бто вільними від зовнішнього конт­ролю, впливу, тиску, зовнішньої пі­дтримки під час виконання професійних обов’язків. Якщо сторона, що подає позов, не має адвоката і жоден з них не береться вести справу, то старшина може призначити адвока­та, який зобов’язаний її прийняти, якщо у нього відсутні істотні причини для відмови.

Адвокати Голландії є незалежними, тобто вони не підлягають зовніш­ньому контролю, впливу, тиску і не повинні жадати від будь-кого підтримки під час виконання професійних обов’язків. Якщо жоден адвокат відмовляється надати особі правову допомогу, суд може це зробити в порядку примусо­вого призначення, від якого адвокат не вправі відмовитися, якщо у нього відсутня вагома причина для цього.

Адвокати Данії є незалежними, тобто вільними від зовні­шнього контролю, впливу, тиску і сторонньої підтримки під час виконання професійних обов’язків. Адвокату не дозволяється вступати у справу від імені клієнта, якщо дору­чення не від нього або від органу, який має повноваження доручити адвокату ведення справи. Фундаментальним принципом є повна свобода клієнта обрати адвоката, якому він хоче доручити справу, або мати радником. Правило додержання професійної таємниці є для адвоката обов’язковим.

Адвокатам Іспанії заборонено рек­ламувати або давати оголошення про послуги як самостійно, так і через засоби масової інформації, Адвокат має право і зобов’язаний зберігати професійну таємницю. Це означає, що адвокат не може розголошувати факти або розк­ривати зміст документів, переданих йому клієнтом. Конфіде­нційність розглядається як безстрокове зобов’язання, яке по­ширюється на персонал Генеральної ради адвокатури, окремі адвокатські асоціації, їх дисциплінарні органи і адвокатські контори.

Адвокати КНР виконують такі основні функції: на запро­шення державних установ, підприємств, громадських органі­зацій, народних комун вони працюють як юридичні консуль­танти; за дорученням сторін беруть участь в цивільному про­цесі як їх представники; за дорученням обвинуваченого в кримінальній справі чи за призначенням народного суду вис­тупають як захисники; при порушенні справи потерпілим або стороною державного обвинувачення беруть участь в судовому розгляді.

У Латвії адвокат повинен захищати свого клієнта сміливо, чесно і наполегливо. Довіра і особиста чесність з ним є фундамента­льними принципами роботи присяжного адвоката. Він не повинен розкривати інформацію, отриману під час юридичної консультації, і після того, як припинив представ­ляти клієнта і діяти від його імені. Присяжний адвокат не повинен розкривати таємниці свого клієнта не тільки під час ведення справи, а й після закінчення представництва в справі. Адвокат має дбати про дотримання цих вимог й його помічни­ками.

У словенському внутрішньому законодавстві принципи адвокатури формулюються так: адвокат має додержувати істини, законності і етичних норм; бути чесним і мужнім; він має бути незалежним і об’єктивним, а також поря­дним, ввічливим і об’єктивним щодо іншої сторони та її законного представника й представників державних органів; адвокат має поінформувати клієнта про правозахисну процедуру, не повинен діяти в спосіб, не сумісний з професі­єю юриста, будь-яка особиста реклама заборонена; адвокатсь­ка контора має уникати дій, що призводять до конфлікту інтересів або відбиваються на особистій репутації й статусі адвоката; він не може бути практикуючим адвокатом, якщо займає посаду у виконавчому органі влади; адвокати конкурують тільки на професійній основі, реклама в суді чи засобах масової інформації заборонена. Він може тільки опублікувати оголошення, що відкрив (закрив) свою контору, або, що переніс її в інше місце; адвокат не може представляти клієнта у випадку, якщо останнього представляє інший адвокат і відсутня на це його згода, або якщо він не повідомив клієнта, що припиняє пред­ставляти його інтереси. Якщо адвокату доручено представляти. клієнта замість іншого, він повинен повідомити, що клієнт сплатив попередньому адвокату гонорар; адвокат не повинен протидіяти бажанню клієнта бути представленим іншим адво­катом чи отримати консультацію у іншого адвоката.

У Таїланді на адвоката покладаються такі права і обов’язки: зверта­тися до правління з письмовим викладенням своєї позиції щодо роботи Об’єднання адвокатів; бути обраним президен­том або членом правління; захищати честь професії і додержу­вати кодексу професійної етики тощо.

  1. Особливості допуску до адвокатської діяльності у Німеччині та Франції: порівняльний аспект.

Німеччина. До адвокатської діяльності у Німеччині допускається лише та особа, яка згідно з Законом про суддів виз­нається придатною до виконання суддівських обов’язків. Це означає, що особа має прослухати на юридичному факультеті вісім семестрів курсу правових наук і скласти державні іспити. Потім випускник практикує в усіх установах судової системи (суді, прокуратурі, нотаріаті, адвокатурі) як стажист-службовець протягом 2—3 років. Заро­бітну плату в цей період він отримує з фонду земельних органів влади. Після закінчення практики йому необхідно скласти другий державний іспит і лише після цього він обирає, ким бути: суддею, адвокатом або консультантом фірми з юри­дичних питань. Від цієї процедури звільняються лише ординарні профе­сори права університетів Німеччини.

Особа, яка відповідає вказаним вимогам, звертається з заявою до управління юстиції адміністративно-територіальної одиниці ФРН — землі, де вона постійно проживає. До прий­няття рішення управління юстиції зважує на думку правління тієї палати адвокатів, на території якої ця особа має намір практикувати. Правління палати зобов’язане негайно скласти експертний висновок про кандидата. Якщо протягом двох місяців палата його не подасть або не повідомить про причини . затримання, то управління юстиції вправі вважати, що у пра­вління палати відсутні підстави для відмови кандидату у доз­волі займатися адвокатською практикою (п.п. 1—4, 8 Федера­льного закону про адвокатуру).

Управління юстиції може зобов’язати претендента подати медичний висновок про стан здоров’я.

Закон передбачає випадки, коли особі, яка звернулася з клопотанням, може бути відмовлено у дозволі займатися адво­катською практикою. Ними є: якщо претендент за рішенням Федерального конституційного суду позбавлений одного з будь-яких основних прав; якщо при засудженні кримінальним судом він позбавлений права обіймати посади в державному апараті; якщо його виключено з адвокатури на підставі рішен­ня суду, коли від дня набрання цим рішенням чинності ще не минуло вісім років; якщо претендент на підставі судового рішення усунутий з посади судді у зв’язку з порушенням конституційних принципів або у дисциплінарному порядку звільнений зі служби в органах правосуддя; якщо вчинив проступок, в силу якого він не може займатися адвокатською діяльністю; якщо претендент протиправним способом висту­пає проти демократичного суспільного ладу; якщо претендент внаслідок фізичних або розумових вад, а також будь-якої ганебної пристрасті не в змозі належним чином виконувати функції адвоката; якщо він займається діяльністю, несумісною з професією адвоката, не рахується з авторитетом адвокатури.

Рішення управління юстиції землі про відмову у дозволі займатися адвокатською практикою, як і негативний відгук палати адвокатів (якщо він став причиною відмови), можуть бути оскаржені до адвокатського суду честі протягом одного місяця від дня вручення відмови. Про допуск до адвокатури претендент одержує від управ­ління юстиції землі свідоцтво. З цього часу особа набуває право зватися «адвокатом» за професією.

Деякі особливості має допуск до адвокатської діяльності у Федеральному верховному суді. До цієї діяльності можуть бути допущені лише особи, обрані комітетом по виборах адво­катів при зазначеному суді. Цей комітет складається з прези­дента і президентів сенатів в цивільних справах Федерального суду, членів президії федеральної палати адвокатів і президії палати адвокатів при Федеральному суді. До списку кандидатів вносяться лише ті адвокати, які досягли 35-річного віку і мають не менше п’яти років стажу безперервної роботи. Виборчий орган доби­рає з списків кандидатів таку кількість адвокатів, що удвічі перевищує число вакантних місць, і передає список відібраних осіб федеральному міністру юстиції, який остаточно вирішує, кому з них віддати перевагу. Адвокат при Федеральному суді не може працювати у іншому суді, крім вищих федеральних судових установ і Федерального конституційного суду.

Франція. Щоб стати адвокатом, необхідно або мати громадянство Франції або гро­мадянство однієї з країн Спільного ринку за умов взаємного визнання дипломів про вищу освіту. Іноземці мають відповідати ряду особливих вимог та пройти перевірку.

Особа, що претендує на звання адвоката, не може бути засуджена за діяння, несумісні з гідністю і порядністю, а також за серйозні дисциплінарні чи адміністративні проступки, чи покарана у зв’язку з банкрутством; не може вона займатися й діяльністю, що є несумісною з професією адвоката (наприк­лад, бути підприємцем).

Кандидат на посаду повинен: мати вищу юридичну освіту (диплом магістра права), скласти вступні іспити (два письмо­вих та один усний) в одному з регіональних центрів професій­ної підготовки, які працюють під егідою адвокатури та вищої школи, провчитися там рік, засвоїти теоретичний курс і прой­ти практичне стажування, скласти іспити (один письмовий і три усних). При додержанні цих умов, молодий юрист прий­мається до ордену адвокатів (колегії) та виголошує присягу такого змісту: «Присягаю, як адвокат, виконувати свої функції з гідністю, сумлінно, незалежно, чесно і гуманно». Після цього молодий адвокат проходить ще дворічне стажування зі спеціальності і одержує посвідчення. Лише після цього його ім’я вноситься до списків ордену як повноправного члена.

Колегії адвокатів докладають чималих зусиль для органі­зації високоякісної теоретичної підготовки (один рік), стажу­вання (два роки), а потім й перепідготовки адвокатів, що практикують. Так, в Парижі функціонує обладнаний за остан­нім словом науки і техніки центр професійної підготовки, бюджет якого складає приблизно 20 мільйонів франків на рік (дві третини дає колегія, одну третину — держава). Вітриною професії називають утримуваний Паризьким орденом адвока­тів інститут перепідготовки.

  1. Правий статус іноземних адвокатів у сучасних країнах світу

Можна виділити два основних підходи із приводу допуску іноземних адвокатів до адвокатської діяльності на території закордонної держави.

Перший складається в повній ізоляції правової системи держави від участі іноземних адвокатів. Наприклад, законодавство України до 2006 року передбачало, що адвокатською діяльністю може займатися винятково адвокат, який у свою чергу може бути тільки громадянином України.

Другий підхід, є найпоширенішим і обмовляє допуск іноземних адвокатів до адвокатської практики певними умовами й наявністю певних обмежень у їхній діяльності.

Питання регулювання діяльності іноземних адвокатів законодавства держав, як правило, відносять до своєї компетенції. Наприклад, відповідно до адвокатського законодавству республіки Вірменія іноземні адвокати здійснюють адвокатську діяльність у порядку, установленому національним законодавством, якщо інше не передбачено міжнародними договорами. Ця стандартна умова властива більшості законодавств і виходить із принципу примата міжнародного права над національним.

Для того, щоб іноземний адвокат міг займатися на території приймаючої держави адвокатською діяльністю необхідна його імплементація. Імплементація іноземного адвоката — це процедура по включенню або віднесенню іноземного адвоката в систему органів або індивідів  адвокатські послуги, що робить, на території держави. Національні законодавства для імплементації іноземного адвоката, як правило, ставлять певні умови, тільки при наявності, яких іноземний адвокат може займатися в цій державі адвокатською практикою. Які ж це можуть бути умови? Їх можна умовно розділити на дві групи: загальні й індивідуальні.

До загальних умов відносяться вимоги, що умовно відносяться до держави громадянства іноземного адвоката. Цією умовою, якою може обумовлюватися імплементація іноземного адвоката, є умова підписання двостороннього договору з державою громадянства іноземного адвоката. Так, іноземний адвокат може надавати юридичну допомогу, якщо із країною його громадянства підписаний відповідний договір.

До індивідуальних умов ставляться вимоги, пропоновані до особистих якостей і навичок іноземного адвоката. Особливо чітко пророблені індивідуальні умови допуску іноземних адвокатів у законодавстві США. Іноземний адвокат може займатися повіреною діяльністю в США якщо: досяг віку 26 років; був протягом  не менш п’яти років, адвокатом у державі свого громадянства; має необхідні моральні якості й задовольняє загальним вимогам придатності для членства в асоціація юристів штату, де він збирається здійснювати свою діяльність; має намір мати в США свій офіс; оплатив мито за видачу ліцензії.

Розглянута імплементація іноземних адвокатів відбувається в певній формі. Її здійснюють або спеціально на те вповноважені виконавчі органи державної влади або суд. У США, наприклад, саме суди видають іноземцеві ліцензію на практику як  юридичний консультант у конкретному штаті.

Після завершення акту імплементації, іноземні адвокати можуть займатися адвокатською практикою, але з деякими обмеженнями. Основним обмеженнями є заборона іноземного адвоката надавати юридичну допомогу, якщо ця діяльність буде пов’язана з державною таємницею. Ще одним можливим обмеженням є заборона деяких держав відкривати іноземним адвокатам на своїй території  адвокатські контори.

Необхідно відзначити, що норми міжнародного права дозволяють державам обмежувати допуск іноземних адвокатів до надання юридичної допомоги на їхній території. Конгрес ООН прийняв «Основні принципи щодо ролі юристів» (Гавана, 27 серпня — 7 вересня 1990 року), закріпивши в них положення відповідно до якого вимога будь-якої держави, про те, що адвокат повинен бути громадянином відповідної країни, не розглядається як дискримінаційне.

  1. Поняття професійної етики адвоката, її зміст та значення.

Професійна етика адвоката є дуже специфічною, має ці­лу низку особливостей, спірних питань і відрізняється від будь-яких інших різновидів професійної етики.

У Вступі до Загального кодексу правил для адвокатів країн Європейського Співтовариства вказується, що адвокату в будь-якому суспільстві уготована особлива роль. Йо­го обов’язки не обмежуються виконанням лише свого обов’язку в межах закону. Адвокат має діяти в інтересах права в цілому так само, як і в інтересах тих, чиї права і свободи йому довірено захищати; не лише виступати в суді від імені клієнта, а й надавати йому юридичну допомогу у вигляді порад і консультацій. У зв’язку з цим на адвоката покладається цілий комплекс зобов’язань як юридичного, так і морального характеру, які часто вступають у взаємну суперечність.

Правила професійної етики передбачають добровільне виконання їх тими, на кого поширюється їхня дія, для за­безпечення виконання адвокатом своїх обов’язків так, як це прийнято в будь-якому цивілізованому суспільстві. Не­додержання адвокатом цих правил карається аж до засто­сування до нього дисциплінарних санкцій.

Адвокатська професія стала істотною запорукою охоро­ни прав людини у відносинах з державною владою. Це фор­мує особливі вимоги до юридичної деонтології. Наявність чітких норм адвокатської етики, які відповідають зага­льновизнаним світовим стандартам, розглядається як не­обхідна передумова, без якої неможлива довіра до захис­ників.

Найбільш показовою в цьому плані є англійська адвока­тура. Тому зупинимося на її традиціях детальніше.

Права й обов’язки англійських адвокатів переважно за­лежать від того, ким — баристером чи соліситором — є ад­вокат. Загальним для них є обов’язкове додержання правил своїх корпорацій, що стосуються етики і професійних дій.

Професійна підготовка соліситора визначається юри­дичним товариством. У 1974 р. прийнято «Керівництво професійною підготовкою соліситорів». Цей документ зобов’я­зує соліситорів додержуватися встановлених правил своєї корпорації як щодо етики, так і щодо професійних дій. Так, соліситори мають зберігати професійну таємницю, фіксувати згідно зі встановленими нормами свої розрахун­ки з клієнтами, ставитися з повагою до суду та інших де­ржавних органів. Соліситор не повинен залучати клієнтів за допомогою реклами. Взаємовідносини між соліситором і клієнтом регулюються одночасно законом і кодексом про­фесійної етики. Вони передбачають сумлінне ставлення соліситора до виконання своїх обов’язків у всьому, що сто­сується клієнта.

Професійна поведінка баристерів регулюється спеціаль­ними правилами, які приймає рада баристерів. На цей час діє кодекс поведінки баристерів Англії та Уельсу від 1 січ­ня 1981 p., який встановлює такі обов’язки баристера: під­корятися приписам кодексу; не займатися тим, що є непо­рядним і може призвести до приниження професії адвока­та або правосуддя; додержуватися правил професійної ети­ки; бути компетентним у своїй професії. Баристер не може виступати в суді без парика, мантії та стрічок. Він не пови­нен вводити суд в оману, умисно приховувати від суду іс­тотні для справи факти. Водночас баристер не повинен ро­бити будь-які заяви про факти зізнання, які Йому довірив клієнт, не може займати правову позицію, яка усвідом­люється ним як шкідлива для інтересів клієнта.

У Франції адвокат має право на мантію своєї професії. Він наділений свободою висловлювання, імунітетом від об­винувачень у скоєнні злочину. Адвокат має право на гоно­рар, розмір якого залежить від угоди з клієнтом. Адвокат зобов’язаний додержуватися професійної таємниці; це одно­часно є і правом, і обов’язком. Адвокат не обмежений у ли­стуванні з клієнтом, навіть якщо той перебуває під вартою. Адвокату забороняється розголошувати професійну таємни­цю, таємницю слідства, відомості, одержані від клієнта. Він не може бути радником, представником чи захисником більш ніж одного клієнта в одній справі. Крім того, адвокат зобов’язаний дотримуватися гідного способу життя.

У Німеччині Федеральний закон про адвокатуру ре­гулює в окремому розділі права та обов’язки адвоката. Згідно з цим законом, адвокат зобов’язаний сумлінно ви­конувати свої професійні обов’язки. Він має бути гідним поваги і довіри, яких вимагає становище адвоката, і поза своєю професією. Якщо адвокат, до якого звернулися за юридичною допомогою, відмовився від її надання, він має негайно пояснити причини цього і відшкодувати шкоду, завдану клієнту внаслідок затримання такого пояснення з вини адвоката.

Таким чином, зміст принципу додержання адвокатами норм професійної етики полягає в тому, що у зв’язку зі специфікою своєї діяльності, крім законодавчих норм, во­ни задля забезпечення якісного виконання своїх обов’язків у повному обсязі повинні безумовно дотримуватися певних морально-етичних норм, вироблених згідно з Існуючими традиціями, практикою та світовими юридичними стан­дартами.

  1. 3агальні правила для адвокатів країн європейського співтовариства.

У Загальному кодексі правил для адвокатів країн Євро­пейського Співтовариства, прийнятому делегацією двана­дцяти країн-учасниць на пленарному засіданні у Страсбурзі в жовтні 1988р., зафіксовані загальні правила для адвокатів країн європейського співтовариства.

Правила висувають наступні основні вимоги до адвокатів: 1) незалежність. Завдання,   що   виконуються   адвокатом   у   процесі професійної  діяльності,  вимагають його абсолютної незалежності і відсутності будь-якого  впливу  на  нього,  пов’язаного,  в  першу чергу,  з  його  особистою  заінтересованістю  або з тиском зовні. Незалежне становище адвоката сприяє зміцненню в суспільстві довіри до  процедур  правосуддя  і  неупередженості суддів.  Таким чином, адвокату необхідно уникати будь-яких ущемлень власної незалежності і  не  поступатися  принципами обов’язку заради інтересів клієнта, суду або інших осіб. 2) Довіра і особиста порядність. Довірчі відносини між адвокатом і клієнтом  можуть  виникнути лише  у  разі  відсутності  в  останнього сумнівів щодо чесності і сумлінності адвоката.

3) Конфіденційність. Особливість професії адвоката полягає в тому,  що  він одержує  від клієнта відомості,  які той не буде повідомляти іншій особі,  а також іншу інформацію,  яку йому  належить  зберігати  в таємниці.   Довіра  до  адвоката  може  виникнути  лише  за  умови обов’язкового  додержання ним принципу конфіденційності. Таким чином,  конфіденційність є першорядним і фундаментальним правом та обов’язком адвоката. Адвокат   зобов’язаний  однаковою  мірою  зберігати  в таємниці як відомості,  одержані ним від клієнта, так і інформацію про клієнта, надану йому у процесі надання послуг клієнту.

Адвокат не повинен займатися саморекламою або прагнути до широкої популярності, якщо це визнається неприпустимим. В інших випадках адвокат може займатися  саморекламою  або  намагатися  до широкої  популярності  лише  у  межах  того,  наскільки це визнано допустимим у правилах, якими він керується.

Зайняття    саморекламою   вважається   допустимим   у випадках,  якщо  адвокат,  який  допустив  подібні  дії  з   метою залучення  потенційних клієнтів,  здатний довести,  що вищевказані дії були вчинені ним у місцях, де вони визнані допустимими.

  1. Кодекс професійної етики США. Порівняльна характеристика з європейськими стандартами.

Сьогодні основним актом в області професійної етики США є Модельні правила професійного поводження, прийняті Конгресом АВА 2 серпня 1983 р. З однієї назви «модельні» стає ясно, що у всіх штатах діють свої, розроблені на основі модельних, правила адвокатської етики. Крім того, а іноді замість того, в окремих штатах етичні норми професійної діяльності юристів утримуються в судових процедурних укладеннях. Називаються ці документи по-різному: Етичні кодекси, Кодекси професійної відповідальності, Кодекси адвокатської етики й т.п.

Приведемо етичні правила адвокатів США на основі Правил Професійної Етики Колегії Адвокатів штату Вірджинія, які встановлюють наступні етичні правила взаємин адвоката із клієнтом: 1. Компетентність. 2. Обсяг надання. Пропонує обов’язкове проходження вимогам і інтересам клієнта, а також указує ситуації, що вимагають виключення. 3. Старанність. Пропонує  адвокатові  проявляти  розумну старанність і швидкість, представляючи інтереси клієнта. 4. Подання  інформації  зі  справи.  Це  правило жадає від адвоката інформування клієнта в розумних межах  про  положення  справи,  уключаючи  можливості  врегулювання цивільної  суперечки  або  угоди  у  відношенні  подаваного  суду по кримінальній справі заяви. 5. Гонорар. Це правило жадає від адвоката встановлення «розумного» гонорару, а також визначає фактори, які повинні прийматися в увагу при встановленні «розумного» гонорару. 6. Конфіденційність інформації. Це правило чітко  визначає,  що  адвокат  не  повинен  розкривати  інформацію, що з до подання клієнта, якщо клієнт не погоджується на це, або якщо ця інформація завдасть шкоди клієнтові. 7. Зіткнення інтересів. Це правило описує ситуації, які мають потенційне зіткнення  інтересів,  якщо  з  погоджується  на  ведення  справи. 8. Наступність  між  державною  й  приватною сферами. Правило забороняє певні дії адвокатові,  що  є  посадовою  особою  або  з,  якщо при цьому очевидна вигода дійсного або колишнього клієнта. 9. Колишній суддя або арбітр. Правило забороняє адвокатові  подання  або  інтересів  у  справі,  якою  він займався як  суддя; і так далі. Також даного правила встановлюють наступні положення етичних норм адвокатів відносно: радників і незацікавлених осіб, зв’язку з особами, що не є клієнтами адвоката, юридичні фірм і асоціацій, діяльності на благо суспільства.

Необхідно відзначити, що в цілому етичні правила США збігаються з етичними правилами в Європі, але деякі положення можуть відрізнятися від штату до штату.

  1. Етичні основи взаємовідносин адвоката з клієнтом, органами слідства, судом, колегами.

У Загальному кодексі правил для адвокатів країн Євро­пейського Співтовариства, прийнятому делегацією двана­дцяти країн-учасниць на пленарному засіданні у Страсбурзі в жовтні 1988р., зафіксовані етичні основи взаємовідносин адвоката з клієнтом, органами слідства, судом, колегами.

  1. Взаємодія з клієнтом. При здійсненні професійної   діяльності   адвокат зобов’язаний керуватися вказівками клієнта. Проте він також може керуватися      вказівками  іншого адвоката,  який діє в інтересах того самого клієнта,  або котрий бере участь у розгляді тієї самої справи за розпорядженням будь-якого компетентного органу. Адвокат зобов’язаний надавати консультації клієнту або виступати від його імені своєчасно, сумлінно і старанно. Він несе особисту відповідальність за відмову слідувати вказівкам клієнта. Адвокат  зобов’язаний  інформувати  клієнта  про  процес  розгляду дорученої йому справи. Адвокат  не  повинен  здійснювати ведення справи,  яка явно не відповідає рівню його професійної компетенції,  без участі в ній іншого адвоката, що володіє необхідною компетенцією. Адвокат не повинен користуватися  правом  відмови  від подальшої  участі в розгляді справи,  якщо обставини не дозволяють клієнту знайти іншого юриста без шкоди для клієнта. Адвокат може відмовитися від надання консультацій двом або  кільком клієнтам,  які беруть участь у розгляді однієї і тієї самої  справи,  або  не  виступати  від  їх  імені  за   наявності суперечності   інтересів   клієнтів   або   обґрунтованої  загрози виникнення такої суперечності. Адвокат  зобов’язаний  припинити  обслуговування  обох клієнтів у випадку вступу їх інтересів у взаємну  суперечність,  а також   при  виникненні  загрози  порушення  конфіденційності  або загрози незалежності самого адвоката. Адвокат не повинен укладати pactum de quota litis. Під  поняттям  «Pactum  de  quota  litis»  розуміється угода,  що  укладається  між  адвокатом  і  клієнтом  до винесення остаточного рішення стосовно  справи,  що  розглядається,  у  якій клієнт  є однією із заінтересованих сторін.  Згідно з вищевказаною угодою клієнт зобов’язується у випадку винесення рішення  на  його користь  виплатити адвокату винагороду у вигляді грошової суми або у будь-якій іншій формі.
  2. Взаємодія з органами слідства, судом. Адвокат, який знаходиться у залі суду або бере безпосередню участь в судових слуханнях у процесі розгляду справи, зобов’язаний підкорятися правилам поведінки, прийнятим для даного суду. Адвокат завжди   зобов’язаний  належним  чином  додержуватись принципу чесного провадження судового розгляду. Наприклад, адвокат не   повинен   вступати   в  контакт  з  суддею  без  попереднього повідомлення адвоката протилежної сторони  або  пред’являти  судді будь-які   предмети  або  документи,  не  попередивши  заздалегідь адвоката іншої сторони,  за винятком випадків,  коли  подібні  дії передбачені правилами проведення судового розгляду. При додержанні належної поваги до суду,  адвокат зобов’язаний захищати інтереси клієнта сумлінно  і  з  максимальною  для  нього вигодою, проте, не виходячи за передбачені законодавством межі. Правила, що визначають поведінку адвоката в суді, поширюються також на його відносини з арбітрами і будь-якими  іншими  особами, котрі  виконують  обов’язки,  які  прямо  чи  побічно пов’язані із судочинством, у тому числі і на непрофесійній основі.
  3. Взаємодія з колегами. Дух корпоративної єдності представників даної професії передбачає відносини довіри і співробітництва,  що підтримуються адвокатами  між  собою  і  заради  інтересів  клієнтів,  і  заради уникнення непотрібних спорів. Протиставлення професійних інтересів інтересам правосуддя та інтересам осіб,  які його  домагаються,  в будь-якому випадку визнається невиправданим. Адвокат зобов’язаний визнавати усіх інших адвокатів із тих держав,  що входять до Співтовариства, як колег за професією і ставитися до них згідно з нормами порядності і поваги. Адвокат не повинен повідомляти будь-які  відомості  особі  за наявності інформації,  що вона або одержує консультації від іншого адвоката,  або цей адвокат виступає  від  її  імені,  за  винятком наявності   домовленості   з   адвокатом  протилежної  сторони  (в останньому  випадку  адвокат  зобов’язаний  інформувати адвоката протилежної сторони про повідомлення ним вищевказаних відомостей).
  1. Адвокатська таємниця та адвокатська недоторканість.

Адвокатська таємниця полягає в тому, що адвокат без дозволу клієнта не має права скористатися отриманого ін­формацією. Він не повинен ігнорувати волю дієздатної людини, яка виступає проти розголошення довірених йому відомостей. Предметом адвокатської таємниці є питання, з яких громадянин або юридична осо­ба зверталися до адвоката, суть консультацій, порад, роз’яснень та інших відомостей, одержаних адвокатом при здійсненні своїх професійних обов’язків. Дані попередньо­го слідства, які стали відомі адвокату у зв’язку з виконан­ням ним своїх професійних обов’язків, можуть бути розго­лошені тільки з дозволу слідчого або прокурора. Правило додержання професійної таємниці є обов’язковим. Воно не може бути обійдено навіть на прохання клієнта. Даючи свідчення в суді, адвокат перебу­ває у привілейованому становищі щодо нерозголошення.

Для забезпечення діяльності й ефективного виконання обов’язків адвоката держава повинна гарантувати адвокатську недоторканність. Адвокатська недоторканність відповідно до міжнародного права — це комплекс заходів у вигляді організаційних і правових гарантій правозахисної діяльності адвоката з метою забезпечення юридичної допомоги населенню, що є забезпеченням одного з основних прав людини — права на захист.

У цьому зв’язку необхідно орієнтуватися на міжнародно-правові акти про ролі адвоката в суспільстві. Це основні положення про ролі адвокатів, прийняті восьмим Конгресом ООН по попередженню злочинів в 1990 р. у Нью-Йорку, основні принципи, що стосуються ролі юристів, прийняті Конгресом ОНН по попередженню злочинності й обігу із правопорушниками, що проходив у серпні-вересні 1990 р. у Гавані. У них говориться, що уряд країн повинен забезпечити адвокатам наступне: 1) можливість виконувати всі їхні професійні обов’язки без залякування, перешкод, занепокоєння й недоречного втручання; 2) можливість вільно пересуватися й консультувати клієнта у своїй країні й за кордоном; 3) неможливість покарання або погрози такого й обвинувачення, адміністративних, економічних і інших санкцій за будь-які дії, здійснювані відповідно до  визнаних професійних обов’язків, стандартами й етичними нормами; 4) у випадках, коли адвокат перебуває під загрозою у зв’язку з виконанням професійних обов’язків, вони повинні бути адекватно захищені владою; 5) адвокати не повинні ідентифікуватися зі своїми клієнтами й справами клієнтів у зв’язку з виконанням своїх професійних обов’язків; 6) суд або адміністративний орган не повинні відмовляти у визнанні права адвоката, що має допуск до практики, представляти інтереси свого клієнта, якщо цей адвокат не був дискваліфікований відповідно до  національного права й практикою його застосування; 7) адвокат повинен мати карний і цивільний імунітет від переслідувань за стосовні до справи заяви, зроблені в письмовій або усній формі, при сумлінному виконанні свого боргу й здійсненні професійних обов’язків у суді, трибуналі або іншому юридичному або адміністративному органі; 8) обов’язком компетентної влади є забезпечення адвокатові можливості своєчасного ознайомлення з інформацією, документами й матеріалами справи, а в карному процесі — не пізніше закінчення розслідування й до судового розгляду справи.

Ці положення визнані міжнародним співтовариством, і будь-яка країна, що вважає себе правовою державою, повинна привести своє законодавство у відповідність із перерахованими нормами.

  1. Поняття та зміст професійної етики адвоката за законодавством Франції

У Франції адвокат має право на мантію своєї професії. Він наділений свободою висловлювання, імунітетом від об­винувачень у скоєнні злочину. Адвокат має право на гоно­рар, розмір якого залежить від угоди з клієнтом. Адвокат зобов’язаний додержуватися професійної таємниці; це одно­часно є і правом, і обов’язком. Адвокат не обмежений у ли­стуванні з клієнтом, навіть якщо той перебуває під вартою. Адвокату забороняється розголошувати професійну таємни­цю, таємницю слідства, відомості, одержані від клієнта. Він не може бути радником, представником чи захисником більш ніж одного клієнта в одній справі. Крім того, адвокат зобов’язаний дотримуватися гідного способу життя.

Розкриємо ці правила адвокатської етики у Франції більш широко.

Основне правило адвокатської етики у Франції – адвокат чесно виконувати свої обов’язки перед клієнтом, судом, своїми колегами чи громадськістю, користу­ватися у них довірою. Хто не заслуговує на це, — не може бути адвокатом. Ці вимоги стосуються не тільки професійної поведі­нки, а й приватного спілкування, яке має важливе значення у взаємних стосунках.

Юрист зобов’язаний компетентно вести справи від імені клієнта, надавати останньому послуги на високому професій­ному рівні. Усі юристи мають пройти предметну перепідготовку та нагромадити відповідний досвід перед тим, як вони почнуть брати участь у веденні цивільних та кримінальних справ. Вимоги, що містяться в правилах, є дещо завищеними ніж формальний дозвіл займатися юридичною практикою. Йдеть­ся про обізнаність в законах щодо конкретних питань, а також у змінах в галузевому законодавстві. Обслуговування включає зобов’язання постійно інформу­вати клієнта про вжиті заходи, відповідати на поставлені ним запитання, давати йому поради, а також мати відповідні мате­ріали, що будуть використані в інтересах клієнта. Усі юристи заінтересовані у високому рівні конкурентоспроможності сво­єї діяльності.

Від юриста вимагається повна об’єктивність. Коли від нього очікують порад, він зобов’язаний дати клієнту всебічний аналіз ситуації, посилаючись на відповідний закон та свій власний досвід. Юрист не повинен переоцінювати шанси клі­єнта на успіх, а в окремих випадках йому не завадить зверну­тися за порадою до свого більш обізнаного колеги. Якщо виявиться, що під час певних дій допущено помилки, він має негайно повідомити про це клієнта і вжити заходів до їх виправлення.

Юрист повинен зберігати у таємниці всю інформацію, про яку йому стало відомо під час виконання свого професій­ного обов’язку, і не розголошувати її без дозволу клієнта. Стосунки між юристом і клієнтом мають привілейовану осно­ву в системі прецедентного права, на якій ґрунтуються відно­сини між сторонами, коли клієнт відверто ознайомлює свого юриста з усіма подробицями, що мають значення для справи.

Юрист не може давати поради чи представляти обидві сторони у справі, не отримавши на це їх згоди. Сторони мають протилежні інтереси, тому юрист може обслуговувати лише одну сторону. Разом з тим клієнти, після повного ознайомлен­ня з усіма деталями справи, можуть вирішити, що їх інтереси захищатимуться одним юристом. Якщо ж виникла конфліктна ситуація, юрист має повністю припинити свою участь у справі.

Адвокат має додержувати усіх законів та правил, що регулюють недоторканість майна та коштовностей клієнта. Якщо для цього відсутні певні правила, адвокат має ставитись до них, як і сам власник.

Адвокат має поводитись в суді чемно і ввічливо, в рамках закону. Він може «без побоювань порушувати перед судом будь-яке питання, парирувати кожний аргумент протилежної сторони, сприймати усе, навіть, неприємне, якщо це може допомогти клієнтові» і прагнути «залучити для вигоди клієнти та захисту його інтересів усе, що дозволяє закон» У основу цього правила закладений постулат, що суд неспроможний функціонувати, якщо адвокат поводиться нечесно. Незважаю­чи на те, що судді мають повноваження контролювати все, що відбувається у суді, це правило є набагато ширшим, ніж зви­чайна влада суду.

Судді вживають санкції проти тих, хто ображає їх гідність, порядок в суді або заважає судочинству. Ця влада є досить широкою. Нещодавно Кабінет міністрів федерального уряду без належних підстав критикував суд, його було притягнено до відповідальності, після чого він був змушений вибачитись. Реакція в таких випадках може бути різною: від догани і вибачення, до грошового штрафу і, навіть, до ув’язнення.

За цим правилом юрист як адвокат має, зокрема, контро­лювати поведінку клієнта, не дозволяючи йому поводитися непристойно, бути впевненим, що останній не вводить в оману суд. Якщо клієнт вдається до неправдивих свідчень і адвокат не може цьому запобігти, він зобов’язаний відмежува­тися від них. Одночасно адвокат може перешкодити суду бути упередженим щодо певних фактів або закону. Адвокати не повинні підмовляти свідка давати показання, чи натискувати або обвинувачувати його. Адвокат не може спілкуватися з суддею, якщо між ними існують особисті або інші неофіційні стосунки, котрі можуть зашкодити правосуддю.

Адвокат, який працює у державній установі, при вико­нанні службових обов’язків має поводитися за правилами, які вимагають зразкової поведінки. Ці правила стосуються адво­катів, обраних або призначених на ці посади на офіційному рівні. Взагалі, до адвокатів, що працюють у державних устано­вах, ставляться більші вимоги, ніж до тих, хто займається приватною практикою. З огляду на це адвокат має бути зібра­ним, коректним, завжди пам’ятаючи про імідж своєї професії.

Адвокат не може домовлятися або одержувати гонорар більший, ніж це встановлено відповідним актом; використову­вати будь-які кошти, довірені йому клієнтом, для оплати наданих послуг без однозначної згоди клієнта.

Адвокати покликані привертати увагу громадськості до поліпшення судочинства, оскільки професійно розуміючи за­кон, вони детально обізнані з роботою правозастосовних уста­нов, організацій. Виходячи з цього, адвокати ініціюють процес пошуку можливостей для вдосконалення системи судо­чинства.

Адвокати мають надавати послуги у доступних для грома­дськості формах й ефективному та зручному вигляді, підтри­муючи повагу і довір’я до юридичної професії. Якщо особі потрібна правова допомога, вона повинна мати можливість без зайвого клопоту знайти кваліфікованого адвоката.

Адвокат повинен оберігати авторитет своєї професії, по­відомляючи юридичному товариству про випадки його ума­лення. Привернення уваги до малих проблем може попередити настання серйозних труднощів, і якщо їх виявити своєчасно, це допоможе краще захистити від них суспільство.

  1. Обов’язкова участь адвоката-захисника у кримінальному процесі за законодавством Великобританії, Німеччини та США: проведення порівняльного аналізу.

Великобританія. Здійснювати захист інтересів сторони в кримінальній справі можуть як баристери, так і соліситори. Вони мають право брати участь в усіх судових інстанціях. Але в Палаті лордів, Апеляційному і Високому судах це право за деякими винятками належить виключно баристеру. При розгляді кри­мінальних справ Судом корони (він розглядає справи з участю присяжних) обвинувачення підтримують баристери.

Баристер не має права особисто укладати з клієнтами угоди на ведення кримінальної справи і одержання від них гонорару. Це є прерогативою соліситора, який розпочинає переговори з клієнтом, домовляється про гонорар, збирає докази.

Повноваження адвокатів із загалу соліситорів ґрунтується на дорученні, яке дається клієнтом у письмовій формі. У такий спосіб виникають договірні відносини.

Надання правової допомоги малозабезпеченим громадя­нам регулюється законом 1974 р. про правову допомогу, який консолідував ряд чинних актів законодавства. Відповідні кош­ти сплачуються «фонду правової допомоги», що фінансується державою. Малозабезпеченою вважається особа, річний доход якої не перевищує 1790 фунтів стерлінгів. Наявність коштів у особи, яка звертається по правову допомогу, визначається комісією з додаткових асигнувань міністерства соціального забезпечення.

Німеччина. Кримінальний процес Федеративної Республіки Німеччини за своєю фо­рмою належить до змішаного конти­нентального процесу з самостійною стадією попереднього слідства Чинний КПК ФРН, як це визнають і процесуалісти Ні­меччини, не дає визначення поняття обвинуваченого. У § 157 є щодо цього лише дві конкретні процесуальні фігури: особа, проти якої прокурором висунуте публічне обвинувачення (по­зов), й особа, щодо якої судом прийнято рішення про відкрит­тя головного судового провадження (за термінологією кримі­нально-процесуального закону України — віддання до суду). Згідно з § 137 КПК обвинувачений має право на будь-якому етапі провадження користуватися допомогою захисни­ка, який допускається до участі в справі зі стадії попереднього слідства до початку допиту обвинуваченого суддею, котрий вирішує питання про підставність арешту за підозрою у вчи­ненні кримінальне караного діяння. Запросити захисника для участі у справі може як сам обвинувачений, так і його законний представник. Для захисту може бути запрошено й декількох захисників, але не більше трьох (§ 227 КПК). Поряд з цим закон не дозволяє одному захиснику захищати одночасно кількох осіб (§ 146 КПК).

КПК ФРН передбачає у ряді випадків обов’язкову участь захисника у кримінальному процесі. Як свідчать німецькі про­цесуалісти, у їхньому товаристві неодноразово обговорювалось питання про те, чи є потрібним обов’язковий захист, і завжди більшість висловлювалася за його збереження.

Випадки обов’язкової участі захисника поділяються на безумовні (абз. 1 § 140) і умовні (абз. 2 § 140). До першої групи належать ті з них, коли справа розглядається по першій інста­нції вищим чи звичайним земельним судом, або обвинуваче­ному ставиться у вину вчинення злочину. Згідно з абз. 1 § 12 КК ФРН злочинами є кримінальне карані діяння, за які передбачено позбавлення волі на строк не менше одного року або більш суворе покарання, коли може бути застосовано позбавлення права займатися певною професійною діяльністю, або обвинувачений є глухим, німим або сліпим, коли обвинувачений на підставі суддівської постанови понад три місяці перебував у закритій установі (під вартою, у лікарні, в опікунській установі) і звільнений менш як за два тижні до початку головного судового розгляду, коли обвинуваченого відповідно до § 81 КПК поміщено до лікарні для підготовки висновку про його психічний стан, коли застосовуються при­мусові заходи медичного характеру (виправлення і безпеки) щодо неосудної або недієздатної особи, а також, коли усунено з процесу захисника, який брав участь в ньому до цього.

У інших випадках голова суду за клопотанням обвинува­ченого або з своєї ініціативи має призначити захисника, якщо участь останнього є бажаною, з огляду на тяжкість діяння або. складність справи, або коли є очевидним, що обвинувачений не зможе себе самостійно захистити. В усіх перелічених випадках (абз. 1 і 2 § 140 КПК) особі, яка ще не має захисника, такий призначається, як тільки обвинуваченого буде викликано для ознайомлення з обвину­вальним висновком.

США. Право обвинуваченого на захист і на захисника було закрі­плено в 1791 р., у VI Поправці до Конституції США 1787 р. В усіх випадках кримінального переслідування «обвинувачений має право… вимагати, щоб його ознайомили з мотивами обви­нувачення і надали очну ставку зі свідками, які показують проти нього; обвинувачений може також вимагати примусового вик­лику своїх свідків і користуватися допомогою адвоката для захисту»‘.

Протягом тривалого часу дія VI Поправки не поширюва­лась на досудові стадії. З середини XX століття з метою усунення порушень прав обвинуваченого Верховний суд США сформулював ряд правил, пов’язаних з участю адвоката в досудових стадіях процесу.

Оскільки на обвинувача у кримінальному процесі США не покладається обов’язок всебічного, повного і об’єктивного дослідження обставин справи, захисникам доводиться вдава­тися до власного розслідування з метою одержання доказів, що виправдовують обвинуваченого або пом’якшують його ві­дповідальність.

За загальним правилом, адвокат вступає в процес на боці обвинуваченого на його запрошення.

Обвинуваченому, який не має коштів, щоб запросити адвоката, останній призначається, але лише у справах про найбільш тяжкі злочини2.

Прийнятий у 1964 р. федеральний закон про кримінальну юстицію закріпив надання юридичної допомоги кожному обвинуваченому, який її потребує, і визначив умови забезпечен­ня безплатного захисту для матеріально неспроможних. Адво­катські колегії було зобов’язано подати до відповідних судів список адвокатів, які призначалися здійснювати захист зазна­ченої категорії обвинувачених. Особа, що звертається з таким клопотанням, має під присягою підтвердити відсутність у неї необхідних для захисту коштів. Якщо перевіркою буде встано­влена неправдивість цих даних, заявник підлягає судовій від­повідальності.

Закон 1964 р. позитивно вплинув на вирішення проблеми організації захисту вказаної категорії обвинувачених в спра­вах, що підлягають юрисдикції штатів. Суд штату може приз­начити захисника неспроможному обвинуваченому у справі про фелонію (про тяжкий злочин), а в окремих випадках — і про місдимінор.

  1. Обов’язкова участь адвоката у цивільному процесі за законодавством Франції Німеччини, Росії.

Франція. Представництво в цивільному процесі Франції здійснює­ться адвокатами і повіреними. Адвокат виконує подвійну фу­нкцію: консультативну — висловлює свою думку і дає поради та захисну — забезпечує захист своїх довірителів, виступає від імені однієї з сторін в процесі у цивільних справах. Адвокат в судах загальної юрисдикції її сторону не представляє (у цій ролі виступає повірений), а лише допомагає суду. Відносини між представниками і довірителями в цивіль­ному процесі Франції ґрунтуються на договорі доручення, який виникає після прийняття адвокатом на себе виконання функцій представництва в цивільному процесі за відповідну винагороду. Довіритель зобов’язаний виконати обов’язки перед дові­реним і сплатити винагороду. Представництво може бути при­пинено достроково з відкликанням доручення довірителем, а також відмовою представника від доручення. Повноваження (мандат) на представництво адвоката по­ширюється на усі процесуальні дії.

Німеччина. У німецькому цивільному судочинстві участь адвоката є в ряді випадків необхідним. Так, наприклад, якщо вартість предмета спору перевищує 10.000 ДМ, то така справа стосується  компетенції суду більше високого рівня — земельного суду (Landgerіcht), у якому участь адвоката обов’язкова. Так само, розгляд цивільних справ у другій інстанції Вищим земельним судом (Oberlandesgerіcht) і Федеральним Верховним суді вимагає участь адвоката. При цьому найважливіші  процесуальні дії: подача позовної заяви, ведення переговорів, подача клопотань і скарг і ін. можуть відбуватися тільки допущеним у суді адвокатом, у противному випадку вони не будуть мати юридичної сили. У таких випадках своєчасне залученні адвоката ставиться в обов’язок. У противному випадку суд може винести рішення не на вашу користь, причому не на підставі розгляду обставин справи, а винятково через  вашу неучасть у процесі (т.зв. Versaumnіsurteіl). До речі скаргу на винесення такого вироку може подати також тільки допущений у суді адвокат. Також необхідна участь адвоката шлюборозлучних процесах (Scheіdungsverfahren) у відділенні суду першої інстанції, що називається сімейним судом (Famіlіengerіcht).

Росія. На сьогодні у ЦПК РФ (ст. 50) закріплена обов’язкова участь адвоката у випадку відсутності представника у відповідача, місце проживання якого невідомо. Дане положення забезпечує такому відповідачеві право на судовий захист, гарантований ст. 46 Конституції РФ, і здійснення цивільного судочинства умовах змагальності й рівноправності сторін. Закріплюючи принцип змагальності, держава разом з тим повинна забезпечити всім суб’єктам судового процесу рівні процесуальні права й можливості по їхній реалізації в незалежності від їхнього матеріального становища.

  1. Правове положення адвоката в Європейському суді з прав людини.

Правове положення адвоката в Європейському суді з прав людини визначається згідно з Регламентом Європейського Суду за прав людини та прецедентною практикою Європейського Суду з прав людини.

Необхідно відзначити, що у ході розгляду справи в Європейському суді адвокат має ті ж повноваження, що й заявник. У цілому зміст прав і обов’язків адвоката в Суді є схожим із правами й обов’язками представника за українським процесуальним законодавством. Разом з тим у Європейському суді права адвоката відносно  збору й подання доказів трохи ширше, ніж у українському праві. Зокрема, адвокат вправі представляти Європейському суду як  докази письмові показання свідків, яких він опитав самостійно, навіть якщо цих свідків не опитували національні слідчі, судові або інші органи. Крім того, адвокат може представляти в Суд докази, які не представлялися в національні суди та не оцінювалися ними.

Якщо в адвоката виникають труднощі з наданням Європейському суду яких-небудь документів у зв’язку з відмовою органів влади відповідача представити їх йому, він може звернутися в Суд з відповідним проханням. Звичайно Європейський суд запитує у влади держави-відповідача матеріали кримінальних справ, внутрішніх перевірок та іншу інформацію, якою може володіти тільки влада.

Із приводу обов’язків адвоката слід особливо зазначити необхідність відповідати на листи Європейського суду у встановлений строк, оскільки непредставлення відомостей або документів, запитаних Секретаріатом Суду, може привести до того, що скарга взагалі не буде розглянута. На адвоката покладається також обов’язок письмово сповіщати про будь-які зміни своєї адреси під час розгляду в справі (п. 6 ст. 47 Регламенту Суду), оскільки Секретаріатом Суду йому направляється вся кореспонденція.

Адвокат повинен проявляти повагу до Європейського суду, його суддям і співробітникам Секретаріату, а також до влади держави-відповідача. У випадку вживання адвокатом образливих, загрозливих і провокаційних висловлень стосовно  перерахованих суб’єктів Європейський суд може відсторонити адвоката від участі в справі або визнати скаргу неприйнятної для розгляду по суті через зловживання правом за змістом пункту 3 ст. 35 Конвенції.

Немаловажно, що на будь-якій стадії провадження Європейський суд вправі відсторонити адвоката від участі в справі та наказати заявникові знайти собі іншого представника. Відповідно до підпункту «b» п. 4 ст. 36 Регламенту підставою для подібного рішення Суду можуть стати обставини справи або дії адвоката. Зокрема , у силу статті 44 «d» Регламенту Суду адвокат може бути відсторонений від участі в розгляді через  використання їм образливих, безпідставних і несумлінних доводів, а також занадто великих аргументів.

  1. Процесуальні основи діяльності адвоката в Європейському суді з прав людини. Вимоги, яким повинен відповідати адвокат-представник у Європейському суді з прав людини.

Процесуальні основи діяльності адвоката в Європейському суді з прав людини визначаються Конвенцією «Про захист прав людини та основних свобод» й Регламентом Європейського Суду, які не містять спеціальних вимог до складання доручення й не зобов’язують засвідчувати її в нотаріуса або інших осіб, а також проставляти на ній апостиль. Вимоги національного законодавства не поширюються на оформлення доручення в Європейський суд, тому йому досить представити доручення в простій письмовій формі, складену на бланку, розміщеному на офіційному сайті Суду. Причому цей бланк повинен бути підписаний не тільки заявником, але й адвокатом, оскільки Європейський суд думає, що підписання адвокатом доручення підтверджує прийняття їм на себе повноважень.

Наприклад, у справі «Рябов проти Росії» Європейський суд указав, що відповідно до пункту 3 ст. 45 Регламенту Суду для цілей розгляду в Європейському суді досить доручення в писемній формі, тому що ні Конвенція, ні Регламент Суду не передбачають якої-небудь форми посвідчення доручення якою-небудь національною владою. Суд нагадав, що він уже неодноразово відхиляв заперечення влади Росії про порушення передбачених російським законодавством правил посвідчення доручень, і відзначив, що бланк доручення може бути надрукований або заповнений від руки або заявником, або його адвокатом, або будь-якою третьою особою. Для Європейського суду важливо лише, щоб документ про повноваження явно з наступне: заявник довірив подання своїх інтересів у Європейському суді адвокатові, а той прийняв це доручення.

Основна вимога, пропонована Європейським судом до доручення, полягає в тому, що вона повинна бути дійсно видана заявником при наявності в того розуміння. Суд виходить із презумпції дійсності представленої йому доручення. На цій підставі обов’язок доведення, що скарга подана без ведена з або доручення отримана шляхом обману, покладає на відповідача.

Зокрема, у справі «Великова проти Болгарії» Європейський суд указав: просте письмове повноваження повинне визнаватися дійсним для представництва в Європейському суді, якщо тільки влади відповідача не доведуть, що цей документ, складений при відсутності на те згоди або розуміння в заявника.

Для представництва в Європейському суді не завжди обов’язково оформляти додаткове доручення, оскільки може виявитися досить доручення для представлення інтересів в будь-яких судових органах, якщо не уточнюється, що це, наприклад, українські суди. Так, у справі «Носов проти Росії» Європейський суд вказав, що йому не обов’язково пред’являти окреме доручення на представництво, досить доручення, у якій зазначене, що заявник доручив представляти його інтереси «у всіх судових органах», якимось і є Європейський суд.

При наявності особливих обставин у справі не виключається подача скарги без доручення. Наприклад, без доручення від імені корпорації може звернутися її акціонер (учасник), що володіє не менш 90% акцій (часток) компанії. Крім того, без доручення можна подати скаргу у випадку, коли заявник не має доступу до зовнішнього миру або є душевнохворим.

У справі «Худобин проти Росії» влади держави-відповідача заявили, що видана заявником доручення недійсне, оскільки підписано не їм самим, а його матір’ю. Вони також указали на відсутність підпису представника в дорученні. Влади просили Європейський суд одержати від заявника письмове схвалення по кожному з документів, поданих його представником, або припинити розгляд справи. Суд нагадав, що він не пред’являє спеціальних вимог до оформлення доручення, навіть якщо такі втримуються в національному законодавстві (ст. 45 Регламенту Суду). Він відзначив, що мати заявника представляла його інтереси в російських судах, а сам заявник з душевнохворим і мав потребу в примусовому медичному лікуванні. При таких обставинах Суд порахував, що мати заявника могла діяти від його імені під час підписання доручення.

Відносно вимог, що відносяться до особи адвоката, то Європейський суд пропонує заявникові вказати адвоката, що буде представляти його інтереси при подальшому розгляді. Відповідно до  пункту 5 ст. 36 Регламенту Суду до адвоката пред’являється вимога, що він повинен «у достатньому ступені» володіти англійською або французькою мовою, оскільки з моменту комунікації скарги всі змагальні документи сторін у справі (заперечення, меморандуми, клопотання), а також вся кореспонденція, що направляється Секретаріатом Суду, складаються на англійській або французькій мові (п. 3 ст. 34 Регламенту Суду). І хоча Європейський суд може дозволити адвокатові на його прохання користуватися при складанні змагальних документів офіційною мовою держави-відповідача, проте за загальним правилом адвокат повинен як мінімум пасивно володіти однією з офіційних мов Суду.

  1. Підготовка та проведення справи в Європейському суді з прав людини.

Підготовка адвоката до справи в Європейському суді з прав людини складається з наступних етапів:

  1. Вивчення матеріалів судових розглядів в Україні, вивчення інших документів, пов’язаних із предметом скарги Заявника в Європейський Суд з прав; 2. вивчення й порівняльний аналіз європейських стандартів і українського законодавства, матеріалів Пленумів Верховного Суду України, Вищого Господарського Суду України й Постанов Конституційного Суду; 3. розробка позиції в справі на основі європейських стандартів з прав людини по аналогічних справах, короткий аналіз судової практики Європейського Суду з прав людини, 4. підготовка досьє в справі, 5. підготовка скарги заявника у Європейський Суд з прав людини, оформлення та направлення скарги заявника в Європейський Суд з прав людини, 6. підготовка й обмін змагальними листами й документами з Європейським Судом з прав людини.

Мабуть, найбільш складною стадією роботи адвоката в Європейському Суді є підготовка скарги. При її підготовці він повинен як мінімум керуватися наступними засадами:

  1. Скарга повинна бути подана належною особою (rafіone personae).
  2. Скарга повинна бути подана на порушення права, гарантованого Конвенцією й додатковими Протоколами до неї (ratіone materіae).
  3. Порушення прав повинне мати місце після ратифікації державою — відповідачем Конвенції.
  4. Порушення прав повинне відбутися в межах юрисдикції держави-відповідача (ratіone locі).
  5. Скарга подається тільки після використання всіх ефективних засобів захисту права на національному рівні.

Чи є судовий порядок захисту права ефективним засобом захисту, вирішується в кожному конкретному випадку. Якщо можна довести неефективність даного засобу захисту права (хоча б за допомогою статистики задоволення подібних скарг), скарга може бути спрямована прямо в Європейський суд.

У випадку, якщо розгляд справи на національному рівні істотно затягується й частина скарги містить посилання на це «перешкода», це саме по собі може вважатися «виснаженням» ефективних засобів захисту (Matos Е. Sіlva vs Portugal (1996).

  1. Скарга повинна бути подана протягом шести місяців після ухвалення остаточного рішення на національному рівні (ratіone temporіs).

Суд приймає до розгляду скарги тільки протягом шести місяців, уважаючи з дати винесення остаточного судового рішення або з дати, коли громадянин довідався про це. Якщо в якій-небудь ситуації для відновлення порушення права не передбачено ніяких національних засобів, то остаточним рішенням у справі буде вважатися саме порушення права.

Проведення адвокатом справи в Європейському суді з прав людини.  Якщо скарга була визнана прийнятної, справа входить у вирішальну стадію, де заявникові і його адвокатові має бути переконати Суд у порушенні державою Європейської Конвенції.

Якщо палата вирішить провести усне слухання, сторони будуть сповіщені про цьому й буде призначена зручна (до певних меж) для всіх сторін дата. Сторонам також буде запропоновано представити свої остаточні доводи суду заздалегідь, щоб вони були переведені й надані суддям на слуханні. Також будуть зроблені необхідні дії для забезпечення перекладу (при твердженні Президента Палати).

Звичайно для виступу перед Судом кожній стороні надається 45 хвилин. У мові повинні бути порушені тільки ключові моменти справи, і висловлюватися вони повинні в простих і чітких фразах. За рідкісним винятком усні слухання публічні.

Як мінімум за 14 днів до дати слухання сторони можуть надати свою заяву Суду в писемній формі. Як мінімум за 10 днів до дати слухання, Суд повинен бути сповіщений про імена й посадові обов’язки осіб, що представляють із. Суд має право обмежити кількість представників або радників.

Першим надається право говорити державі й уже за ним — заявникові. Після вступних мов судді можуть задавати питання. Суд не очікує негайної відповіді. Після того як будуть задані всі питання, у слуханні буде зроблена перерва приблизно на 20 хвилин, протягом  якого сторони з підготувати відповіді Суду. На відповіді, а також на заключні доводи стороні приділяється близько 15 хвилин.

Рішення Суду повідомляється сторонам практично на наступну добу. На підготовку остаточної версії рішення йде біля місяця. Мотивоване рішення Палати по суті передається заявникові, державі, а також буде надруковано в періодичних виданнях Суду.

Протягом трьох місяців після дати винесення Палатою рішення заявником може бути подане прохання про те, щоб справа зі спрямована на розгляд Великої Палати. У випадку, якщо справа піднімає серйозне питання, що зачіпає тлумачення або застосування Конвенції або Протоколів до неї або серйозну проблему загального значення, Камера в складі п’яти членів Великої Палати може передати справу на розгляд у Більшу Палату, і тоді остання ухвалює рішення щодо істоти.

Рішення Європейського Суду з прав людини обов’язкові для виконання відповідачем. Рішення передається Комітету Міністрів, що контролює його виконання.

  1. Участь адвоката на стадії виконання рішення Європейського суду з прав людини.

Згідно ст. 46б § 1 Конвенції, держави «зобов’язуються виконувати остаточні  постанови Суду по справах, у яких вони є сторонами». Це зобов’язання  спричиняє   чіткі обов’язки держав-відповідачів. З одного боку, вони повинні  вжити заходів на користь заявників, щоб покласти кінець порушенню їхніх прав і, наскільки це можливо, згладити наслідки порушень (restіtutіo іn іntegrum), а з іншого боку — вжити заходів, необхідні для запобігання нових подібних порушень.

Таким чином, першим обов’язком є виплата компенсації (звичайно в грошовому  еквіваленті), що Суд може присудити заявникові по ст. 41 Конвенції і яка покриває відповідно майновий і моральний  збиток і/або витрати.

Виплата такої компенсації строго обов’язкове і порядок її чітко визначається в рішенні Суду.

Однак негативні наслідки порушення прав, які випробувала потерпіла  сторона, не завжди можуть бути адекватно виправлені виплатою компенсації. Залежно від  обставин виконання рішення може також зажадати від відповідача вживання індивідуальних заходів на користь заявника: наприклад, поновлення виробництва в справі, знищення інформації, неправомірно зібраної в порушення права на приватне життя, або скасування рішення про депортації, прийнятому незважаючи на те, що дана особа може піддатися жорстокому обігу в країні висилки. Також іноді необхідне прийняття таких загальних мір, як перегляд законодавства, підзаконних нормативних актів або судової практики, спрямованих на те, щоб запобігти новим подібним порушенням.

Існує безліч прикладів подібних обов’язків, покладених на держави Судом і Комітетом міністрів (наприклад, рішення Суду  в справі Скоццари й Джунта (Scozzarі and Gіunta) і Резолюції  DH(99)245 у справі  Соціалістична партія й інші проти Туреччини й  DH(99)434 про діяльності силових відомств у Туреччині, а також порядок застосування ст. 46 § 2 , прийнятий Комітетом міністрів).

Якщо порушення права продовжує робити несприятливий вплив, що не було виправлено справедливою компенсацією, присудженої заявникові, Комітет міністрів розглядає питання про необхідність прийняття національною владою заходів індивідуального характеру. Такі міри спрямовані на те, щоб покласти кінець триваючим порушенням і виправити, наскільки це можливо, їхнього наслідку (restіtutіo іn іntegrum).

Характер індивідуальних заходів залежить від природи порушення й ситуації заявника.

Поновлення провадження у внутрішньодержавному суді може бути ефективним  способом виправлення наслідків порушення Конвенції, викликаного несправедливим  розглядом у внутрішньодержавному суді (див., наприклад,  рішення й Резолюцію DH  (94) 84) у справі Barber, Messegu and Jabardo).

До інших заходів можуть відноситися: знищення поліцією всіх матеріалів (або принаймні   тих, які використалися для оперативних цілей), що містять інформацію, отриману в порушення права на приватне життя.

Міри загального характеру для запобігання подальших подібних порушень іноді важко визначити й здійснити. Органи державної влади повинні спочатку провести детальне дослідження причин порушення Конвенції. У деяких випадках обставини справи ясно показують, що порушення з’явилися результатом діючого внутрішньодержавного законодавства. Іноді до порушення прав людини приводить відсутність такого законодавства. У таких випадках з метою дотримання рішення Суду держава повинне змінити існуюче законодавство або прийняти новий, належний законодавчий акт.

Завданням адвоката на цій стаді є ефективна підготовка документів на основі рішення Європейського суду щодо його виконання.

  1. Особливості оплати юридичної допомоги адвоката-представника в Європейському суді з прав людини.

Згідно правила 94 Регламенту Суду гонорари сплачуються адвокатам або іншим особам, призначеним відповідно до пункту 4 правила 36. Коли це доцільно, гонорар може бути сплачений більш як одному такому представникові. Правова допомога може надаватися не лише для сплати гонорарів за представництво, а може також охоплювати дорожні витрати, прожитковий мінімум та інші необхідні витрати, яких зазнають заявник або призначений

Гонорар адвокатові, що представляв інтереси заявника в Європейському суді, а також витрати, які виниклі в нього, включаються до складу витрат, що відшкодовуються.

Разом з тим якщо Європейському суду заявлена вимога про компенсацію витрат і витрат на адвоката, він може його не розглядати під час відсутності деталізованого рахунку. У цьому рахунку доцільно вказати наступні аспекти: вид витрат (наприклад, транспортні, поштові й т.д.); підтверджуючі документи у відношенні їх; розрахунок витраченого часу із вказівкою кожного складеного документа (аж до вказівки конкретних статей Конвенції й Протоколів до неї, про порушення яких заявлено в скарзі); ставку його погодинної оплати як адвоката.

Так, в адвокатському рахунку, представленому Європейському суду, можна вказати час, витрачений на наступні аспекти: підготовку короткої (первісної) скарги в Суд; підготовку розгорнутої скарги; бесіди із заявником; опитування свідків і запис їхніх показань для Суду; підготовку документів у владні органи; досягнення дружнього врегулювання в справі (укладення мирової угоди); підготовку й переклад відповіді на поставлені їм питання; підготовку й переклад зауважень (меморандуму) на заперечення (меморандум) влади держави-відповідача; підготовку зауважень (меморандуму) по питанню прийнятності скарги; підготовку зауважень (меморандуму) по суті скарги; іншу переписку із Судом.

Крім того, Європейський суд оцінює необхідність витрат з погляду ефективного захисту прав заявника й виходить із того, що на саму сторону покладають витрати, пов’язані з її вимогами, клопотаннями й іншими змагальними документами, які були відхилені. Якщо скарга була визнана неприйнятної в якій-небудь частині, він відмовляє у відшкодуванні витрат у відповідній частині, що представляється йому розмірної.

Слід зазначити, що критерії необхідності й розумності витрат тісно зв’язані між собою, особливо при відшкодуванні витрат на декількох адвокатів. Європейський суд відшкодовує витрати більш ніж на одного адвоката за умови, що питання фактів і правові питання, підняті в справі, були надзвичайно складні й тому зажадали залучення декількох юристів для серйозних досліджень і їхнього аналізу

Європейський суд не відшкодовує витрати, пов’язані із змагальними документами або претензіями по скарзі, які їм відхилені. Якщо які-небудь фрагменти скарги були визнані неприйнятними або відхилені деякі змагальні документи, Суд у відповідній частині, що представляється йому справедливою, зменшує суму відшкодування витрат на адвокатів. При цьому він може взяти до уваги деталізований рахунок від адвоката, де зазначене час, витрачений адвокатом на документи, які були відхилені.

При оцінці складності розглянутої справи Європейський суд бере до уваги наступні аспекти: складність справи з погляду дослідження фактів і правових питань, що піднімаються; тривалість розгляду; велику кількість представлених документальних доказів; наявність декількох сторін у справі; значимість підмета захисту права; поширеність аналогічних справ у судовій практиці або повторюваність справи за участю сторони.

  1. Участь адвоката у Європейському суді з прав людини.

Фізична або юридична особа (далі — заявник) може подати в Європейський суд з прав людини (далі — Європейський Суд) скаргу про порушення прав, гарантованих Конвенцією про захист прав людини й основних свобод (далі — Конвенція), самостійно, без залучення юриста або адвоката. На етапі подачі скарги представляти інтереси на підставі доручення може будь-яка особа, що навіть не є юристом. Однак якщо скарга не була відхилена відразу як неприйнятна, а була комуніцирована владі держави-відповідача, то із цього моменту заявник не може представляти свої інтереси самостійно без залучення адвоката. За загальним правилом з моменту комунікації скарги інтереси заявника повинні бути представлені особою, що володіє статусом адвоката й проживаючої на території держави-відповідача або кожного з держав — учасників Конвенції (п. 2 ст. 36 Регламенту Суду). У зв’язку із цим послу комунікації скарги Європейський суд пропонує заявникові вказати адвоката, що буде представляти його інтереси при подальшому розгляді. Отже, участь адвоката у Європейському суді з прав людини є необов’язковою, але на практиці у більшості випадків адвокати обов’язково залучаються до участі у справі в Європейському Суді.

  1. Європейський суд з прав людини — головний орган міжнародного захисту прав людини

Європейський суд з прав людини є головним органом міжнародного захисту прав людини. Правовою основою діяльності Європейського суду з прав людини є Конвенція про захист прав людини та основних свобод і Регламент Європейського суду з прав людини. Відповідно до Конвенції Суд може розглядати, по-перше, заяви держав — учасниць Європейської конвенції з прав людини з питань порушення Конвенції в інших державах-членах (ст. 33). По-друге, заяви окремих осіб, груп і неурядових організацій про порушення прав людини, що мали місце в державах-членах (ст. 34). Порядок розгляду таких заяв різний.

У разі звернення до Суду із заявою держави — члена Ради Європи (таке звернення може бути зроблене з метою захисту прав людини в іншій державі-учасниці) Суд розглядає цю заяву, досліджує представлені факти, а в разі необхідності може провести розслідування. Держава, щодо якої було подано заяву, повинна створити всі необхідні умови для встановлення викладених у ній фактів.

Процедура розгляду Європейським судом з прав людини заяв окремих осіб, груп і неурядових організацій більш складна.

По-перше, мають бути дотримані умови, що стосуються суб’єкта подачі заяви. Якщо це особа, то мають бути дотримані вимоги, встановлені ст. 1 Конвенції. Особою вважається як індивід, так і юридична особа. Якщо це група осіб, то справа має стосуватися ідентичного порушення прав усіх осіб, що складають групу. Групою можуть бути визнані, наприклад, подружжя, члени організації. Якщо це неурядова організація, то для її звернення до Європейського суду в порядку ст. 34 Конвенції мають бути дотримані умови, що належать до критеріїв «асоціацій» (ст. 11 Європейської Конвенції про захист прав людини). По-друге, для звернення до Європейського суду з прав людини необхідно, щоб заявник вичерпав усі внутрішньодержавні способи правового захисту свого права. Практика Суду говорить про те, що в якості засобів правового захисту розглядаються лише засоби судового захисту, а також про те, що заявник повинен вичерпати не тільки всі доступні йому в державі засоби судового захисту, а й повноцінно використовувати існуюче законодавство. Заявник може звернутися до Європейського суду з прав людини, не використовуючи всіх внутрішньодержавних засобів правового захисту, в тому разі, коли вони безсумнівно неефективні. Звернення до органів несудового захисту (Уповноваженого із прав людини, прокурора, органів виконавчої влади, Президента держави тощо) не розглядаються Судом як звернення до правових засобів захисту. По-третє, заява може бути прийнята, якщо з дня ухвалення останнього рішення у цій справі внутрішньодержавними судовими органами минуло не більше шести місяців. По-четверте, Європейський суд не розглядає: анонімні заяви (ст. 35), але заявник може просити не вказувати його ім’я в разі офіційного опублікування результатів розгляду скарги; заяви, які одного разу вже були розглянуті Європейським судом з прав людини (ст. 35); заяви, що є предметом розгляду в іншому міжнародному органі із захисту прав людини (ст. 35); заяви, несумісні з положеннями Конвенції, явно необґрунтовані або такі, що містять ознаки зловживання правом звернення до Суду (п. 3 ст. 35).

Неприйнятними є заяви, за якими Європейський суд з прав людини некомпетентний приймати рішення, наприклад не пов’язані з порушенням прав, проголошених у Європейській конвенції з прав людини.

  1. Правове положення адвокатури у системі правових інститутів США. Структура організаційно-правові форми адвокатської діяльності у США та Канаді.

В США законодавче закріплення принципів організації та діяльності адвокатури фактично відсутнє як на федеральному рівні, так і у більшості штатів. Щоправда, Конституція США у шостій поправці зак­ріплює право обвинуваченого на захист у кримінальному про­цесі («В усіх випадках кримінального переслідування обвину­вачений має право… на допомогу адвоката для захисту»)’,,. проте не визначає механізм його реалізації. Федеральне зако­нодавство (Закон про кримінальне правосуддя 1964 р.) перед­бачає лише можливість призначення захисника у випадку відсутності у обвинуваченого матеріальних засобів, щоб най­няти його.

Щодо штатів, то вони практично декларують право обви­нуваченого на захист.

Важливе значення мають правила судочинства (в кримі­нальних та цивільних справах), що застосовуються Верховним судом США щодо федеральних судів, а також верховними . судами штатів для місцевих судів. Ними, зокрема, встановлю­ються обов’язкові вимоги, яким мають відповідати представ­ники будь-чиїх інтересів, щоб мати право виступати в конкре­тному суді. Ними фактично визначаються умови допуску до виконання функцій, наприклад, захисника в кримінальних справах в певному штаті чи у федеральних судах.

Структура адвокатур і форми діяльності адвокатів США. У кожному штаті є асоціація адвокатів штату. У більшості штатів встановлено обов’язкове членство в асоціації для всіх осіб, допущених до адвокатської практики; у таких штатах адвокатура, по американській термінології, «інтегрована»: асоціація адвокатів тотожна поняттю «адвокатури». Асоціації адвокатів як добровільні організації існують у меншостях штатів, а також у великих містах і графствах. Загальнонаціональною організацією адвокатів з добровільним членством є Американська асоціація юристів.

У завдання асоціацій адвокатів входить установлення норм професійної етики, сприяння адвокатам, вживання дисциплінарних заходів, розробка стандартів адвокатської діяльності, сприяння вдосконалюванню права й правосуддя й т.п.

У США адвокат може практикувати поодинці або в складі невеликих адвокатських контор або великих адвокатських фірм. До половини адвокатів практикує поодинці; разом з невеликими (2-3 чоловік) адвокатськими конторами одинаки становлять 2/3 від загального числа адвокатів.

Але по своїй важливості головною формою адвокатської діяльності є великі (50 чоловік і більше) адвокатські фірми. Такі фірми, як правило, не займаються кримінальними справами й воліють вести справи забезпечених клієнтів, переважно корпорацій. Адвокати, що працюють у фірмі, підрозділяються на дві категорії — партнери (співвласники фірми, що ділять дохід) і співробітники (адвокати, що одержують платню від фірми).

Частина адвокатів працює у відомствах «публічних захисників» — організаціях, що знаходяться на державному бюджеті штату й обслуговуючих на безкоштовній основі обвинувачуваних із числа незаможних громадян.

Існують і аналогічні служби юридичної допомоги незаможним, фінансовані з федерального бюджету. Ці служби з’явилися в середині 60-х років у рамках програм боротьби з бідністю. В 1974 році Конгрес заснував і фінансував Корпорацію по наданню юридичної допомоги — свого роду головну організацію для всіх цих служб.

Для юридичного обслуговування певних категорій населення (конкретних етнічних або професійних груп) у багатьох містах організовані товариства юридичної допомоги, що існують переважно на різного роду благодійні пожертвування й добровільні внески; органи влади також можуть виділяти кошти цим суспільствам.

В 1970-х роках з’явилася й така форма адвокатської діяльності, як адвокатські контори, «захищаючі інтереси суспільства» (pro bопо publico). Вони не ведуть справи окремих клієнтів, а виступають із позовами проти держави або корпорацій, пов’язаними з охороною прав і інтересів великих категорій громадян (споживачів, політичних активістів, виборців, платників податків) або з охорони природи й охорони здоров’я населення (наприклад, позови проти підприємств, що забруднюють навколишнє середовище, або проти будівництва атомних електростанцій).

Структура адвокатур і форми діяльності адвокатів Канади. «Бар енд бенч» (перегородка і лава) і адвокати та судді — це два стовпи правової системи Канади. Проте вони не є єдиними складови­ми судового процесу, але займають в ньому ключові позиції, маючи, зокрема, повноваження вирішувати справи, що стосу­ються інтересів найвищих державних інстанцій, а також спра­ви, що виникають між урядом і приватними особами, та між останніми.

Описати стисло систему Канади досить важко, оскільки Канада є федеративною державою, де законодавча, виконавча та юридична влади розподілені між федеральним (централь­ним) та десятьма урядами провінцій. Крім питань, окреслених Конституцією Канади, уряди провінцій наділені правом законотворчості у питаннях цивіль­ного права. Уряди ж провінцій наділені єдиним правом твори­ти закони щодо юридичної професії. Через це права юристів у провінціях є різними. Крім цього, Канада є дещо унікальною й у іншому, маючи дві системи закону про приватну власність: систему цивільного права у Квебеку та систему прецедентного права у іншій частині Канади. Подвійна система законів походить з історії Канади та ряду законодавчих актів, що призвели до створення канадської конфедерації. Та частина Канади, що була колонією Англії, сприйняла юридичну систему, заснова­ну на прецедентному праві. Територія ж Канади, яку тепер займає провінція Квебек, була у свій час колонізована Фран­цією, яка встановила тут свою систему управління і традицій, в тому числі й цивільного права. Після того, як Квебек було приєднано до англійської корони (у 1750 р.), британський уряд, щоб привернути на свій бік населення завойованої тери­торії, надав французьким колоністам ряд пільг, серед яких — можливість залишити свою систему цивільного законодавства» Це право легалізувалося серією конституційних актів, які були включені до Акта Британської Північної Америки у 1867 р. — основи законодавства Канадської конфедерації. Ось чому в Канаді зараз дві системи законів про приватну власність.

Не зважаючи на те, що кожна провінція на свій розсуд регулює діяльність юристів, на практиці між законодавчими актами існує певна схожість. Кожна провінція має законодав­ство, на підставі якого створюються спілки адвокатів чи юридичні товариства, які вирішують усі питання, пов’язані з їх професією. Кожне товариство має свій керуючий орган, свого «бенчера» (керівника), котрий встановлює правила про­фесійної поведінки юристів. Обов’язками спілок юристів є: вирішення питань, які мають відношення до юридичної професії у провінції; вироб­лення стандартів допуску до юридичної практики; нагляд за додержанням кодексу та правил доброчинної поведінки юри­стів; стягнення з тих, хто їх порушує; страхування клієнтів від помилкових або некваліфікованих дій юристів; нагромаджен­ня коштів для правової допомоги особам з низьким рівнем доходів або тих, які не мають їх зовсім.

  1. Американська Асоціація адвокатів, її структура та повноваження.

В усіх штатах, містах та великих на­селених пунктах адвокати, що допу­щені до ведення справ в судах, подають за плату правову допомогу, створюють на добровільних засадах свої професійні об’єднання, асоціації. В одних штатах вони діють на основі статутів («конституцій»), які затверджуються членами асоціа­цій на загальних зборах, в інших — на основі спеціальних  законів штату чи судових правил. В штатах другої групи (їх двадцять вісім) з «інтегрованою адвокатурою» членство в асо­ціації є обов’язковою умовою зайняття адвокатською практи­кою.

В організаційному відношенні асоціації штатів та міст за своєю структурою та призначенням мають багато спільного. Їх члени на своїх щорічних зібраннях (у великих штатах через делегатів або шляхом голосування по пошті) обирають прези­дента асоціації та виконавчий орган (5—10 членів), відомий як «виконавчий комітет», «рада керуючих» або «рада уповнова­жених». При виконавчому органі, як правило, створюється декілька комітетів або комісій (з галузей права, прийняття нових членів, дисциплінарні та ін.). В окремих штатах молоді адвокати (з стажем роботи менше 5 років) створюють свої організації, завданням яких є підвищення кваліфікації, поліп­шення умов праці їх членів та ін.

Адвокатські асоціації штатів утворюють національну ор­ганізацію адвокатів — Американську асоціацію адвокатів, яка заснована у 1878 p. Її мета — обмін досвідом, обговорення проблем юридичної освіти, реформи законодавства та ін. Хоча ця організація не є асоціацією усіх американських адвокатів, вона об’єднує у тій або іншій формі майже всі великі органі­зації практикуючих адвокатів та нараховує у своєму складі понад 130 тис. членів (всього в США допущено до ведення справ в судах понад 300 тис. адвокатів). Асоціації адвокатів в штатах безпосередньо їй не підпорядковані, проте, як правило, додержують її рекомендацій, особливо норм професійної ети­ки.

Вищим органом управління в Американській асоціації адвокатів є палата делегатів, яка складається з 250 членів. Делегати обираються від округів, в яких проживає не більше двох тисяч виборців — членів асоціації. Крім того, до палати входять по одному «делегата штату», які обираються по пошті на три роки членами асоціації, що проживають в даному штаті. До складу членів палати включаються також представники інших загальнонаціональних організацій юристів (Асоціації американських юридичних учбових закладів, Ради голів верхо­вних судів та ін.) та особи, які займають високі юридичні посади в уряді, зокрема, генерал-аторней (міністр юстиції). Між сесіями палати делегатів функції управління здійснює рада керуючих (21 член), яку очолює президент асоціації. Її діяльність здійснюється головним чином через секції, постійні та тимчасові (спеціальні) комітети та комісії (разом понад сімдесят). Палата делегатів та рада керуючих мають свої комі­тети (понад двадцять). Багато комітетів мають на місцях своїх постійних представників та консультативні органи.

Адвокати, що практикують, можуть бути членами й інших великих організацій юристів — Американського товариства правосуддя. Американської асоціації патентного права, Асоці­ації морського права та ін.

  1. Умови набуття адвокатської професії; допуск до адвокатської практики у США. Канаді, охарактеризувати відмінності.

Умови набуття адвокатської професії США. За станом на 1998 рік, у США налічувалося приблизно 220 вищих (приватних або державних) юридичних навчальних закладів — юридичних шкіл або коледжів, що входять до складу університетів (по суті, юридичних факультетів), або самостійно функціонуючих юридичних шкіл або коледжів. Із загального числа юридичних вузів 181 є офіційно визнаними. Організацією, що вирішує питання про офіційне визнання юридичного вузу, виступає (за уповноваженням міністерства освіти США) Американська асоціація юристів (ААЮ). Вона затверджує перелік вимог, яким повинен відповідати юридичний вуз. Серед цих вимог — наявність певного контингенту кваліфікованих викладачів, придатні й зручні для занять приміщення, певний мінімальний бюджет, кількісний і тематичний мінімум навчальних, довідкових і нормативних видань у бібліотеці, наявність певних навчальних програм і ін.

Строк навчання, як правило, три роки. Обов’язкові навчальні дисципліни існують тільки на першому курсі; до них звичайно відносяться договірне, майнове, деліктне, кримінальне й адміністративне право, професія юриста, філософія права та ін. На другому й третьому році навчання обов’язкових предметів немає, однак встановлюється певне мінімальне їхнє число, що слід вивчити студентові. Встановлюється також обов’язкова мінімальна кількість навчальних годин на тиждень.

Метод навчання в юридичних вузах заснований переважно на аналізі прецедентів і рішень апеляційних судів; законодавчий і адміністративно-правовий матеріал залучається остільки, оскільки він допомагає аналізу прецеденту.

Умови набуття адвокатської професії Канади. Більшість канадських юридичних учбових закладів вимагають від абітурієнтів наявності у них вищої освіти, перш ніж вони розпочнуть навчання на факуль­теті права. Ряд студентів-першокурсників вже мають спеціаль­ність вчителя, інженера, фізика тощо, а деякі з них — достат­ній професійний і діловий досвід. Внаслідок цього студентське товариство на факультетах права є досить строкатим. Зросла й кількість жінок та студентів з расових та етнічних меншин, які вивчають право. Сьогодні вимоги до тих, хто прагне здобути юридичну освіту, стали більш конкретними і різноманітними. Звичайна програма юридичного закладу складається з курсу стаціонарного навчання, розрахованого на три роки. Не існує вечірніх або факультативних видів навчання. Освіта, яку дає заклад, потребує від студента критичного мислення, вивчення систе­ми закону, вміння аналізувати ситуації та знаходити відповідні рішення. Багато уваги приділяється практичному навчанню, яке триває й у наступні роки після закінчення закладу.

Допуск в адвокатури США. Випускник юридичного вищого навчального закладу разом з дипломом і ступенем не отримує автоматичного права займатися адвокатською практикою. Для того щоб стати адвокатом, одержати патент на адвокатську практику, необхідно здати іспит, пройти свого роду додаткову атестацію. Причому патент видається на право займатися адвокатською практикою не взагалі, повсюдно в США, а тільки на території того штату, де збирається практикувати даний кандидат в адвокати.

Принцип і умови допуску до адвокатської практики встановлюються звичайно верховним судом штату, однак питання про самий допуск вирішується спеціальною комісією з допуску в адвокатури, сформованою асоціацією адвокатів штату. Комісія вирішує це питання на основі вивчення моральних якостей кандидата й результатів іспиту, що влаштовуються нею.

Для ведення справ у федеральних судах необхідний дозвіл, що видається автоматично особам, допущеним до адвокатської практики в штаті.

Допуск в адвокатури Канади. Юридичні товариства встановлюють у кожній провінції критерії, яким має відповідати особа, що бажає стати адвока­том, для одержання дозволу займатися юридичною практи­кою. Юридична освіта, здобута у юридичному учбовому зак­ладі, складає лише мінімум вимог, встановлених всіма юриди­чними товариствами. Особи, що успішно завершили навчання, мають пройти період стажування у практикуючого юриста; строки цього періоду передбачені правилами допуску до практики тієї про­вінції, де студент бажає займатися практикою. Період стажу­вання включає й перебування на курсах, організованих юри­дичним товариством. Юрист має проходити практику в провінції, де він є членом адвокатури. Ті, що бажають працювати у іншій про­вінції, але мають менше п’яти років практичного стажу, за звичайних вимог зобов’язані повторити стажування у цій про­вінції. Юристи, що мають значний досвід, повинні скласти письмові іспити на підтвердження своєї ознайомленості із специфікою законодавства провінції, де вони працюватимуть, та сплатити відповідну суму за вступ до , спілки адвокатів юридичного товариства.

  1. Правовий статус адвоката у США та Канаді. Порівняльно-правовий аналіз з країнами Європи.

У США немає нормативно-правового акту, що закріплював би правовий статус адвоката в судочинстві. Статус захисника випливає зі звичаю, прецедентного права, норм професійної етики та статутів асоціацій адвокатів, до яких належить захисник. Основна умова участі адвоката в судочинстві викладено у виправленні VІ (1791 р.) до Конституції США: При всякому кримінальному переслідуванні обвинувачуваний має право на швидкий і публічний суд безсторонніх присяжних того штату й округу, раніше встановленого законом, де був скоєний злочин; обвинувачуваний має право бути обізнаним про сутність і підстави обвинувачення, він має право на очну ставку зі свідками, що показують проти нього, право на примусовий виклик свідків зі своєї сторони для свого захисту». У свою чергу, це виправлення знайшло свій розвиток у ряді рішень судів (1963 р., справа «Гидеон проти Уэйнтрайта»; 1966 р., справа «Миранда проти штату Аризона»; 1977 р., справа «Брюер проти Вільямса» і т.д.). Суть цих рішень зводиться до наступних принципів: право на адвоката є фундаментальним і необхідним для справедливого суду, при цьому право на адвоката має кожний, кому може бути призначене покарання у вигляді позбавлення волі; адвокат повинен брати участь у тих процесуальних діях, де він може зробити обвинувачуваному допомогу, щоб упоратися із правовими проблемами, або дати пораду, коли йому протистоїть інша процесуальна сторона».

 В Канаді також не існує єдиного нормативно-правового акту, який би визначав правовий статус адвоката. Оскільки Канада є федеративною державою, де законодавча, виконавча та юридична влади розподілені між федеральним (централь­ним) та десятьма урядами провінцій. Через це права юристів у провінціях є різними. Кожна провінція Канади на свій розсуд регулює правовий статус адвоката, на практиці між законодавчими актами існує певна схожість. Кожна провінція має законодав­ство, на підставі якого створюються спілки адвокатів чи юридичні товариства, які вирішують усі питання, пов’язані з їх професією. Кожне товариство має свій керуючий орган, свого «бенчера» (керівника), котрий встановлює правила про­фесійної поведінки юристів. Обов’язками спілок юристів є: вирішення питань, які мають відношення до юридичної професії у провінції; вироб­лення стандартів допуску до юридичної практики; нагляд за додержанням кодексу та правил доброчинної поведінки юри­стів; стягнення з тих, хто їх порушує; страхування клієнтів від помилкових або некваліфікованих дій юристів; нагромаджен­ня коштів для правової допомоги особам з низьким рівнем доходів або тих, які не мають їх зовсім.

На відміну від США та Канади правовий статус адвоката практично у всіх державах Євросоюзу (крім Великобританії) визначений конкретно у нормативно-правовий актах. У Німеччині діє Федеральний закон про адвокатуру регулює в окремому розділі права і обов’язки адвоката. Так, згідно з § 43 цього закону, «адвокат зобов’язаний сумлінно виконува­ти свої професійні обов’язки, бути гідним поваги і довіри, яких вимагає становище адвоката». У Франції Ос­новний закон про статус адвокатів № 71-1130 від 31 грудня 1990 p. В Великобританії відсутній будь-який єдиний закон або нормативний акт про правовий статус адвоката. Як й інші установи та інститути англійського права, адвокатура у її нинішньому вигляді формувалась протягом століть і зараз функціонує на досить розпливчатому ґрунті королівських хартій, окремих парламентських актів, урядових постанов, правил, прийнятих керівними органами адвокатських організацій, судових рі­шень, традицій та звичаїв.

  1. Роль адвоката-захисника у системі кримінального судочинства США.

Право обвинуваченого на захист і на захисника було закрі­плено в 1791 р., у VI Поправці до Конституції США 1787 р. В усіх випадках кримінального переслідування «обвинувачений має право… вимагати, щоб його ознайомили з мотивами обви­нувачення і надали очну ставку зі свідками, які показують проти нього; обвинувачений може також вимагати примусового вик­лику своїх свідків і користуватися допомогою адвоката для захисту»‘.

Протягом тривалого часу дія VI Поправки не поширюва­лась на досудові стадії. З середини XX століття з метою усунення порушень прав обвинуваченого Верховний суд США сформулював ряд правил, пов’язаних з участю адвоката в досудових стадіях процесу.

Оскільки на обвинувача у кримінальному процесі США не покладається обов’язок всебічного, повного і об’єктивного дослідження обставин справи, захисникам доводиться вдава­тися до власного розслідування з метою одержання доказів, що виправдовують обвинуваченого або пом’якшують його ві­дповідальність.

За загальним правилом, адвокат вступає в процес на боці обвинуваченого на його запрошення.

Обвинуваченому, який не має коштів, щоб запросити адвоката, останній призначається, але лише у справах про найбільш тяжкі злочини.

Прийнятий у 1964 р. федеральний закон про кримінальну юстицію закріпив надання юридичної допомоги кожному обвинуваченому, який її потребує, і визначив умови забезпечен­ня безплатного захисту для матеріально неспроможних. Адво­катські колегії було зобов’язано подати до відповідних судів список адвокатів, які призначалися здійснювати захист зазна­ченої категорії обвинувачених. Особа, що звертається з таким клопотанням, має під присягою підтвердити відсутність у неї необхідних для захисту коштів. Якщо перевіркою буде встано­влена неправдивість цих даних, заявник підлягає судовій від­повідальності.

Закон 1964 р. позитивно вплинув на вирішення проблеми організації захисту вказаної категорії обвинувачених в спра­вах, що підлягають юрисдикції штатів. Суд штату може приз­начити захисника неспроможному обвинуваченому у справі про фелонію (про тяжкий злочин), а в окремих випадках — і про місдимінор».

Право порушувати кримінальне пе­реслідування, збирати докази і підг­римувати обвинувачення в суді фор­мально має кожний потерпілий від злочину, як фізична так і юридична особа. Зрозуміло, що таку можливість має тільки той, у кого є необхідні кошти, щоб найняти адвоката або агента приватного розшукового бюро. Тому в переважній більшості випадків кримінальне переслідування порушується і провади­ться поліцією. Федеральним бюро розслідувань, органами аторнейської служби, рідко коронером, а в окремих випадках й, іншими органами. Основним органом слідства є поліція, яка поділяється на поліцію федеральну, штатів і місцеву.

Визнанню особою своєї вини у вчиненому злочині в кри­мінальному процесі США надається вирішального значення, і воно здебільшого має такі ж правові наслідки, як і в Англії — його достатньо для засудження обвинуваченого. Тому поліція та інші правоохоронні органи прагнуть одержати зізнання вже при провадженні попереднього слідства, не завжди додержуючи за­кону. Через це в ряді рішень судових органів була визнана доцільною участь захисника з моменту пред’явлення обвинувачення. Згідно з практикою, що склалася, особі, заарештованій поліцією, має бути надана можливість зателефонувати адвокату або родичам чи знайомим (така можливість надається, як прави­ло, не більше двох разів) з тим, щоб забезпечити затриманому допомогу захисника. Обвинуваченому, який перебуває під вар­тою у поліції, може бути дозволено зустрітися з своїм адвокатом або проконсультуватися з ним по телефону.

Кримінально-процесуальне право США не містить вимо­ги обов’язкового ознайомлення захисника обвинуваченого з матеріалами попереднього слідства. Після закінчення розслі­дування поліцією, ФБР чи аторнеєм справа передається до суду.

Офіційно вважається, що процесуальною стадією розслі­дування кримінальної справи є її попередній розгляд в суді, розслідування ж поліцією і ФБР злочинів є непроцесуальною діяльністю. Зібрані ними дані не мають доказової сили, поки не пройдуть стадію процесуального закріплення в суді як матеріали для підтримання обвинувачення.

Тому першою стадією кримінального процесу в США вважається доставленя до мирового судді особи, заарештова­ної за підозрою у вчиненні злочину, для вирішення питання про обґрунтованість кримінального переслідування та про за­побіжний захід.

У разі відмови обвинуваченого від попереднього розгляду справи суддя пред’являє йому обвинувачення, вирішує питан­ня про запобіжний захід (ним може бути застава або взяття під варту). Крім того, він може (а при наявності відповідного клопотання аторнею зобов’язаний) допитати свідків обвинува­чення і зафіксувати їх показання у протоколах.

Якщо обвинувачений не відмовляється від попереднього розгляду справи, суддя може надати йому певний строк для запрошення захисника і для виклику свідків захисту.

Попередній розгляд провадиться в судовому засіданні. Тут адвокат допитує свідків з боку обвинувачення, обвинува­ченого, свідків з боку захисту. Показання свідків, пояснення і показання обвинуваченого заносяться до протоколу або стено­графуються. Вже на цьому етапі провадження мета захисту може бути досягнута, бо якщо суддя, заслухавши пояснення і показання, дійде висновку про недостатність підстав для при при­тягнення до кримінальної відповідальності, то він виносить рішення про зняття обвинувачення, тобто про закриття спра­ви. Якщо ж таких підстав виявилося достатньо, суддя притягує обвинуваченого до кримінальної відповідальності і, якщо справа йому підсудна, вирішує справу і ухвалює вирок (в тому числі й тоді, коли обвинувачений відмовився від свого права на розгляд справи судом присяжних).

У разі невизнання підсудним своєї вини суддя надає йому час для підготовки до захисту (не менше двох днів). Протягом нього підсудний має можливість підготуватися разом з адвокатом до захисту, виро­бити з ним тактику захисту. Адвокат може порадити „підсудно­му відмовитись від суду присяжних, якщо йому не загрожує смертна кара чи інше суворе покарання. В цьому разі справу розглядатиме колегія професійних суддів. Частіше це є доціль­ним, коли підсудний насправді є невинним, оскільки вірогід­ність помилки в цьому разі є меншою, ніж при розгляді судом присяжних. У більшості штатів з участю присяжних розгляда­ється лише близько 10 відсотків кримінальних справ.

Захисник бере участь у формуванні остаточного складу журі присяжних, використовуючи своє право на відвід тих осіб з їх числа, які, на його думку, не можуть бути неупередженими при оцінці доказів і вирішенні питання про винність чи не­винність підсудного.

Після сформування журі розпочинається дослідження до­казів і промови сторін. Виступаючи після обвинувача з вступ­ною промовою, адвокат «відкриває захист», стисло ознайом­люючи суд з тим, що він має намір доказувати, далі бере участь у допиті свідків обвинувачення і захисту, а потім як свідка й підзахисного, якщо вважає за доцільне, щоб -і підсудний дав перед присяжними відповідні пояснення з приводу інкриміно­ваних йому дій.

У разі винесення присяжними вердикту про винність під­судного адвокат має право до призначення суддею покарання (його міра може бути визначена через кілька днів або навіть тижнів) подати судді докази на користь пом’якшення відпові­дальності підсудного.

Обвинувачений може запросити адвоката і для участі в розгляді справи в порядку сумарного провадження. У такому, спрощеному, порядку розглядаються кримінальні справи ни­жчими судами штатів і, так званими, комісарами. Вони можуть призначити покарання у вигляді короткочасного арешту або порівняно невеликого штрафу.

Процес у судах сумарної юрисдикції дуже спрощений і швидкий. Докази досліджуються лише тоді, коли обвинуваче­ний не визнає себе винним, як свідки і обвинувачі часто виступають поліцейські, які затримували і допитували особу в поліції. Як і при розгляді справи в суді присяжних, тут для адвоката основним завданням є доведення хибності доказів, наявності сумнівів у вині підзахисного.

Від імені й в інтересах засудженого адвокат-захисник може подати апе­ляцію як з питань факту, тобто оска­ржити вердикт присяжних і резолю­цію судді, якими підсудного визнано винним у вчиненні зло­чину, так і з питань права, тобто оскаржити вирок суду у зв’язку з неправильним застосуванням кримінального закону або порушенням під час судового розгляду процесуальних норм, які обмежили права підсудного і призвели до неправи­льного рішення.

На терені штатів апеляції подаються: до судів першої інстанції (судів загальної юрисдикції) — на рішення нижчих судів (судів обмеженої юрисдикції), які є судами сумарної юрисдикції; до проміжних апеляційних судів — на рішення судів першої інстанції і до вищих судів штатів — на рішення будь-якого нижчестоящого суду.

За загальним правилом, рішення вищих судів є остаточ­ними. Але адвокат може скористатися винятком з цього пра­вила, коли єдиною підставою для апеляційного оскарження до Верховного суду США є наявність у справі «федерального питання», тобто такого важливого для вирішення криміналь­ної справи питання, на яке можна відповісти тільки шляхом тлумачення і застосування положень Конституції США, феде­ральних законів і договорів, зокрема, при порушенні гарантій прав особи, закріплених у Конституції.

Апеляційними інстанціями в системі федеральних судів є: окружні суди — щодо комісарів США, апеляційні суди — щодо окружних федеральних судів, Верховний суд США — щодо федеральних окружних і апеляційних судів, а також щодо вищих судів штатів.

Для того щоб Верховний суд США розглянув справу, адвокат має переконати його не стільки в тому, що нижчестояща інстанція винесла помилкове рішення, скільки в тому, що в цій справі наявним є федеральне питання.

Встановивши незаконність або необґрунтованість пере­бування засудженого в ув’язненні, і виходячи з характеру допущеної помилки, суддя видає наказ про звільнення з-під варти, котрий підлягає негайному виконанню, або повертає справу до суду штату для нового розгляду. На будь-яке з цих рішень судді може бути подана апеляція до вищестоящого суду. Адвокат може також допомогти засудженому у складанні й поданні петиції про перегляд рішення суду за нововиявленими обставинами, з мотивів, що вони є настільки важливи­ми, що коли б вони були відомі суду при розгляді справи, він не постановив би обвинувального вироку.

  1. Правове положення іноземних адвокатів у США.

Іноземні адвокати допускаються до практики у США за допомогою наступної процедури.

Суд за своїм розсудом може без іспитів видати ліцензію на практику як  юридичний консультант у штаті іноземцеві, що досяг віку старше 26 років, який відповідає наступним ознакам: ця особа в іноземній державі протягом  не менш п’яти років до звернення була адвокатом; вона має необхідні моральні якості та задовольняє загальним вимогам придатності для членства в асоціації юристів штату; вона має намір практикувати в даному штаті як  юридичний консультант, а також мати для цієї мети офіс у даному штаті.

На підтвердження зазначених вище обставин претендентові необхідно надати секретареві суду наступні документи: виданий професійною організацією або державною установою в закордонній державі, що є вищою інстанцією в питаннях професійної дисципліни, сертифікат що засвідчує, що заявник допущений до практики із вказівкою дати допуску, а також засвідчує дійсність його статусу адвоката; рекомендаційний лист від одного зі членів виконавчого органа такої професійної організації або державної установи або від одного із суддів суду вищої інстанції або суду першої інстанції такої іноземної держави; належним чином засвідчений переклад такого сертифіката та такого листа на англійську мову; інші документи, що підтверджують освіту та професійну кваліфікацію заявника, його моральні якості і загальну придатність; свідоцтво про народження.

З урахуванням деяких обмежень особа, що має ліцензію на практику як  юридичний консультант, уважається адвокатом, що складається при асоціації юристів даного штату, і має всі права, що випливають звідси, у тому числі права і обов’язку члена асоціації юристів даного штату відносно роботи в одній і тій же юридичній фірмі з одним або більше членами асоціації юристів даного штату, у тому числі за допомогою будь-якого партнерства, і що, мабуть, найбільше важливо, конфіденційності відносин с клієнтом і відповідною документацією й подібними привілеями.

Особа, що одержала ліцензію на практику як  юридичний консультант, відповідно до зазначених положень зобов’язана додержуватись професійної дисципліні в такий же спосіб і в тому ж обсязі, що й члени асоціації юристів даного штату; у тому числі вона зобов’язана надати в суд: зобов’язання або належне підтвердження наявності страховки професійної відповідальності на таку суму, яку встановлює суд.

Заявник, що бажає одержати ліцензію на практику як  юридичний консультант, зобов’язаний сплатити мито за видачу ліцензії.

У випадку, якщо особа, що одержала ліцензію на практику як  юридичний консультант, відповідно до зазначених положень, згодом буде прийняте у члени асоціації юристів даного штату, ліцензія юридичного консультанта, видана такій особі, буде вважатися скасованою патентом, виданим такій особі на право здійснювати юридичну практику як член асоціації юристів даного штату.

  1. Особливості організації та діяльності адвокатури у Великобританії.

Загальне керівництво адвокатурою Великобританії (як соліситорами, так і баристерами) здійснюється лордом-канцлером (членом Вер­ховного суду).

Усі соліситори об’єднані у Юридичне товариство, створене відповідно до Королівської хартії 1845 p., яка пізніше зазнала відповідних змін та доповнень. Членство у товаристві є добровільним — понад 85% з нині практикуючих соліситорів є його членами. Воно управляється Радою з 65 членів, які обираються щороку, з них 30 чоловік репрезентують Лондон, а 35 — провінцію. Очолює Раду президент та віце-президент, які обираються із складу Ради. Їм допомагає секретаріат товариства — секретар та шість його заступників, посади яких позначаються літерами (А, В, С та ін.) і пов’язані з виконанням визначених наперед постійних обов’язків. Причому секретар та його заступники не є соліситорами та виконують свої функції як звичайні службовці (разом їх у юридичному товаристві понад 1,5 тис.). Рада товариства збирається на свої засідання в середньому раз на три тижні, поточними ж справами займається секретаріат, котрий, .зокрема, організовує на регулярній основі роботу кількох десятків комісій та підкомісій Юридичного товариства (з галузей права, з проблем професійної діяльності та ін.).

Для організації та діяльності Юридичного товариства ха­рактерною є атмосфера кастової замкненості, приватного зак­ритого клубу. Соліситори збираються на свої щорічні збори та конференції разом з подружжям, самі ж конференції за тради­цією поєднуються з обідами та банкетами. Юридичне товари­ство має свої (закриті для сторонніх) спортивні клуби, бари, бібліотеки та читальні, свою службу телетайпного зв’язку і, навіть, бюро подорожей.

Не зважаючи на добровільність об’єднання, Юридичне товариство має встановлені консолідованим законом 1974 р. про соліситорів повноваження щодо своїх членів. Так, товари­ство за згодою лорда—головного судді та лорда—хранителя судових архівів вправі видавати інструкції з професійної під­готовки щодо освіти та навчання осіб, які бажають вступити в товариство чи працювати як соліситори (ст. 2). Рада товарис­тва: складає правила про поводження соліситорів з грошовими сумами клієнтів, правила про щорічне складання ними фінансового звіту; регулює на підставі відповідних правил профе­сійну практику, поведінку та дисципліну соліситорів. Поряд з цим Юридичне товариство турбується про добробут солісито­рів та прагне підтримувати добрі ділові стосунки з іншими організаціями та громадськістю. Рада влаштовує читання лек­цій, що стосуються нового в праві, організовує навчальні курси та видає брошури з ряду питань, які мають важливе значення для всіх соліситорів, зокрема про нове законодавст­во. Рада також виявляє осіб, що виконують функції соліситора без відповідних повноважень, і в ряді випадків починає проти них судові процеси. Питаннями, що перебувають поза сферою діяльності Юридичного товариства, є такса оплати послуг соліситорів, котра встановлюється законом, а також дисциплінарне прова­дження, яким відає дисциплінарний трибунал в справах соліситорів, що є незалежним органом.

Соліситори виконують свої функції тільки за сертифіка­тами Юридичного товариства, які засвідчують, що особа від­повідає вимогам інструкції з професійної підготовки, а також вимогам щодо особистих якостей та здатності особи бути соліситором Верховного суду. Сертифікат підтверджується щороку. Вони не потрібні для тих, хто не практикує приват­ним порядком, а обслуговує, зокрема, державні органи.

Організація баристерів є досить архаїчною. Вони об’єднані у чотири «судових Інни»: Інн Грея, Інн Темпля, Інн Лінкольна та Інн Мідл-Темпл. Ці Інни виникли в XIV ст., знаходяться у Лондоні і, мабуть, походять від назви жилих кварталів, облаштованих особами, які займаються судовою практикою. Вони і до цього часу контролюються суддями. Судові Інни є не тільки об’єд­наннями адвокатів, а й крупними землевласниками: володію­чи великими земельними угіддями, баристери отримують чи­малу ренту.

Судові Інни складаються з трьох груп членів: бенчери, які є суддями або старійшинами корпорації баристерів та керують Їннами; баристери, які приймаються в корпорації бенчерами; учні, котрі ними же допускаються у корпорацію. До отримання практики баристер має пройти учнівський стаж строком не менше 12 місяців під керівництвом старшого члена корпорації. Умовами прийняття учня у її члени є: вік — не молодше 21 року; сплата вступного внеску; складання остаточ­ного екзамену за фахом. Цікаво, що протягом року (в осінню, зимову, весняну та літню сесії) влаштовуються обіди в трапезній судового Інна, у яких беруть участь усі його члени. В ній за окремим столом сидять судді без мантій та париків, а за другим — студенти і учні. Ця своєрідна форма спілкування дає можли­вість майбутнім адвокатам увійти в роль обраної ними професії, і взагалі просто «поваритися у казані» судового Інна.

Всі баристери поділяються на дві групи (чи ранги): «коро­лівські радники» та «молодші» баристери. Королівських рад­ників призначає королева за рекомендацією лорда-канцлера. Вони отримують право перебувати в суді у шовковій мантії (а не у вовняній, як інші). Їх головна роль полягає у веденні складних справ, наданні спеціальних порад. Слід також під­креслити, що більшість членів Вищого суду призначається з числа королівських радників.

Органом управління судових Іннів та корпорації баристе­рів є сенат, що складається з 90 членів. В нього входять: генеральний аторней, генеральний соліситор та представник правової освіти, двадцять чотири представники університетсь­ких коледжів (по три від кожного Інна), тридцять дев’ять представників баристерів, обраних корпорацією (з них шість не повинні бути практикуючими баристерами) і не більше 12 додаткових членів. Судові Інни пов’язані рішеннями сенату, в компетенцію якого входить також консультування Іннів. У деяких випадках сенат має право приймати рішення від імені всієї корпорації баристерів, хоча їх рада зберігає свою автоно­мію у питаннях спеціальних повноважень та функцій. Напри­клад, у функції сенату входить дача вказівок, що стосуються прийняття учнів в члени корпорації баристерів, регулювання питань професійної підготовки, розслідування скарг на бари­стерів, управління фінансами корпорації та ін.

Органом управління поточними справами є Рада, утворе­на за ініціативою баристерів в 1894 p. Рада обирається їх загальними зборами. Вона складається з 60 членів та обирає з свого середовища виконавчу раду з трьох чоловік. При Раді створюються об’єднані комісії Іннів з галузей права, питань юридичної допомоги, оплати праці, етикету та професійної поведінки, міжнародних зв’язків тощо. Головне призначення Ради полягає в підтриманні престижу, гідності та незалежності корпорації.

  1. Умови набуття адвокатської професії у Великобританії.

В Англії порядок прийняття до адвокатури є різним для соліситорів і баристерів. Так, кандидати у соліситори, якщо вони не закінчили університет або юридичний коледж, прохо­дять в юридичному товаристві (професійне об’єднання соліси­торів) п’ятирічне стажування, по закінченні якого складають Відповідні екзамени. Строк стажування для осіб, що мають вищу юридичну освіту — три роки. Крім того, кандидат у соліситори повинен оплатити свою підготовку.

У «судові Інни» (об’єднання баристерів) приймаються, як правило, особи, що закінчили Оксфордський або Кембридж­ський університети. До отримання практики баристер має здобути учнівський стаж строком не менше 12 місяців під керівництвом старшого члена корпорації. Умовами прийняття учня у її члени є: вік не молодше 21 року; сплата вступного внеску; заняття протягом восьми семестрів та складання екза­мену за фахом. Протягом року (у зимову, весняну, літню і осінню сесії) влаштовуються обіди в трапезній судового Інна, де за окремим столом сидять судді без мантій та париків, а за другим — студенти і учні. Ця своєрідна форма спілкування дає можливість майбутнім адвокатам придивитися, увійти в роль обраної ними професії, і взагалі просто «поваритися» у казані» «судового Інна». По закінченню стажування кандидат у бари­стери складає екзамен та обирає судовий Їнн, в якому йому доведеться працювати.

  1. Розподіл функцій між адвокатами, правовий статус соліситорів та баристерів.

Відмінною рисою англійської адво­катури є історично сформований по­діл практикуючих правозахисників на дві групи — соліситорів та баристерів. Роз’єднання професії правозаступників розпо­чалося з 1340 p., коли виникла професійна адвокатура. Адво­катами, які мали право виступати в судах загального права, були доктори права та баристери, у часи, коли уся підготовча робота здійснювалася судовими чиновниками, які називалися аторнеями. В XV ст. вперше з’являються соліситори, котрі практикували у суді канцлера. Спочатку вони були рангом нижче за аторнеїв і не були так пов’язані зі своїми клієнтами, як перші. До XVIII ст. обов’язки аторнеїв та соліситорів часто виконувались одними й тими ж особами, у зв’язку з чим різниця між ними була повністю усунута законом 1873 р. про судоустрій, згідно з яким аторнеї стали призначатися соліситорами.

Баристери, які в XIV ст. об’єдналися у корпорації (так звані судові Інни), після XVI ст. не приймали у своє товарис­тво аторнеїв та соліситорів. У результаті останні об’єднались і в 1793 p. утворили власну професійну організацію в Лондоні — «Товариство джентельменів, котрі практикують в судах загального права та справедливості», яка передувала теперіш­ньому Юридичному товариству.

Отже, розподіл професії правозаступників має історичне коріння, поставлений у залежність від характеру функцій, виконуваних баристерами та соліситорами. Баристери є пере­важно адвокатами, що виступають в судах усіх інстанцій. Соліситори мають обмежене право на публічні виступи в них.

Є й інші важливі відмінності між двома групами цієї професії. Соліситори можуть входити у товариства як партне­ри, баристери ж не можуть. Оскільки соліситори працюють зі своїми клієнтами на договірних засадах, вони можуть вимага­ти винагороди через суд; баристери, винагорода яких розгля­дається здебільшого як гонорар, а не оплата за договором, не можуть цього робити. З цих же ж причин на соліситорів може бути покладена відповідальність перед клієнтами за професій­ну недбалість. Історично склалося так, що баристери звільнені від такої відповідальності. Перед початком судового процесу вони одягають парик і мантію, соліситори перебувають у мантії лише у деяких судах, зокрема, в суді графства, але завжди без парика. Соліситорам клієнти самі доручають ведення справи; баристерам же ж доручається ведення справи тіль­ки через соліситора.

Бути одночасно соліситором та баристером не можна, і хоча в принципі перехід з однієї категорії у іншу не забороне­ний, членство в соліситорах і баристерах фактично є довіч­ним.

Соліситори та баристери не можуть працювати разом в одній фірмі чи навіть в одному приміщенні, допомагати та виконувати один за другого доручення юридичного характеру.

  1. Позитивні та негативні аспекти розподілу функцій між адвокатами у Великобританії.

В Англії професія адвоката ділиться на дві відособлені групи: баристери (адвокати) і солісітори. Кожна з них виконує власні обов’язки, хоча в них є й деякі загальні функції.

Необхідно зазначити, що наявність двох категорій адвокатів не відповідає сучасній світовій практиці й пояснюється в основному історичними причинами, консерватизмом англійської судової системи, складністю ведення судових справ у країнах англосаксонської системи права (необхідністю застосування значної кількості судових прецедентів і відсутністю строгої системи правових актів), а також небажанням баристерів губити привілейоване положення й пов’язані із цим матеріальні вигоди.

Спочатку баристери були випадковими підручними сторін спору, які зі своєї ініціативи давали їм поради і поступово були визнані судами як особи, які можуть бути «у раді» (of counsel) з сторонами спору. Однак згодом право «призивати до судового бар’єра», (calling to the bar), тобто  наділяти кандидатів званням баристера, здійснюється чотирма потужними корпораціями (школами-гільдіями), які відомі під загальним ім’ям «Судових Іннов». Сюди відносять Lincoln’s Inn (Линкольнс-Інн), Middle Temple (Середній Темпль), Inner Temple (Внутрішній Темпль) і Gray’s Inn (Грэйс-Інн). Ці корпорації, очолювані виборними старійшинами, користуються повним самоврядуванням, і зараз фактично не існує парламентського акту, що піддавав би їхньому якому-небудь контролю. Така ситуація є певним мінусом у функціонуванні системи баристерів, адже не вступивши у зазначені корпорації баристерів, особа, яка бажає отримати такий правовий статус, не зможе цього зробити.

Одним з мінусів в діяльності баристерів є те, що баристер у сутності не має інших судових справ, крім тих, які йому дає солісітор, якщо його не залучає до своїх справ Корона або яка-небудь корпорація, тобто в баристера відносно обмежена його адвокатська самостійність. Однак успішно практикуючий баристер може підвищити свій статус. Для цього він повинен звернутися до лорда-канцлера із проханням клопотатися перед королевою про введення його в ранг королівського адвоката. Королівські адвокати — це еліта баристерів, з яких обираються судді Вищого суду та судді графств. Баристерська практика в королівського адвоката повинна бути не менш 10 років. Тому плюсом системи баристерів є те, що кар’єрний зріст професійної діяльності можливий і на суддівський посаді.

Солісітори з’явилися в XVI ст. у зв’язку із судами справедливості й стали займатися клопотаннями (to solisit – клопотати) по справах сторін, яким доводилося подовгу очікувати в прийомних начальників Канцлерського суду. Історично професія солісітора виникла внаслідок злиття аторнеїв при судах загального права, солісіторів при Канцлерському суді, прокторів старих церковних судів і письмоводителів.

Сьогодні солісітор– самостійний адвокат, особисто або в співробітництві з іншими солісіторами консультує своїх клієнтів з юридичних питань. Він веде їхні справи із земельною нерухомістю, складає договори та заповіти, бере на себе керування спадкоємним майном і дає рекомендації з питань оподатковування, страхування, конкуренції й підприємництва. Крім того, солісітори правочні вживати необхідних заходи для початку судового процесу. Отже, головним плюсом в системі солісіторів є те, що ці адвокати є незалежними від адвокатських корпорацій. Однак мінусом в солісіторів є те, що їх допуск до судової ланки є обмеженим.

Королівська юридична комісія, протягом декількох років вивчала всі «за» і «проти» злиття обох адвокатських професій, прийшла до остаточного висновку про збереження існуючої системи. Комісія також перевірила й численні норми «кодексів поводження» баристерів і солісіторів, що обмежують конкуренцію між ними, і підсумувала, що вони перебувають у повній відповідності із суспільними інтересами.

  1. 3агальні положення діяльності адвокатури Німеччини.

Адвокатура в Німеччині є незалежною організацією у системі правосуддя, а адвокати — практично вільними підпри­ємцями, які мають свої контори, наймають службовців. Вони відрізняються від інших бізнесменів тим, що не наділені пра­вом рекламувати свою професійну діяльність, а також не сплачують, так званого, ремісничого податку.

Будь-якого контролю за роботою адвокатів з боку держа­вних органів не існує. Проте він повинен мати репутацію чесної людини бути всебічно ерудованим в галузі права та вміти із знанням справи захищати законні інтереси підзахис­ного або довірителя.

Основним нормативним актом, який закріплює правовий статус адвокатури в Німеччині, є Федеральне положення про адвокатуру 1959 p., що діє з незначними змінами і сьогодні.

Усі існуючі в Німеччині палати адвокатів об’єднуються у Федеральну палату адвокатів.

Кожна місцева палата очолюється правлінням, до складу якого входить сім членів. Проте загальні збори палати можуть збільшити їх число. Члени правління обираються загальними зборами палати. Членом правління може бути обраний адво­кат, якому виповнилося 35 років і що має 5-річний стаж безперервної роботи в адвокатурі. Члени правління палати обираються на чотири роки, причому кожні два роки його склад оновлюється наполовину.

Палата адвокатів, що діє при Федеральному суді, має структуру, аналогічну структурі місцевих палат адвокатів окре­мих земель Німеччини, з тією лише різницею, що звіт про свою діяльність палата адвокатів подає не управлінню, а феде­ральному міністру юстиції.

Що стосується Федеральної палати адвокатів як об’єд­нання усіх адвокатських палат Німеччини, то її структура дещо інша. Ця палата очолюється лише президією і не має правління. Президія обирається на чотири роки на загальних зборах членів Федеральної палати адвокатів. Президія складається з президента, трьох віце-президентів і скарбника палати. Ви­щим органом Федеральної палати адвокатів є загальні збори. Як місцева, так і Федеральна палата адвокатів за зако­ном є корпорацією публічного права, тобто автономною гро­мадською організацією.

Державний нагляд за діяльністю місцевої палати адвока­тів здійснює управління юстиції землі. За діяльністю Федера­льної палати державний контроль здійснює міністр юстиції.

Функції нагляду в обох випадках обмежуються перевірка­ми додержання законів і статуту палати, а також виконання завдань, покладених на конкретну палату.

  1. Правовий статус адвоката у Німеччині, його професійні права та обв’язки.

Згідно німецького положення про адвокатуру адвокат є незалежною особою у сфері правосуддя. Будучи людиною вільної професії, адвокат є ком­петентним консультантом і представником інтересів громадян. Його право виступати з юридичних питань в загальних судах, арбітражних або адміністративних органах обмежується лише федеральним законом.

Федеральний закон про адвокатуру регулює в окремому розділі права і обов’язки адвоката. Так, згідно з § 43 цього закону, «адвокат зобов’язаний сумлінно виконува­ти свої професійні обов’язки, бути гідним поваги і довіри, яких вимагає становище адвоката».

Адвокат, до якого звернулися по юридичну допомогу, а він відмовився її надати, має негайно пояснити причини. Він зобов’язаний відшкодувати шкоду, завдану клієнту немотиво­ваною відмовою.

Адвокат відстороняється від виконання професійних обо­в’язків: якщо його діяльність суперечить принципам високої моралі та права; коли він консультував або представляв у конкретній справі іншу сторону; якщо він брав участь у тій же, вже розглянутій справі, як суддя, арбітр, прокурор або предс­тавник офіційної установи; коли йдеться про тлумачення до­кумента, котрий він склав як нотаріус.

Адвокат не може виступати у загальних або третейських судах як адвокат клієнта, якого він обслуговує у зв’язку з постійними службовими та іншими діловими відносинами.

Особи, які працюють суддями або державними службов­цями, не мають права виконувати обов’язки адвоката. Проте управління юстиції землі може зробити виняток для конкрет­ної особи, якщо це не зашкоджує інтересам правосуддя.

Адвокат зобов’язаний взяти на себе представництво сто­рони в цивільному процесі, якщо він призначений на підставі норм цивільно-процесуального кодексу і закону про суди в трудових спорах або на підставі інших законоположень про тимчасово неоплачуваний захист. Такий же обов’язковий ха­рактер має захист в кримінальних справах, якщо адвокат призначений захисником на підставі норм кримінально-про­цесуального кодексу, закону про порушення громадського порядку або закону про надання міжнародної правової допо­моги.

Адвокат може надавати обвинуваченому свої послуги в будь-якій стадії процесу як на попередньому слідстві і дізнан­ні, так і під час судового розгляду справи. До обов’язків адвоката віднесено й зберігання матеріалів справи протягом п’яти років після виконання доручення.

Одночасно закон надає адвокатові право відмовити кліє­нту у видачі копій документів справи до задоволення претензій по оплаті мита і витрат.

Якщо адвокат неспроможний протягом понад тиждень виконувати професійні обов’язки, або має намір цього ж строку бути відсутнім у конторі, федеральне положення про адвокатуру зобов’язує його потурбуватися про заступника. Останній має ті ж повноваження, що й адвокат, якого він заступає.

Окремі обов’язки адвоката пов’язані з його відносинами з практикантом. Так, перший зобов’язаний наставляти другого як виконувати поставлені перед ним завдання, надавати ста­жисту різнобічну допомогу оволодівати професійними навич­ками.

Федеральний закон про адвокатуру надав адвокату право ознайомлюватися зі своєю особовою справою. При цьому він може робити витяги з документів або знімати з них копії.

Адвокат, призначений в суд процесуальним уповноваже­ним, може передати представництво іншому адвокату.

  1. Спеціалізовані адвокати Німеччини.

Правове положення адвоката визначається, крім законів загального характеру й процесуальних законів, декількома спеціальними актами, зокрема, Федеральним положенням про адвокатів (Bundesrechtsanwaltsordnung) і Федеральне положення про адвокатські збори (Bundesgebuhrenordnung fur Rechtsanwalte). Адвокат, як і суддя, є правоохоронним органом (Organ der Rechtspflege). Це значить, що він у своїй діяльності зв’язаний нормами права та забезпечує дотримання тільки законних інтересів свого клієнта. Адвокат, діючи в інтересах клієнта, не може порушувати закон. У той же час адвокат — незалежний консультант і представник по всіх правових питаннях у судових органах і інших державних установах, покликаний надавати кваліфіковану юридичну допомогу. Адвокат — особа вільної професії, тобто він, з одного боку, не знаходиться на державній службі і не працює за наймом, а з іншого боку — не є комерсантом. Кожний адвокат має постійне місце здійснення діяльності — канцелярію. При бажанні адвокати можуть поєднуватися й працювати в рамках спільної канцелярії. Вони можуть також поєднуватися із представниками суміжних вільних професій: нотаріусами, податковими консультантами, аудиторами, патентними повіреними, а також із закордонними колегами.

Основні завдання адвоката складаються в поданні й забезпеченні дотримання законних інтересів клієнта. У першу чергу, це означає, що адвокат повинен об’єктивно розглянути ситуацію клієнта, дати їй правову оцінку й зважити шанси на успіх, передбачити можливі аргументи противної сторони й підготувати заперечення на них, і намагатися всіма можливими в рамках закону способами забезпечити досягнення сприятливого для клієнта результату. При цьому адвокат утримується від здійснення дій, які могли б завдати шкоди клієнтові; він також зобов’язаний зберігати професійну таємницю, а також постійно вчитися й підвищувати професійний рівень щоб мати можливість надавати клієнтам кваліфіковану допомогу.

Правовий статус спеціалізованих адвокатів визначає Положення про спеціалізованих адвокатів (Fachanwaltsordnung). Розподіл адвокатів відбувається за принципом локалізації, тому кожний адвокат може бути допущений тільки до певного суду загальної юрисдикції, обраному за його розсудом. По цивільних справах адвокати мають право виступати тільки в тому земельному суді або Вищому суді землі, у яких вони допущені до адвокатської практики, і їхня участь при розгляді справ у цих судах є обов’язковим. У Верховному федеральному суді й у вищих судах земель можуть виступати тільки адвокати, що мають акредитацію при цих судах.

  1. Юридична освіта та допуск до адвокатської діяльності у Німеччині.

До адвокатської діяльності у Німеччині допускається лише та особа, яка згідно з Законом про суддів виз­нається придатною до виконання суддівських обов’язків. Це означає, що особа має прослухати на юридичному факультеті вісім семестрів курсу правових наук і скласти державні іспити. Потім випускник практикує в усіх установах судової системи (суді, прокуратурі, нотаріаті, адвокатурі) як стажист-службовець протягом 2—3 років. Заро­бітну плату в цей період він отримує з фонду земельних органів влади. Після закінчення практики йому необхідно скласти другий державний іспит і лише після цього він обирає, ким бути: суддею, адвокатом або консультантом фірми з юри­дичних питань. Від цієї процедури звільняються лише ординарні профе­сори права університетів Німеччини.

Особа, яка відповідає вказаним вимогам, звертається з заявою до управління юстиції адміністративно-територіальної одиниці ФРН — землі, де вона постійно проживає. До прий­няття рішення управління юстиції зважує на думку правління тієї палати адвокатів, на території якої ця особа має намір практикувати. Правління палати зобов’язане негайно скласти експертний висновок про кандидата. Якщо протягом двох місяців палата його не подасть або не повідомить про причини . затримання, то управління юстиції вправі вважати, що у пра­вління палати відсутні підстави для відмови кандидату у доз­волі займатися адвокатською практикою (п.п. 1—4, 8 Федера­льного закону про адвокатуру).

Управління юстиції може зобов’язати претендента подати медичний висновок про стан здоров’я.

Закон передбачає випадки, коли особі, яка звернулася з клопотанням, може бути відмовлено у дозволі займатися адво­катською практикою. Ними є: якщо претендент за рішенням Федерального конституційного суду позбавлений одного з будь-яких основних прав; якщо при засудженні кримінальним судом він позбавлений права обіймати посади в державному апараті; якщо його виключено з адвокатури на підставі рішен­ня суду, коли від дня набрання цим рішенням чинності ще не минуло вісім років; якщо претендент на підставі судового рішення усунутий з посади судді у зв’язку з порушенням конституційних принципів або у дисциплінарному порядку звільнений зі служби в органах правосуддя; якщо вчинив проступок, в силу якого він не може займатися адвокатською діяльністю; якщо претендент протиправним способом висту­пає проти демократичного суспільного ладу; якщо претендент внаслідок фізичних або розумових вад, а також будь-якої ганебної пристрасті не в змозі належним чином виконувати функції адвоката; якщо він займається діяльністю, несумісною з професією адвоката, не рахується з авторитетом адвокатури.

Рішення управління юстиції землі про відмову у дозволі займатися адвокатською практикою, як і негативний відгук палати адвокатів (якщо він став причиною відмови), можуть бути оскаржені до адвокатського суду честі протягом одного місяця від дня вручення відмови. Про допуск до адвокатури претендент одержує від управ­ління юстиції землі свідоцтво. З цього часу особа набуває право зватися «адвокатом» за професією.

Деякі особливості має допуск до адвокатської діяльності у Федеральному верховному суді. До цієї діяльності можуть бути допущені лише особи, обрані комітетом по виборах адво­катів при зазначеному суді. Цей комітет складається з прези­дента і президентів сенатів в цивільних справах Федерального суду, членів президії федеральної палати адвокатів і президії палати адвокатів при Федеральному суді. До списку кандидатів вносяться лише ті адвокати, які досягли 35-річного віку і мають не менше п’яти років стажу безперервної роботи. Виборчий орган доби­рає з списків кандидатів таку кількість адвокатів, що удвічі перевищує число вакантних місць, і передає список відібраних осіб федеральному міністру юстиції, який остаточно вирішує, кому з них віддати перевагу. Адвокат при Федеральному суді не може працювати у іншому суді, крім вищих федеральних судових установ і Федерального конституційного суду.

  1. Гонорарна практика адвокатів у Німеччині.

Гонорар за надання адвокатом юридичної допомоги встановлюється за згодою між ним і клієнтом у межах, встановлених федеральним статутом, адвокатських тарифів. Розмір конкретної винагороди обраховується, як правило, ви­ходячи з оспорюваної цінності майна, або суми укладених угод. Це стосується як судових, так і позасудових справ (нап­риклад, консультацій, оформлення договорів тощо).

Встановлено такі види винагород за адвокатські дії: ве­дення судових процесів (за участь в розгляді справи; за відшу­кання нових доказів, якщо заявлено протест; за мирову угоду, досягнуту під час процесу); за участь в позасудових справах: за оформлення ділових (наприклад, торговельних) операцій; за переговори, проведені з опонентом (діловим партнером); за досягнуту мирову угоду. За складання договорів, що мають громадське значення (наприклад, про створення сумісного підприємства), завжди виплачується подвійна винагорода. За­лежно від-складності справи адвокат може вимагати гонорар у повному розмірі або частку від нього.

Відхилення від федерального статуту адвокатських тари­фів допускається лише у двох випадках: коли адвокат укладає із своїм клієнтом письмову угоду, обумовлену більш вищим розміром гонорару, ніж довелося б йому платити за тарифом;

при тривалому консультаційному процесі допускається пого­динна винагорода. В цьому разі адвокату оплачується фактич­но витрачений час. «Вартість» справи в даному випадку не відіграє будь-якої ролі. Адвокат може також вимагати відшко­дування, пов’язаного з поштовими і телефонними витратами, відрядженнями тощо.

Частину свого заробітку адвокати передають у спеціаль­ний фонд, призначений для соціального забезпечення адвока­тів та їх сімей.

Отже, описаний порядок організації адвокатури Німеччи­ни дає уявлення про її правовий статус та її вплив на забезпе­чення прав громадян.

Для корпорації адвокатів Німеччини характерним є також жорсткий порядок допуску до її системи і досить вимогливий перелік обов’язків, що загалом є запорукою високого профе­сіоналізму її представників.

Окремо слід підкреслити, що професія адвоката в Німеч­чині не відокремлюється від професій судді, прокурора, нота­ріуса — вона є різновидом єдиної системи правосуддя.

Говорячи про адвокатуру Німеччини як незалежної органі­зації, слід мати на увазі, що державні органи зберігають ряд елементів нагляду за її діяльністю. Зокрема, органи юстиції, як федеральні, так і земель, наділені в даному разі певною компе­тенцією. Так, за діяльністю Федеральної палати адвокатів держа­вний контроль здійснює .міністр юстиції, а за діяльністю місце­вих палат — управління юстиції. Наприклад, міністерство юсти­ції має право встановлювати межі тарифів оплаті юридичної допомоги.

Управління юстиції земель наділені повноваженнями ви­рішувати долю заяв громадян, що виявили бажання «присвяти­ти себе адвокатській діяльності, вимагати щорічні звіти від правлінь палати адвокатів про зміст її діяльності та ін. І хоча зазначені функції нагляду обмежуються лише контролем за додержанням законів і статутів, це вряд чи відповідає природі адвокатури як вільної професії.

  1. Правове регулювання адвокатської діяльності Франції.

Сучасна адвокатура Франції пережи­ває період свого реформування при збереженні багатих традицій минулого. Першого січня 1992 p. набрали чинності поправки до Ос­новного закону про статус адвокатів № 71-1130 від 31 грудня 1990 p., мета яких реформувати деякі судові та інші юридичні професії.

Головна особливість реформи — створити нову професію, члени якої носитимуть звання адвоката (ст. 1 закону № 71-1130 у новій редакції). Проте це не означає будь-яких радика­льних перетворень. Усі основні традиції французької адвока­тури зберігаються. Справа в тому, що коли раніше адвокати, об’єднані у «ордени адвокатів» (далі ми використовуватимемо звичний, хоч і умовний термін, — «колегії адвокатів»), працю­вали окремо від своїх колег — юрисконсультів, які реєструва­лись окремо, то з початком реформи останні за бажанням могли стати членами адвокатських колегій, одержавши право на здійснення саме адвокатської діяльності, а адвокати, збері­гаючи свій статус, можуть виконувати функції, які раніше були властиві тільки професії юрисконсульта.

За законом, професія адвоката визначається як лібераль­на та незалежна. Це визначення створює право на специфічні організаційні форми здійснення професійної діяльності, охоронюваної законом та відокремленої від держави, що є ефек­тивним засобом забезпечення самоврядування, незалежності та захищеності. Віднині вони доповнили свою традиційну боротьбу за права та свободи людини діяльністю по захисту економічних прав та свобод.

  1. Структура адвокатури Франції та організаційні форми її діяльності.

Структура адвокатури Франції виглядає наступним чином.

Адвокати Франції об’єднані по судових округах, в сфері діяльності кож­ного трибуналу вищої інстанції. Не­залежне професійне самоврядоване об’єднання адвокатів зве­ться орденом адвокатів. На його чолі стоїть обраний таємним голосуванням строком на два роки голова, який репрезентує орден (колегію) у відносинах з судовими та державними орга­нами, громадськістю. Він має дисциплінарні й арбітражні (спори між адвокатами) повноваження, керує службами орде­ну, стежить за професійною підготовкою і перепідготовкою адвокатів і головує в раді, яка обирається таємним голосуван­ням членами колегії на три роки. Члени ради здійснюють щоденне керівництво різнобіч­ними справами ордену. Так, паризький орден має такі комісії: з деонтології, з розвитку, коштів, по зв’язках із засобами масової інформації; а також комісії в кримінальних справах, з прав людини, міжнародну, економічну і соціальну по Спіль­ному ринку.

Близько 19 тис. французьких адвокатів об’єднано в 180 колегій, різних за кількістю членів. Закон охороняє незалеж­ність колегій, створює умови для успішного функціонування, оскільки інститут адвокатури в цілому та її колегії розглядаю­ться у Французькій республіці, як один з важливих елементів правової держави. Наприклад, паризький орден, заснований в XVII столітті, складається з 7 тис. професіоналів, без яких немислимими є суспільне й економічне життя великого регіо­ну. Це бюджет, який обчислюється приблизно в 100 млн. франків, 27 тис. безплатних консультацій на рік для населен­ня, це 9,5 тис. кримінальних справ, у яких адвокати беруть участь за призначенням, це 12 допоміжних служб, які перебу­вають в розпорядженні адвокатів, це центри підготовки і дослідні центри, які їх обслуговують, та багато іншого, що в цілому складає імідж шанованого суспільного інституту, який служить законним інтересам людини.

Система самоврядування адвокатів Франції доповнюєть­ся на загальнонаціональному рівні національною Радою коле­гій адвокатів, яка користується правами юридичної особи та представляє професію адвоката в державних органах і стежить за додержанням норм та звичаїв у адвокатській діяльності. Рада обирається за двоступеневою системою адвокатами Франції. Вона займається узгодженням навчальних програм, координує роботу центрів по підготовці адвокатів, встановлює загальні умови визначення їх спеціалізації, розподіляє кошти, виділені на зазначені цілі.

Організаційні форми діяльності адвокатури Франції виглядають наступним чином.

У зв’язку з реформою французькі адвокати одержали можливість обирати форму організації праці. Поч­немо з традиційних. Частина адвокатів може працювати інди­відуально, як фізичні особи. У індивідуала, зокрема, є можли­вість організувати товариства з обмеженою відповідальністю. До цього слід додати, що адвокат в силу тих чи інших причин (наприклад, відсутність коштів на придбання власності у виг­ляді контори) може укласти контракт про співпрацю з іншим адвокатом або групою адвокатів.

Виключенням з режиму ліберальної професії (введеного в 1940 p.) є наявність дозволу працювати за наймом у іншого адвоката (групи адвокатів). В цьому випадку укладається тру­дова угода. Адвокат, що працює за наймом, зберігаючи осно­вні права, не має права на власну клієнтуру. На адвоката-най­мача покладено цивільно-правову відповідальність за про­фесійну діяльність найманого адвоката, а останній при виконанні своїх функцій зобов’язаний нагадувати, що він працює за наймом у певного адвоката.

Групові форми роботи адвокатів є надзвичайно різнома­нітними. Число варіантів збільшилося після реформи 1992 р.

Традиційно найбільш поширеними є асоціації і цивільно-професійні товариства. Асоціація — це об’єднання адвокатів, кожний з яких персонально відповідає перед своїм клієнтом. Права члена асоціації мають особовий характер і не можуть передаватися. Члени асоціації повною мірою зберігають свою правосуб’єктність. Угода про їх створення укладається в пись­мовому вигляді і доводиться до відома Ради ордену адвокатів. Дуже поширеною формою групового об’єднання є цивільно-професійні товариства (ЦПТ). ЦПТ вносяться до списків об’єднань адвокатів і підлягають реєстрації як юридичні осо­би.

Крім розглянутих вище, є способи об’єднання не для «повного здійснення професійної адвокатської діяльності», а лише об’єднання з метою «визначення матеріальних коштів»;

для полегшення здійснення цієї діяльності. Такими є «цивіль­не товариство з об’єднаними коштами» чи, наприклад, «гру­пова адвокатська контора», зв’язки та взаємозалежність між членами яких є значно менш жорсткішими.

Ще до реформи представникам деяких ліберальних про­фесій (наприклад, юридичним консультантам) було дозволено здійснювати свою діяльність у іншій формі. Згідно з законом № 1258 від 31 грудня 1990 р. адвокати нової професії (мається на увазі їх консолідація з юридичними консультантами) мають право створювати товариства комерційного типу.

  1. Умови необхідні для допуску до адвокатської діяльності у Франції.

Щоб стати адвокатом у Франції, необхідно або мати громадянство Франції або гро­мадянство однієї з країн Спільного ринку за умов взаємного визнання дипломів про вищу освіту. Іноземці мають відповідати ряду особливих вимог та пройти перевірку.

Особа, що претендує на звання адвоката, не може бути засуджена за діяння, несумісні з гідністю і порядністю, а також за серйозні дисциплінарні чи адміністративні проступки, чи покарана у зв’язку з банкрутством; не може вона займатися й діяльністю, що є несумісною з професією адвоката (наприк­лад, бути підприємцем).

Кандидат на посаду повинен: мати вищу юридичну освіту (диплом магістра права), скласти вступні іспити (два письмо­вих та один усний) в одному з регіональних центрів професій­ної підготовки, які працюють під егідою адвокатури та вищої школи, провчитися там рік, засвоїти теоретичний курс і прой­ти практичне стажування, скласти іспити (один письмовий і три усних). При додержанні цих умов, молодий юрист прий­мається до ордену адвокатів (колегії) та виголошує присягу такого змісту: «Присягаю, як адвокат, виконувати свої функції з гідністю, сумлінно, незалежно, чесно і гуманно». Після цього молодий адвокат проходить ще дворічне стажування зі спеціальності і одержує посвідчення. Лише після цього його ім’я вноситься до списків ордену як повноправного члена.

Колегії адвокатів докладають чималих зусиль для органі­зації високоякісної теоретичної підготовки (один рік), стажу­вання (два роки), а потім й перепідготовки адвокатів, що практикують. Так, в Парижі функціонує обладнаний за остан­нім словом науки і техніки центр професійної підготовки, бюджет якого складає приблизно 20 мільйонів франків на рік (дві третини дає колегія, одну третину — держава). Вітриною професії називають утримуваний Паризьким орденом адвока­тів інститут перепідготовки.

  1. Оплата правової допомоги адвокатів у Франції, надання безоплатної правової допомоги.

Оскільки професія адвоката віднесена законом до «незалежних і лібера­льних», втручання у фінансові від­носини адвоката та його клієнта значно обмежені. В їх основу покладений принцип двосторонньої добровільної угоди. Стат­тя 10 закону про статус адвокатів в редакції від 10 липня 1991 р. передбачає, що тарифи на складання цивільно-процесуаль­них документів та участь в процесуальних діях встановлюють­ся цивільно-процесуальним законодавством. Гонорари ж за консультації, допомогу в суді, надання порад, складання пра­вових документів, що не потребують посвідчення, та за виступ в дебатах, встановлюються за домовленістю з клієнтом.

При відсутності згоди з останнім гонорар визначається за звичаєм, з урахуванням матеріальної забезпеченості клієнта, складності справи, витрат адвоката тощо. Заборонено встано­влювати розмір гонорару залежно від результатів судової спра­ви. Проте не забороняється до основного гонорару доплачува­ти додатковий в разі позитивного для клієнта рішення суду.

Наведемо приклад визначення розміру гонорару за внут­рішнім регламентом колегії адвокатів судового округу Верх­ньої Сени: це час, відданий справі, характер, складність, гро­мадське значення інтересів, що захищаються, витрати в справі, зроблені конторою, де працює адвокат,  характер спеціалізації адвоката, і нарешті виграш, одержаний клієнтом. Рада ордену адвокатів може щороку встановлювати орієнтовні критерії винагороди.

Проте іноді виникають спори щодо гонорарів між адво­катом та клієнтом. У цьому випадку питання розв’язується керівництвом ордену адвокатів.

Багато часу та енергії віддають французькі адвокати на­данню безплатної юридичної допомоги населенню. Так, нап­риклад, в Парижі приблизно 27 тисяч жителів щороку одержує такі консультації. Виникає чимало проблем й у наданні безо­платної або пільгової правової допомоги. Так, число осіб, які користуються нею у цивільних справах, постійно зростає. Однак, винагорода в цих випадках є досить скромною і вип­лачується з чималим проміжком у часі. У деяких департамен­тах Франції, де допомога надається на пільгових засадах, вона складає 50% усіх цивільних справ.

Сьогодні адвокати ставлять питання про перегляд ставок винагороди і створення таких механізмів, які виключали б затримку оплати і неузгодженість дій колегій та держави. Пропонується також створити спеціальний фонд оплати праці адвокатів, які допомагають тим, хто має право на пільги. Такий фонд при розумному управлінні може дати прибуток.

  1. Порівняльно-правовий аналіз основних інститутів адвокатури у країнах Євросоюзу.

Для порівняння візьмемо інституту оплати праці та дисциплінарного провадження.

Великобританія. Дисциплінарне провадження. Згідно з законом 1974 p. дисциплінарне провадження покладено на Дисциплінарний трибунал в справах солісито­рів, який є незалежним від Юридичного товариства органом. Він складається з лорда — хранителя судових архівів, з членів-соліситорів з стажем роботи у цій якості не менше десяти років та членів-непрофесіоналів, котрі не можуть бути ні соліситорами, ні баристерами. Трибунал є судовим органом. Своє рішення він виносить у складі трьох членів, включаючи двох соліситорів та одного непрофесіонала. Трибунал може заслуховувати показання сві­дків, які даються під присягою, примушувати їх явкою в засідання за повісткою. Він може, також за згодою лорда—хра­нителя судових архівів, сам виробляти правила, що регулюють його власний судовий процес. Дисциплінарний трибунал може застосовувати такі санк­ції: виключити соліситора із списку, усунути від практики (як правило, на період не більше 5 років); штрафувати (не більше ніж на три тисячі фунтів стерлінгів); оголосити догану.

Оплата праці. За свої послуги соліситори отримують винагороду. Оплата юридичної допомоги здійснюється як за зго­дою, так і за таксою послуг за порядком, встановленим зако­нами та актами, котрі, як правило, видаються у інтересах громадян та контролюються судами. Так, закон 1974 р. надає соліситору право укладати з клієнтом письмову угоду, яка регулює розмір винагороди в справах, пов’язаних із судовим спором. В даному випадку розмір винагороди залежить від того, є справа спірною чи ні. Як відмічає англійський профе­сійний юрист Рональд Уолкер, англійське право завжди стави­лося до такого роду гонорарів як до незаконних через те, що вони суперечать публічним інтересам. При відсутності угоди соліситор може подати звіт, який містить або докладний пере­лік усіх видів виконаної роботи, або загальну суму видатків. Оплата послуг за таксою передбачає порядок, за яким витрати соліситора оцінюються як за твердою шкалою, так і за максимальними та мінімальними сумами.

Італія. Дисциплінарне провадження. Дисциплінарне провадження проти адвокатів здійснюється дисциплінарними судами, які створені при ра­дах асоціацій адвокатів і прокурорів. Дисциплінарні суди роз­глядають справи на адвокатів, що допустили дисциплінарний проступок. На адвокатів, які його вчинили, можуть бути накладені такі стягнення: попередження, осуд, заборона займатися адво­катською діяльністю від двох місяців до одного року, скасу­вання членства в асоціації (з правом поновлення через рік), виключення з асоціації (з правом поновлення через п’ять років). Адвокат, на якого накладено дисциплінарне стягнення, має право звернутися з апеляцією до Національної спілки адвокатів. У випадку виявлення з боку адвоката зловживань, омани, шахрайства і т.д. він підлягає кримінальній відповідальності.

Оплата праці. Оплата праці адвоката здійснюється відповідно до професійних тарифів попередньо погоджених з Націона­льною спілкою адвокатів, які переглядаються і затверджуються раз на два роки міністерством юстиції. Тарифи залежать від роду занять адвоката або прокурора (для останнього тариф встановлюється менший) залежно від того, у якій категорії справ він спеціалізується (кримінальних чи цивільних), безпосередньо в суді чи дає консультації. Під­вищений тариф встановлюється для адвокатів, які працюють у вищих судових інстанціях. Тарифи застосовуються лише в тих випадках, коли між сторонами не було укладено попередньої угоди про конкретну суму винагороди. Якщо така угода була досягнута, сторони зобов’язані її додержувати. Адвокату (прокурору) категорично заборонено домовля­тися з клієнтом про отримання певного процента від суми, яку суд міг би стягнути на користь клієнта.

Німеччина. Дисциплінарне провадження. Дисциплінарне провадження проти адвокатів здійснюється судом честі, який створюється при кожній палаті адвокатів. Він складається з голови, декількох головуючих і членів, які призначаються з числа адвокатів управлінням юс­тиції строком на чотири роки (після попереднього погодження з правлінням палати адвокатів). Члени суду честі протягом своєї діяльності мають статус професійних суддів, хоча на цій посаді вони працюють на громадських засадах і не можуть бути одночасно членами правління палати адвокатів або займатися при ній адвокатською діяльністю. Контроль за судом честі здійснює управління юстиції землі. Щодо завинилих адвокатів судом честі можуть застосову­ватися такі дисциплінарні заходи впливу: попередження, дога­на, штраф у розмірі 50 тис. марок, заборона працювати пред­ставником або захисником у певних галузях правосуддя про­тягом від одного до п’яти років, виключення з адвокатури. Заходи впливу у вигляді догани і грошового штрафу можуть бути застосовані одночасно.

Оплата праці. Гонорар за надання адвокатом юридичної допомоги встановлюється за згодою між ним і клієнтом у межах, встановлених федеральним статутом, адвокатських тарифів. Розмір конкретної винагороди обраховується, як правило, ви­ходячи з оспорюваної цінності майна, або суми укладених угод. Це стосується як судових, так і позасудових справ (нап­риклад, консультацій, оформлення договорів тощо). Встановлено такі види винагород за адвокатські дії: ве­дення судових процесів (за участь в розгляді справи; за відшу­кання нових доказів, якщо заявлено протест; за мирову угоду, досягнуту під час процесу); за участь в позасудових справах: за оформлення ділових (наприклад, торговельних) операцій; за переговори, проведені з опонентом (діловим партнером); за досягнуту мирову угоду. За складання договорів, що мають громадське значення (наприклад, про створення сумісного підприємства), завжди виплачується подвійна винагорода. За­лежно від складності справи адвокат може вимагати гонорар у повному розмірі або частку від нього.

Франція. Оплата праці. Оскільки професія адвоката віднесена законом до «незалежних і ліберльних», втручання у фінансові від­носини адвоката та його клієнта значно обмежені. В їх основу покладений принцип двосторонньої добровільної угоди. Стат­тя 10 закону про статус адвокатів в редакції від 10 липня 1991 р. передбачає, що тарифи на складання цивільно-процесуаль­них документів та участь в процесуальних діях встановлюють­ся цивільно-процесуальним законодавством. Гонорари ж за консультації, допомогу в суді, надання порад, складання пра­вових документів, що не потребують посвідчення, та за виступ в дебатах, встановлюються за домовленістю з клієнтом. При відсутності згоди з останнім гонорар визначається за звичаєм, з урахуванням матеріальної забезпеченості клієнта, складності справи, витрат адвоката тощо. Заборонено встано­влювати розмір гонорару залежно від результатів судової спра­ви. Проте не забороняється до основного гонорару доплачува­ти додатковий в разі позитивного для клієнта рішення суду.

  1. Система юридичної освіти у Франції. Допуск до адвокатської діяльності.

Система юридичної освіти у Франції. У Франції після подій 1968 року поділ університетів на факультети скасовано, замість цього створений ряд спеціалізованих навчальних центрів (на базі Паризького університету — 13 навчальних центрів, у тому числі 6 — юридичних спеціальностей). Відповідно до нової системи французької вищої освіти, недавно прийнятої під впливом Болонского процесу, підготовка юристів проходить у три етапи. Перший етап триває три роки (6 семестрів) і завершується одержанням ступеня лисанс (Lіcence). Якщо перевести цей ступінь на англосаксонську модель освіти, то вона буде відповідати бакалаврові. По закінченню наступного дворічного щабля підготовки (4 семестри) присвоюється ступінь магістра (Master). На доктора юриспруденції (Doctorat) додатково навчаються ще три роки й більше.

Перші три роки навчання в ліценциатурі формують загальні базові знання по діяльності юридичної системи в цілому. Наприклад, у знаменитій Сорбонні, іменованої також Париж 1, першокурсники вивчають цивільне, суспільне, міжнародне суспільне й приватне право, право й системи оподатковування в різних суспільствах, адміністративне право, історію розвитку основних правових систем.

Безпосередні спеціалізації починаються з магістратури. При цьому, якщо на перший рік магістратури приймають всіх студентів, що одержали лисанс по праву, то на другому курсі можливий поділ на магістратуру професійну (masters professіonnels) і магістратуру дослідницьку (masters recherche). Перша орієнтована на практичну професійну роботу, а дослідницька магістратура націлена на проведення наукових вишукувань у певній області права.

До основних галузей права, у яких спеціалізуються магістранти, відносяться: адміністративне, цивільне, порівняльне, конституційне, сімейне, карне, міжнародне, комерційне, податкове, процесуальне, в області охорони здоров’я, засобів масової інформації, прав людини, трудових відносин, охорони навколишнього середовища, приватної власності.

Допуск до адвокатської діяльності. Щоб стати адвокатом у Франції, необхідно або мати громадянство Франції або гро­мадянство однієї з країн Спільного ринку за умов взаємного визнання дипломів про вищу освіту. Іноземці мають відповідати ряду особливих вимог та пройти перевірку.

Особа, що претендує на звання адвоката, не може бути засуджена за діяння, несумісні з гідністю і порядністю, а також за серйозні дисциплінарні чи адміністративні проступки, чи покарана у зв’язку з банкрутством; не може вона займатися й діяльністю, що є несумісною з професією адвоката (наприк­лад, бути підприємцем).

Кандидат на посаду повинен: мати вищу юридичну освіту (диплом магістра права), скласти вступні іспити (два письмо­вих та один усний) в одному з регіональних центрів професій­ної підготовки, які працюють під егідою адвокатури та вищої школи, провчитися там рік, засвоїти теоретичний курс і прой­ти практичне стажування, скласти іспити (один письмовий і три усних). При додержанні цих умов, молодий юрист прий­мається до ордену адвокатів (колегії) та виголошує присягу такого змісту: «Присягаю, як адвокат, виконувати свої функції з гідністю, сумлінно, незалежно, чесно і гуманно». Після цього молодий адвокат проходить ще дворічне стажування зі спеціальності і одержує посвідчення. Лише після цього його ім’я вноситься до списків ордену як повноправного члена.

Колегії адвокатів докладають чималих зусиль для органі­зації високоякісної теоретичної підготовки (один рік), стажу­вання (два роки), а потім й перепідготовки адвокатів, що практикують. Так, в Парижі функціонує обладнаний за остан­нім словом науки і техніки центр професійної підготовки, бюджет якого складає приблизно 20 мільйонів франків на рік (дві третини дає колегія, одну третину — держава). Вітриною професії називають утримуваний Паризьким орденом адвока­тів інститут перепідготовки.

  1. Надання адвокатом безоплатної правової допомоги у країнах Європи.

Досліджуючи законодавство країн СНД і Балтії щодо безоплатної правової допомоги, насамперед виділимо дві групи країн залежно від того, чи створено в державі систему безоплатної правової допомоги, зокрема чи прийнято відповідні закони.

У Молдові, Грузії та країнах Балтії прийнято відповідні закони і створено си­стеми безоплатної правової допомоги. Так, у Республіці Молдова діє закон від 26 липня 2007 р. «Про юридичну допомогу, гарантовану державою», у Грузії — за­кон від 19 червня 2007 р. «Про юридичну допомогу», у Литовській Республіці діє закон «Про зміну закону Литовської Республіки «Про гарантовану державою правову допомогу» від 20 січня 2005 р., у Латвійській Республіці — Закон «Про державну правову допомогу» від 17 березня 2005 р., а у Естонській Республіці — Закон «Про державну юридичну допомогу», прийнятий естонським парламентом 28 червня 2004 р.

Інші країни (Азербайджан, Вірменія, Узбекистан, Киргизія, Росія, Україна, Туркменія, Казахстан, Білорусь, Таджикистан) не прийняли окремих законів про правову допомогу і частково врегульовують питання надання безоплатної право­вої допомоги, окрім конституційних норм, нормами законів про адвокатуру (адво­катську діяльність), кримінально-процесуальних кодексів та правовими актами органів виконавчої влади.

Так, у Азербайджанській Республіці надання правової допомоги за рахунок держави регламентовано статтею 20 Закону Азербайджансь­кої Республіки «Про адвокатів і адвокатську діяльність» від 28 грудня 1999 p.: «Надання адвокатами правової допомоги затриманим по кримінальних справах та малозабезпеченим особам, які потребують правової допомоги в суді, здійснюється без обмежень за рахунок держави відповідно до законодавства Азербайджанської Республіки. Сума і порядок оплати правової допомоги, що на­дається за рахунок держави, встановлюються відповідним органом виконавчої влади. Статті ж 19, 193, 194 Кримінально-процесуального кодексу Азербайд­жанської Республіки зобов’язують дізнавача, слідчого, прокурора або суд вжити заходів для забезпечення права потерпілого, підозрюваного чи обвинуваченого на отримання кваліфікованої юридичної допомоги.

У Республіці Казахстан стаття 4 Закону «Про адвокатську діяльність» визна­чає, що кожна особа є вільною у виборі адвоката, за винятком випадків, коли ад­вокат призначається для надання безоплатної правової допомоги, а також як за­хисник по кримінальних справах, якщо підзахисний не вибрав чи не зміг вибра­ти собі адвоката, а стаття 6 вказаного закону встановлює, що адвокати надають безоплатну правову допомогу для довірителів за їх проханням при веденні справ про стягнення аліментів, про відшкодування шкоди, заподіяної смертю годуваль­ника, каліцтвом чи іншим ушкодженням здоров’я, пов’язаним з роботою, а також учасникам Великої Вітчизняної війни та особам, прирівняним до них, військово­службовцям строкової служби, інвалідам І та II груп, пенсіонерам при наданні консультацій з усіх питань, крім підприємницької діяльності, всім громадянам з питань призначення пенсій, соціальних виплат, реабілітації. Порядок надання ад­вокатами безкоштовної правової допомоги визначається статутом колегії адво­катів. Громадяни, з урахуванням їх матеріального стану, можуть бути звільнені від оплати правової допомоги президією колегії адвокатів, завідувачем юридич­ної консультації, власником (власниками) адвокатської контори та адвокатом, який здійснює свою діяльність індивідуально без реєстрації юридичної особи. Оплата правової допомоги у вказаних справах здійснюється за рахунок коштів відповідно колегії адвокатів, адвокатської контори.

Окремо можна виділити Російську Федерацію, де, хоча й не прийнято феде­рального закону про безоплатну правову допомогу, проте, на виконання вимог Федерального закону «Про загальні принципи організації законодавчих (пред­ставницьких) та виконавчих органів державної влади суб’єктів РФ» від 06 жовт­ня 1999 р., у порядку, визначеному Федеральним законом «Про адвокатську діяльність та адвокатуру в РФ», на місцевому рівні прийнято відповідні закони, наприклад, Закон Волгоградської області від 9 січня 2007 р. «Про внесення змін до Закону Волгоградської області від 17 березня 1998 р. «Про гарантії юридичної допомоги та про поширення правових знань серед населення Волгоградської об­ласті», Закон Красноярського краю від 26 листопада 2004 р. «Про надання безо­платної юридичної допомоги громадянам РФ на території Красноярського краю», Закон Амурської області «Про надання юридичної допомоги громадя­нам РФ на території Амурської області безоплатно».

Так, стаття 26 Федерального закону «Про адвокатську діяльність та адвокату­ру в Російській Федерації» від 31 травня 2002 р. визначає випадки надання гро­мадянам безоплатної юридичної допомоги, а також, покладає на суб’єкти Російської Федерації обов’язок визначити перелік та порядок надання документів для отримання безоплатної юридичної допомоги. Відповідно до вказаної статті закону юридична допомога громадянам Російської Федерації, дохід яких нижчий від розміру прожиткового мінімуму, встановленого законом відповідного суб’єкта Російської Федерації, надається безоплатно позивачам при розгляді в суді першої інстанції справ про стягнення аліментів, відшкодування шкоди, спричиненої смертю годувальника чи іншим ушкодженням здоров’я, пов’язаним з трудовою діяльністю, ветеранам Великої Вітчизняної війни — з питань, не пов’язаних з ко­мерційною діяльністю, громадянам Російської Федерації — при підготовці заяв про призначення пенсій та допомог, з питань, пов’язаних з реабілітацією. Юри­дична допомога надається безоплатно в усіх випадках неповнолітнім, які утриму­ються в установах системи профілактики бездоглядності та правопорушень непо­внолітніх.

Статтею 9 Закону Республіки Узбекистан «Про гарантії адвокатської діяль­ності та соціальний захист адвокатів» від 21 грудня 1998 р. визначено, що затри­маному, обвинуваченому, підсудному і засудженому повинні бути забезпечені не­обхідні умови для безперешкодної і конфіденційної зустрічі та консультації з ад­вокатом, а стаття 11 закону Республіки Узбекистан «Про адвокатуру» від 27 грудня 1996 р. визначає, що при звільненні особи від оплати юридичної до­помоги через неплатоспроможність, юридична допомога адвоката, який бере участь у кримінальному процесі за призначенням, здійснюється за рахунок дер­жави в порядку, встановленому Кабінетом Міністрів Республіки Узбекистан.

Стаття 26 Закону Киргизької Республіки «Про адвокатську діяльність» від 21 жовтня 1999 р. визначає, що в разі відсутності у громадянина коштів юри­дична допомога та захист забезпечуються йому за рахунок держави в порядку та на умовах, визначених Урядом Киргизької Республіки.

У Туркменістані конституційно закріплене право на професійну юридичну допомогу на будь-якій стадії судочинства, яку надають адвокати та інші особи (стаття 106 Конституції), проте законодавство Туркменістану не містить норм, спрямованих на реалізацію права особи на безоплатну правову допомогу.

Отже, за категоріями справ, у яких надається безоплатна правова допомога, можна виділити групу країн, де безоплатна правова допомога надається лише у кримінальних справах, і групу країн, де законодавство визначає обов’язок надава­ти безоплатну правову допомогу й у інших категоріях справ. Так, обов’язок нада­вати безоплатну правову допомогу лише у кримінальних справах законодавець покладає на відповідних суб’єктів у Азербайджані, Вірменії, Узбекистані, Кир­гизії, Україні, Туркменістані. Серед країн, де правова допомога надається, окрім кримінальних і у цивільних та адміністративних справах, — Казахстан, Росія, Білорусь, Молдова, Естонія, Таджикистан, Литва, Латвія, Грузія.

Повертаючись до категорій справ, у яких надається безоплатна правова допо­мога, варто детальніше зупинитися на досвіді країн, де така допомога надається, окрім кримінальних, у окремих цивільних та адміністративних справах (Казах­стан, Росія, Білорусь, Молдова, Естонія, Таджикистан, Литва, Латвія, Грузія).

У цьому контексті варто виокремити досвід Естонії та Латвії, де заявник має право подати клопотання про надання правової допомоги у міжнародному спорі. В цілому досвід Молдови, Грузії та країн Балтїї у створенні систем безоплатної правової допомоги є цікавим для України. Молдова та Литва, створюючи систе­ми безоплатної правової допомоги пішли шляхом її розподілу на первинну і вто­ринну (Литва) та на первинну і кваліфіковану (Молдова). Україна у законопроекті «Про безоплатну правову допомогу» також має намір йти шляхом розподілу бе­зоплатної правової допомоги на первинну і вторинну. Латвія, Естонія та Грузія не ділять безоплатну правову допомогу на первинну і вторинну, проте в Грузії кон­сультація відділяється від правової допомоги і надається всім особам.

У вказаних країнах створено окремий орган, що здійснює управління надан­ням безоплатної правової допомоги. У Молдові це Національна рада з гарантова­ної державою юридичної допомоги, у Литві — Координаційна рада з питань гаран­тованої державою правової допомоги, у Латвії — Управління безоплатної юридич­ної допомоги, в Грузії — Служба юридичної допомоги. Лише в Естонії окремий орган управління наданням безоплатної правової допомоги не створено, а роз­поділено обов’язки з управління наданням безоплатної правової допомоги між міністерствами юстиції, фінансів, Колегією адвокатів, керівництвом адвокатських контор. Причому з прийняттям закону роль Колегії адвокатів у Естонії зросла, а суди, прокуратура та органи слідства є органами, що приймають рішення про не­обхідність надання безоплатної юридичної допомоги.

Законодавство всіх цих країн визначає, що фінансування надання безоплатної правової допомоги здійснюється з державного бюджету, проте, окрім коштів дер­жавного бюджету, на фінансування безоплатної правової допомоги можуть вико­ристовуватися дозволені законодавством інші кошти. Наприклад, у статті 25 Зако­ну Грузії «Про юридичну допомогу» визначено, що джерелом фінансування служби юридичної допомоги є цільові кошти, виділені з Державного бюджету, по­жертвування і гранти, дозволені законодавством Грузії інші доходи.

Законодавство цих країн також передбачає обов’язок особи відшкодувати ви­трати за надану правову допомогу, якщо особа надала неправдиві дані про своє фінансове становище, а в Естонії особа зобов’язана відшкодувати витрати за на­дану правову допомогу, якщо протягом п’яти років її фінансове становище суттєво поліпшилося. Відповідно до вимог статті 23 закону Молдови «Про юри­дичну допомогу, гарантовану державою», особа, яка отримала кваліфіковану юридичну допомогу в результаті надання фальсифікованої чи недостовірної інформації, зокрема про своє фінансове становище, чим ввела в оману тери­торіальне бюро, зобов’язана відшкодувати витрати з надання правової допомоги.

Якщо ж у ході процесу чи виконання судового рішення фінансове становище осо­би змінилось в сенсі повної чи часткової втрати права на кваліфіковану юридич­ну допомогу, особа зобов’язана відшкодувати витрати з надання правової допомо­ги. У Грузії та Молдові в разі винесення судом рішення на користь особи, яка ско­ристалася безоплатною правовою допомогою, оплата витрат за таку допомогу відшкодовується з іншої сторони.

Відповідно до вимог статті 20 конституційного закону Республіки Таджикис­тан «Про адвокатуру» від 4 листопада 1995 р., якщо особа звільнена від оплати правової допомоги прокурором, судом чи органом попереднього розслідування, витрати по оплаті праці адвокатів здійснюються за рахунок місцевого бюджету.

Також Конституційний закон Республіки Таджикистан «Про адвокатуру» від 4 листопада 1995 p., зокрема, стаття 19, визначає, що члени колегії адвокатів на­дають безоплатну юридичну допомогу, окрім позивачів у справах про стягнення аліментів, про відновлення на роботі, учасникам Великої Вітчизняної та афгансь­кої воєн, інвалідам та особам, які втратили годувальника під час громадянської війни в Таджикистані, біженцям, громадянам по скаргах на неправильність у спи­сках виборців, також народним депутатам при наданні консультацій з питань за­конодавства, пов’язаних із здійсненням ними депутатських повноважень. Пре­зидія адвокатів, орган попереднього розслідування, прокурор і суд можуть звільнити особу від оплати юридичної допомоги. При вирішенні цього питання враховується не лише майновий стан особи, а й вік, працездатність, стан здоров’я, наявність неповнолітніх чи непрацездатних дітей та осіб, що перебувають на утриманні.

У Великобританії як і соліситори, баристери надають безоплатну юридичну допомогу малозабезпеченим клієнтам. Багато часу та енергії віддають французькі адвокати на­данню безплатної юридичної допомоги населенню. Так, наприклад, в Парижі приблизно 27 тисяч жителів щороку одержує від членів колегії такі пільгової правові допомоги. Число осіб, які користуються нею в цивільних справах, постійно зростає. Винагорода, яка виплачується у цих випадках адвокату, є досить скромною за розміром і робиться це з великим проміжком у часі. У деяких департаментах Франції справи, де допомога надається на пільгових підставах, іноді складають 50% усіх цивільних справ.

В Україні поки що відсутній закон про безоплатну правову допомогу, і її мож­на віднести до числа держав, що частково врегульовують питання надання безо­платної правової допомоги, окрім конституційних норм (статті 29, 59 Конституції України), нормами законів про адвокатуру (стаття 12 Закону України «Про ад­вокатуру» від 19 грудня 1992 р.), нормами Кримінально-процесуального кодек­су (статті 21, 47) та правовими актами органів виконавчої влади.

  1. Правове регулювання адвокатської діяльності країн СНД (Азербайджану, Вірменії, Казахстану, Таджикистану та Узбекистану)

У більшості країнах СНД право людини на отримання правової допомоги закріплено у конституціях, а можливість його реалізації певною мірою кон­кретизована у відповідних законах. Кожна з країн СНД так чи інакше прагне до реалізації права людини на отримання правової допомоги і має особли­вості формування цього інституту, тому аналіз законодавства цих країн є цікавим для України, як однієї з країн, що знаходиться на шляху створення системи пра­вової допомоги.

Практично у всіх розглянутих державах (Азербайджані, Вірменії, Казахстані, Таджикистані та Узбекистані) адвокатура стала предметом регулювання законодавчих актів. У Вірменії, Азербайджані порядок здійснення адвокатської діяльності визначається тільки законами, серед яких ЦПК і КПК (ст.2). У Казахстані встановлено, що законодавство про адвокатську діяльність складається із закону РК «Про адвокатську діяльність» і іншого законодавства, що регулює адвокатську діяльність. Однак процесуальні права й обов’язки адвокатів при здійсненні ними захисти й представництва по справах фізичних і юридичних осіб установлюються тільки законами. У Таджикистані правові основи юридичної допомоги, діяльність адвокатур регламентуються разом із законами також парламентськими постановами й указами республіки. Сам же закон про адвокатуру одержав вищу юридичну форму — «конституційний закон». Це єдина держава СНД, що надала закону про адвокатуру такий статус. Звичайно, у ряді конкретних випадків акти більше низького рівня також можуть відповідати завданням адвокатури. Наприклад, Уряд Узбекистану прийняло постанову «Про заходи по соціальному захисту осіб, що займаються адвокатською практикою в адвокатських об’єднаннях Республіки Узбекистан» від 15 липня 1996 року N 250, що визначило особливості діяльності адвокатських формувань як добровільних професійних об’єднань осіб, що займаються адвокатською практикою. З метою забезпечення їхньої ефективної діяльності й соціальної захищеності встановлено, що адвокатські об’єднання вносять страхові внески у фонд соціального страхування в розмірі 7 проц. от фонду заробітної плати після втримання з їхніх сум на загальні потреби адвокатських формувань. Законодавство про адвокатуру в Руз включає як закони, так і «інші акти законодавства» (ст.2). У цілому ж правомірно зробити висновок у тім, що ця сфера суспільних відносин має певні межі відомчої регламентації.

  1. Поняття та зміст адвокатської діяльності за законодавством країн СНД.

У рамках огляду поняття «адвокатська діяльність» звернемо увагу на те, що в законах ряду держав СНД повноваженням здійснювати захист наділяється тільки адвокат. Це, наприклад, закріплене в ст.5 Закону РА, у ст.4 Закону РК, відповідно до якої професійний захист по кримінальних справах здійснюється тільки адвокатами, у ст.4 Закону АР — «захисниками на судовому процесі по кримінальних справах, попередньому слідству або в дізнанні відповідно до  процесуального законодавства АР можуть бути тільки адвокати». Положення законів Руз не містять таких уточнень. Існує однозначне розуміння, що тільки адвокати можуть надавати будь-яким особам будь-яку юридичну допомогу, у тому числі в рамках судового засідання, а юрисконсульти надають її тільки в рамках тієї юридичної особи, де вони здійснюють свої професійні функції як найманий працівник. У судах же, маючи на увазі систему господарських судів і судів по цивільних справах, вони можуть здійснювати функції лише представництва з дотриманням правил підвідомчості. У Росії в Законі РФ «Про адвокатську діяльність і адвокатуру в Російській Федерації» адвокатська діяльність визначається як кваліфікована юридична допомога, надавана на професійній основі особами, що одержали статус адвоката в порядку, установленому дійсним Федеральним законом, фізичним і юридичним особам з метою захисту їхніх прав, свобод і інтересів, а також забезпечення доступу до правосуддя. В Україні у ЗУ «Про адвокатуру» визначені основні принципи адвокатської діяльності (адвокатура України  здійснює  свою  діяльність на принципах верховенства   закону,  незалежності,  демократизму,  гуманізму  і конфіденційності) та види адвокатської діяльності (адвокати   дають  консультації  та  роз’яснення  з  юридичних питань, усні і  письмові  довідки  щодо  законодавства;  складають заяви,  скарги  та інші документи правового характеру; посвідчують копії  документів  у  справах,   які   вони   ведуть;   здійснюють представництво в  суді, інших державних органах, перед громадянами та юридичними особами; подають  юридичну  допомогу  підприємствам, установам, організаціям; здійснюють правове забезпечення підприємницької та  зовнішньоекономічної діяльності громадян і юридичних осіб, виконують  свої обов’язки   відповідно до кримінально-процесуального законодавства у процесі дізнання та попереднього слідства).

  1. Визначення поняття адвокатури як виду професійної діяльності за законодавством країн СНД.

Згідно з Законом України «Про адвокатуру» адвокатура України є добровільним професійним громадським об’єднанням,    покликаним  згідно   з Конституцією  України сприяти  захисту  прав,  свобод  та представляти законні інтереси громадян України, іноземних громадян, осіб без громадянства, юридичних осіб, подавати їм іншу юридичну допомогу. Адвокат   має   право   займатись   адвокатською   діяльністю індивідуально,  відкрити  своє  адвокатське  бюро, об’єднуватися з іншими адвокатами в колегії, адвокатські фірми,  контори  та  інші адвокатські  об’єднання,  які  діють відповідно до цього Закону та статутів адвокатських об’єднань. Порядок утворення, діяльності,  реорганізації  та  ліквідації адвокатських   об’єднань,   структура,   штати,  функції,  порядок витрачання коштів, права та обов’язки керівних органів, порядок їх обрання та інші питання, що належать до їх діяльності, регулюються статутом відповідного об’єднання.

У Росії в Законі РФ «Про адвокатську діяльність і адвокатуру в Російській Федерації» адвокатура є професійним співтовариством адвокатів і як інститут громадянського суспільства не входить у систему органів державної влади й органів місцевого самоврядування. Формами адвокатських утворень є: адвокатський кабінет, колегія адвокатів, адвокатське бюро і юридична консультація. Адвокат вправі відповідно до  Федерального закону самостійно обирати форму адвокатського утворення й місце здійснення адвокатської діяльності.

В Узбекистані уперше Закон «Про гарантії адвокатської діяльності й соціальному захисту адвокатів» увів ліцензування здійснення адвокатської діяльності і її видів, закріпив принципи, організаційні форми діяльності адвокатури. Так, особі, що одержала у встановленому порядку ліцензію на право заняття адвокатською діяльністю, було дозволено здійснювати її індивідуально, відкриваючи своє адвокатське бюро, або на добровільній основі утворювати з іншими адвокатами (партнерами) колегії й фірми.

Закон РА від 18 червня 1998 року в ст.9 дозволив адвокатам для організації своєї діяльності обирати будь-яку встановлену законодавством організаційно-правову форму роботи.

Законодавець Киргизстану в ст.20 Закону «Про адвокатську діяльність»  від 21 жовтня 1999 року N 114 указав, що адвокатські установи можуть створюватися «на основі будь-яких видів власності й організаційно-правових форм». При цьому засновником адвокатської установи може бути будь-яка юридична й фізична особа. Закон Киргизстану, визначаючи форми діяльності з, дозволяє їм здійснювати адвокатську діяльність або через адвокатські установи, або як індивідуальну підприємницьку діяльність (ст.19). У Законі Таджикистану повірник прямо визначений як «підприємець, що робить юридичну допомогу на підставі ліцензій» (ст.2).

Організація адвокатської діяльності в Казахстані така: адвокат вправі здійснювати свою діяльність: а) через юридичну консультацію; б) заснувати самостійно або разом з іншими адвокатами адвокатську контору; в) індивідуально без реєстрації юридичної особи. Наявність у Законі цієї третьої форми є відмінна риса казахського законодавства про адвокатури.

В Азербайджані адвокати також вільні у виборі організаційно-правової форми здійснення адвокатської діяльності: або індивідуально, або в складі організованих структур, до яким віднесені «юридичні консультації, адвокатські бюро, адвокатські фірми й ін.» (ст.5). Засновниками цієї структури, так само як і в Узбекистані, повинні бути тільки адвокати. Закон Руз більш тонко й правомірно враховує реалії дня, інтереси як самого адвоката, так і осіб, якою він надає юридичну допомогу.

  1. Правовий статус адвоката за законодавством країн СНД.

Згідно з Законом України «Про адвокатуру» адвокатом  може  бути  особа,  яка  має вищу юридичну освіту, підтверджену  дипломом  України  або  відповідно  до   міжнародних договорів  України  дипломом  іншої  країни,  стаж роботи у галузі права  не  менше  двох  років,  володіє  державною  мовою,  склала кваліфікаційні  іспити,  одержала в Україні свідоцтво про право на зайняття адвокатською  діяльністю  та  прийняла  Присягу  адвоката України.  Адвокат  не  може  працювати  в суді, прокуратурі, нотаріаті, органах  внутрішніх  справ, служби безпеки, державного управління. Адвокатом  не  може бути особа, яка має судимість.

У Росії. Законі РФ «Про адвокатську діяльність і адвокатуру в Російській Федерації»  статус адвоката в Російській Федерації вправі набути особа, яка має вищу юридичну освіту, отриману в освітній установі, що має державну акредитацію, вищу професійну освіту, або вчений ступінь по юридичній спеціальності. Зазначена особа також повинна мати стаж роботи з юридичної спеціальності не менш двох років або пройти стажування в адвокатському утворенні в строки, установлені дійсним Федеральним законом.

В Узбекистані. Адвокатом у Республіці Узбекистан може бути громадянин Республіки Узбекистан, який має вищу юридичну освіту й одержав у встановленому  порядку   ліцензію на право заняття  адвокатською діяльністю (далі — ліцензія).

Відповідно до  Закону Бєларусі «Про адвокатуру» адвокатом Республіки Бєларусь може бути громадянин Республіки Бєларусь, який має вищу юридичну освіту, стаж роботи зі спеціальності не менш трьох років або не має такого стажу, але минуле стажування строком від шести місяців до одного року в адвокатурах, який здав кваліфікаційний іспит і одержав спеціальний дозвіл (ліцензію) (далі — ліцензія) на здійснення адвокатської діяльності. Адвокати й стажисти не можуть перебувати на службі в державних, суспільних і інших організаціях, за винятком осіб, що займаються науковою або педагогічною діяльністю.

  1. Членство адвоката у професійних об’єднаннях за законодавством країн СНД.

У ст.4 Закону АР встановлено, що адвокатською діяльністю може займатися особа, прийнята у встановленому порядку членом Колегії адвокатів і що принесла присягу адвоката, а в ст.9 цього ж закону підкреслюються: «Особи, що не є членами Колегії адвокатів, не можуть займатися адвокатською діяльністю». У РК адвокат обов’язково повинен бути членом колегії адвокатів і лише тоді він має право надавати юридичну допомогу на професійній  основі в рамках адвокатської діяльності, регламентованої зазначеним законом (ст.7, 27). У Вірменії членство в союзі адвокатів також є обов’язковою умовою для одержання ліцензії на адвокатську діяльність (ст.11 Закону). Претендент повинен подати в Раду адвокатів заяву із проханням про вступ у нього й прикласти копію свідчення, виданого Державною кваліфікаційною комісією. Особа, що вступила в раду, одержує ліцензію, скріплену печаткою й завірену підписом голови ради. Законодавець Киргизстану підкреслив, що надавати юридичну допомогу на всій території республіки вправі «адвокат, що знаходиться в списку професійного суспільного об’єднання адвокатів». Така обов’язковість членства в професійних громадських організаціях або в колегіях адвокатів не зовсім коректна. Звичайно, адвокати повинні бути членами суспільного об’єднання адвокатів. Однак це не може нав’язувати через адміністративні інструменти або одержання/придбання таким суспільним об’єднанням повноважень ліцензування й т.п.